Boletin Trigonometria 2018-I (Masters)

Descargar como pdf o txt
Descargar como pdf o txt
Está en la página 1de 100

Trigonometría

Ciclo Verano 2018

PROF: BREYNER OROYA CRUZ

CUSCO-PERÚ
Prof: Breyner Oroya Cruz

PRESENTACIÓN
El presente trabajo está dirigido a los estudiantes preuniversitarios que
inician el estudio del curso de Trigonometría.

El objetivo de la obra es, la comprensión de las relaciones existentes


entre los lados y ángulos de un triángulo, y el desarrollo, en los estudiantes, del
hábito de utilizarlos en los diferentes problemas.

El texto consta de 11 capítulos. Cada unidad se divide en tres bloques:


primero, la exposición teórica con ejemplos didácticos; segundo, problemas para
resolver en clase, dosificados en orden creciente de dificultad; tercero, la tarea
domiciliaria.

Espero sinceramente que este texto se constituya en un buen compañero


de trabajo de los estudiantes preuniversitarios.

Gracias.
El Profesor

CONTENIDO TEMÁTICO

Capítulo I Sistema de Medida Angular


Capítulo II Razones Trigonométricas de Ángulos Agudos
Capítulo III Razones Trigonométricas de Ángulos en Posición Normal
Capítulo IV Reducción al Primer Cuadrante
Capítulo V Circunferencia Trigonometrica
Capítulo VI Identidades Trigonométricas
Capítulo VII Razones Trigonométricas Ángulos Compuestos
Capítulo VIII Razones Trigonométricas de Ángulos Múltiples
Capítulo IX Transformaciones Trigonométricas
Capítulo X Resolución de Triángulos
Capítulo XI Funciones Trigonométricas

BOC
Trigonometría 2
La trigonometría es la parte de la geometría   m trigonométrico  
que estudia la relación entre los lados de un
triángulo rectángulo y sus ángulos.
Actualmente, esta idea básica ha sido superada 2. Si se cambia el sentido de rotación de un
y las funciones trigonométricas tienen, ángulo, entonces su medida cambiara de
matemáticamente hablando, sentido propio. signo
B B
La trigonometría es una disciplina fundamental,
tanto para el estudio geométrico, como para el
conocimiento del cálculo y el análisis  
matemático. O O
A A
ÁNGULO TRIGONOMÉTRICO
Observación:
El ángulo trigonométrico es la figura Para realizar operaciones con ángulos
generado sobre un plano por la rotación de un trigonométricos estos deberán estar en el
rayo alrededor de su origen, desde una mismo sentido.
posición inicial hasta una posición final y en un
sentido determinado. De la figura se tiene :
B
Si la rotación se realiza en sentido horario,   (  )  180º
entonces la medida del ángulo será negativa y 
si se realiza en sentido antihorario, entonces la       180º
medida del ángulo será positiva, tal como se
muestra en la figura siguiente: C O A

 Lado  SISTEMAS DE UNIDADES ANGULARES


 
 Final 
B Existen infinidad de sistemas de unidades
Sentido angulares, pues se toma en forma arbitraria la
Origen del rayo m  es ( ) unidad de un sistema. Los sistemas dependen
Antihorario en cuantas partes se divide el ángulo de una
(Vértice)
 A  Lado  vuelta o comparando la longitud del radio con la
O Rayo   longitud de arco que subtiende el ángulo
  Inicial  central.
Sentido Los sistemas más usados son:
m  es ( )
C Horario
1. Sistema Sexagesimal o Inglés (S)
 Lado  La unidad de medida en este sistema es el
 
 Final  grado sexagesimal (1º)

Características:
1. La medida del ángulo trigonometría no se 1vta
encuentra sujeto a restricciones pudiendo 360
ser un ángulo de cualquier magnitud. Se divide 1º
en 360 O
partes iguales 1 vuelta


m 1 vta  360º

Trigonometría 3
Prof: Breyner Oroya Cruz
Unidades Complementarias: Equivalencias:
 Minuto Sexagesimal  (1') 1 g   100 m 1m   100 s 1 g   10000 s
 Segundo Sexagesimal  (1'')

Nota: ag bm c s   ag  bm  c s
Equivalencias: g g m s
1º   60' 1'   60'' 1º   3600'' ab,cdef   ab cd ef

o Ejemplos:
 b c 
Nota: aº b ' c ''  aº  b ' c ''   a   Expresar los siguientes angulos en grados,
 60 3600  minutos y segundos centesimales:

Ejemplos:  12,3456 g  __ g __ m __ s
Expresar los siguientes angulos en grados,  32,752g  __ g __ m __ s
minutos y segundos sexagesimales:
 1,34567 g  __ g __ m __ s
 12,345  __   _, ___   0,0102g  __ g __ m __ s
 '  123,45 g  __ g __ m __ s
12,345  __   _, ___    
    1,973g  __ g __ m __ s
12,345  __   __, _'
 ''  3. Sistema Radial o Circular (R)
12,345  __   __' _, _'   La unidad de medida en este sistema es el
 ' 
radian (1 rad), el cual se define como el ángulo
12,345  __   __' __'' central que subtiende en toda circunferencia un
12,345  __  __' __'' arco de igual longitud que la de su radio.

 73,673  __   _, ___ 
 '
73,673  __   _, ___    
  Lados
73,673  __   __, __' O 1rad del
ángulo
 '' 
73,673  __   __' _, __'  
 ' 
73,673  __   __' __, _''
73,673  __  __' __, _''
m 1 vta  2 rad
2. Sistema Centesimal o Francés (C)
La unidad de medida en este sistema es el Observaciones:
grado centesimal (1g)
1rad   57º 17' 44''  1rad  1º  1g
Aproximación de “  ”:
1vta
  3,1416
400  2 3
Se divide 1g 
22
en 400 O   10
7
partes iguales 1 vuelta
m 1 vta  400 g Equivalencias Fundamentales
Podemos encontrar la siguiente relación entre
los tres sistemas más conocidos:
Unidades Complementarias:
m 1 vta  360º   400g   2 rad
 Minuto Centesimal  (1 m)
 Segundo Centesimal  (1 s)  rad   180º   200g

Trigonometría 4
Prof: Breyner Oroya Cruz
Equivalencia entre sistema sexagesimal y
centesimal. RELACIÓN DE SISTEMAS ANGULARES
9º   10g (Relación de grados)
Si S, C y R son los números que representan la
27'   50m (Relación de minutos) medida “θ” de un ángulo en los sistemas
sexagesimal, centesimal y radial,
81''   250s (Relación de segundos) respectivamente, se tiene:

Factores de Conversión B
Son fracciones equivalentes a la unidad y se
obtienen dividiendo dos cantidades
equivalentes, colocando en el numerador una   Sº  Cg  Rrad
medida en la unidad deseada y en el O
A
denominador se coloca su equivalente en la
unidad a eliminar. S C R S C 20R
  ó  
180 200  9 10 
Ejemplos:
Expresar los siguientes angulos en el sistema Observaciones:
que corresponda:  Para resolver la ecuación y simplificar
  expresiones se recomienda utilizar una
 36o   __ g  36    
  constante “k” de proporcionalidad.
  
 54o   __ g  54o    S  9k
  S C 20R 
   k C  10k
  9 10  
 37o   __rad  37o    
  R  k
 20
 
 60   __  60    
g o g

   Para hallar la variable “R”, se recomienda


  despejar “S” y “C” en función de “R”.
 33   __  33    
g o g

   180R
S
  S C 20R  
 50   __ rad  50     

g g
9 10   200R
  C
 
3 3  
 rad   __ 
o
rad   
4 4    Recordar que S, C y R representan al valor
2 2   numérico del ángulo.
 rad   __ g  rad   
5 5   Sº  Cg  R rad
 
 45'   __  45'   
o

   Para todo ángulo positivo se cumple que:


  CSR
 28m   __ g  28m    
 
 Para grados, minutos y segundos.
 
 23''   __'  23''   
 
Sexagesimales Centesimales
 
 54'   __ m  54'    # de grados S C
 
# de minutos 60S 100C
 
 200m   __'  200m    # de segundos 3600S 10000C
 
 
 162''   __ s  162''  
 

Trigonometría 5
Prof: Breyner Oroya Cruz
xyz 375
E   15
Problemas Resueltos xyz 375

Problema 01: Del gráfico, hallar “y” Problema 04: Se inventan 2 sistemas de
medición angular “x” e “y”, tal que:
25x   50g , además 80y   90º .
Determinar la relación de
yº xº conversión entre estos dos
sistemas x/y.
40º xº yº
Solución 04: Con los datos del enunciado,
tenemos:
Solución 01: Trabajando en el gráfico, tenemos:
25 x   50g  1x   2g ...(1)
(yº xº) 80 y   90º  8 y   9º ...(2)
Dividiendo (1) entre (2), tenemos:
yº xº
1x 2g  9º  1x 1
  g   y 
40º xº yº 8 y
9º  10  8 5
La relación de sistemas es: 5x  8y
xº  yº  xº  yº 40º  180º Por lo cual:
2y º  140 º x y x 5
  
y   70 5 8 y 8

o
 45  Problema 05: Hallar el valor de S.
Problema 02: Convertir  8  a grados,
  Si: S  3x x y C  2(x x  4)
minutos y segundos
sexagesimales. Solución 05: Recordemos que:
S C
Solución 02: Trabajando con la pregunta, 
9 10
tenemos:
o
Reemplazando los datos tenemos:
 45  3x x 2(x x  4)
 8   5,625º  5º 0,625º 
  9 10
 60'  5x  3x x  12
x
 5º 0,625º     5º 37,5'
 1º  2x x  12  x x  6
 60''  Nos piden:
 5º 37' 0,5'  5º 37' 0,5'  
 1'  S  3x x  3(6)  S  18
 5º 37' 30''  5º 37' 30''
Problema 06: Siendo S, C y R lo conocido,
g calcular:
15 m
Problema 03: Si:  x g yz ;
4 CS C  2S C  6S
E  
xyz CS CS CS
Calcular: E 
xyz
Solución 06: Recordar que:
Solución 03: Del enunciado tenemos: S C 20R S  9k 
  k  R k
m 15g 9 10  C  10k 20
x g yz   3,75g
4 Reemplazando en la pregunta, tenemos:
m
x g yz  3g 75m
Reemplazando los valores de x, y e z en la
pregunta tenemos:

Trigonometría 6
Prof: Breyner Oroya Cruz
20  x 20  x
CS C  2S C  6S 
E   9 10
CS CS CS
200  10x  180  9x
10k  9k 10k  18k 10k  54k 20
   x
10k  9k 10k  9k 10k  9k 19
Con lo cual, tenemos:
19 k 28 k 64 k S  20  x C  20  x
  
k k k 20 20
S  20  C  20 
 19  28  8  19  36 19 19
360 400
 19  6  25  5 S C
19 19
Despejando R de (2):
Problema 07: Si: “S” y “C” son las medidas de
S C
un mismo ángulo en grados R ó R
sexagesimales y centesimales 180 200
respectivamente. Hallar la medida Nos piden:
en radianes. Si: 3C  2S  1 S  360   2
R    R
180  19  180 19
Solución 07: Recordar que:
2
 180R Entonces el ángulo mide: rad
S 19
S C 20R   
  
9 10   200R Problema 09:
C Halle la medida en

  radianes, de aquél ángulo tal que la
Reemplazando en la pregunta, tenemos: diferencia de su número de
3C  2S  1 segundos sexagesimales y de su
número de minutos centesimales
 200R   180R 
3   2    1 sea 15700.
    
600R 360R
 1 Solución 09: Trabajando con el enunciado,
  tenemos:
240R  # de segundos   # de minutos 
1     15700
  Sexagesimales   Centesimales 
  3600 S  100 C  15700
R   rad
240 240  180R   200R 
36     157
     
Problema 08: La medida de un ángulo en grados
6280R 
sexagesimales es (20  x)º y en el  157  R 
 40
sistema centesimal (20  x)g .
Calcular la medida de dicho ángulo
en radianes.

Solución 08: Recordar que:


Sº  Cg  R rad …(1)
S C 20R
Además:   …(2)
9 10 
Trabajando con los datos en (1), tenemos:
(20  x)º  (20  x)g
S  20  x ; C  20  x
Reemplazando en la relación (2):

Trigonometría 7
Prof: Breyner Oroya Cruz
05. En el gráfico:
Problemas Dirigidos
Tema: Ángulo Trigonométrico y
01. Señale la relación correcta entre α y β. 35 x

Si y  4x  25 , entonces el valor de “x” es:


A) 30° B) 36° C) 40°
 D) 35° E) 34° CEPRU 2016–II

 06. En la figura, halle la suma del menor y


mayor valor de menor y mayor valor entero
A)     90º B)     90º de “x”.
C)     90º D)     0
E)     90º
y
02. A partir del gráfico, determine “x”. y  2x xy

A) 126° B) 89° C) 86°


D) 62° E) 96°
x
210 Tema: Sistema de Medida Angular
72 11
 rad
07. Determine el valor de: E  5 50
g
300 10
A) 2 B) 3 C) 4
A) 155° B) 150° C) 160° D) 5 E) 6
D) 170° E) 175°
08. La suma de dos ángulos es 90g y su
03. A partir del gráfico, determine “ x  10 ”. diferencia es 19°. Encuentra uno de los
ángulos en sexagesimales.
A) 40° B) 45° C) 82°
D) 31° E) 15°

30  6x 3x  30 09. Las medidas de los ángulos internos de un


x
A) 10° B) 30° C) – 30° triángulo ABC son 3x , xg y rad .
300
D) 20° E) – 10°
¿Cuál es la medida del menor ángulo
interno del triángulo?
04. En el triángulo ABC de la figura. AB  BC . A) 16° B) 24° C) 36°
Calcular:   x   D) 40° E) 72°
B 10. Del grafico hallar “ x  y ”, si: y  5x  15

150
A x  30 g
 C
3x  9 30  2yg
A) 103° B) – 37° C) 72°
D) – 53° E) 105° A) 9 B) – 9 C) 10
D) – 10 E) 12

Trigonometría 8
Prof: Breyner Oroya Cruz
11. Los ángulos iguales de un triángulo 17. Convertir 9000m  900s al sistema
isósceles miden 4(x 3) y 5(x  1)g sexagesimal.
respectivamente. Hallar la medida del A) 814'52'' B) 8042'
ángulo desigual en radianes. C) 8214' D) 805'25'
   E) 7914'52''
A) rad B) rad C) rad
12 4 6
 2 18. ¿Cuántos segundos centesimales están
D) rad E) rad
5 5 contenidos en un ángulo que equivale a la
milésima parte del ángulo de una vuelta?
12. El número de grados sexagesimales que A) 2900 B) 4300 C) 3000
D) 4000 E) 3700
tiene un ángulo es ab y su número de
grados centesimales es bc . Calcule la o l ll
19. Si:   a5 b6 c7 , es el complemento del
medida del ángulo en radianes.
ángulo de medida 14,3925°.
  
A) rad B) rad C) rad ab
8 6 4 Calcular: H 
c
  A) 5/7 B) 1 C) 2
D) rad E) rad
3 12 D) 3 E) 5

13. Los ángulos α y β miden 30° y 50g, 20. La medida de un ángulo α es 1080’ en el
respectivamente. Hallar la medida de    sistema sexagesimal y (3 x 5)g en el
en un nuevo sistema cuya unidad de sistema centesimal. Si la medida del ángulo
medida (1u) corresponde a las tres cuartas
partes del ángulo de una vuelta. β excede en (11x)g a la medida de α, halle
u u u la medida del ángulo β en radianes.
 11   18   7 
A)   B)   C)   3  3
 18   7   18  A) rad B) rad C) rad
5 5 8
u u
 18   5   2
D)   E)   D) rad E) rad
 5   18  9 9

14. Sean α y β ángulos en los sistemas 21. Los números que representan las medidas
sexagesimal y centesimal, respectivamente. de dos ángulos positivos, α y β, en minutos
Si el número de grados sexagesimales de α sexagesimales y minutos centesimales,
con el número de grados centesimales de β respectivamente, son iguales. Si la suma de
 77
está en relación de 3 a 4, halle . dichos ángulos es rad , halle la medida
 270
A) 3/4 B) 1/3 C) 5/6 de α en radianes.
D) 2/5 E) 3/5  3 3
A) rad B) rad C) rad
4 27 4
Tema: Relación de Grados, Minutos y Segundos 5 5
D) rad E) rad
3 7g 27 6
15. Calcular el valor de: E   m 4
10 ' 20
A) 7 B) 7 C) 8 22. El profesor de trigonometría comienza su
clase a las 07:00 a.m., los alumnos que no
D) 9 E) 11
se quieren perder su clase llegan 10
minutos antes. ¿Cuál es el ángulo que
x  x '
16. Simplificar: E  forman las manecillas del reloj analógico
x g  22xm que todos los alumnos ven al entrar al
A) 22/27 B) 25/23 C) 25/27 salón?
D) 3/2 E) 2/3 A) 80° B) 85° C) 90°
D) 95° E) 100°

Trigonometría 9
Prof: Breyner Oroya Cruz
Tema: Relación de Sistemas Angulares 28. Siendo “S” el número de grados
23. Sabiendo que la diferencia de los sexagesimales de un determinado ángulo
cuadrados de los números de grados 18 4
que cumple:  S  3 , calcular la
centesimales y sexagesimales de un 4
S
ángulo, es al producto de dichos números; mediad de dicho ángulo en radianes.
como 38 veces su número de radianes es a
9 8 7
135 , señale la medida radial del ángulo. A) rad B) rad C) rad
20 15 15
 
A) rad B) rad C)  rad 6 5
4 2 D) rad E) rad
25 18
3 3
D) rad E) rad
2 4 29. Los números que representan la medida en
sexagesimal y centesimal de un ángulo se
24. Hallar el valor de S en radianes. encuentran en progresión aritmética; si
Si: S  3x x y C  2(x x  4) además su número de grados
sexagesimales y radianes están en
  
A) rad B) rad C) rad progresión geométrica de la misma razón,
5 10 20 calcular el ángulo en sexagesimales.
  o o o
D) rad E) rad         
18 6 A)   B)   C)  
 20   30   18 
o o
81(S  C)(C  S)      
25. Simplificar: M  D)   E)  
38(SC)  40   45 
9 9 11
A) B) C) 30. Sean S y C los números de grados
10 20 20
sexagesimales y centesimales,
20 10 respectivamente, de un ángulo. Si
D) E)
9 9 10 1 2 3 4
     ...( C  1) , halle la
9S C C2 C3 C4
26. Sean S°, Cg y R rad las medidas de un
medida de dicho ángulo en radianes.
ángulo en grados sexagesimales y radianes
respectivamente tal que 9  9
A) rad B) rad C) rad
3 3 3 10 20 20
 27   30   3  3
 S    C    20R   64 4 
      D) rad E) rad
Calcule la medida del dicho ángulo en 25 10
radianes.

A)
3
rad B)
2
rad C)
4
rad
Problemas Propuestos
10 5 5
3 6 01. ¿A cuánto equivale la suma de los dos
D) rad E) rad tercios del ángulo de una vuelta en sentido
5 5 antihorario con un medio del ángulo recto
en sentido horario?
27. Señale la mediad circular de un ángulo que A) 180° B) 195° C) 210°
1729 D) 235° E) 250°
verifica: S2 (RC)1  C2 (RS)1 
3R
Siendo S, C y R lo conocido para un mismo 02. De la gráfica, cual es la relación correcta.
ángulo.
 3
A) rad B)  rad C) rad
2 4  
3 2
D) rad E) rad
2 3 A)     90º B)     180º
C)     90º D)     180º
E)     90º

Trigonometría 10
Prof: Breyner Oroya Cruz
03. A que es igual:      a partir del gráfico k
08. Hallar “k” si: rad  18  60g
adjunto. 5
A) 1 B) 2 C) 3
 D) 4 E) 5

09. Calcular el valor de:
x  y  (x  y)
 x rad  y rad  (x  y)rad

A) – 450º B) – 360º C) – 720º   


A) B) C)
D) 360º E) 720º 60 100 180
 
D) E)
04. De la figura mostrada determine “ x  y ”. 200 150
120º
10. La diferencia de las medidas de dos
x

ángulos suplementarios es rad .
y 3
Determine el mayor de ellos.
A) 60° B) 40° C) 50° A) 90° B) 100° C) 120°
D) 70° E) 80° D) 160° E) 130°

05. En el gráfico: 11. En la figura:

g
y

35 x

Si: y  4x  25 , entonces el valor de “x”, El valor de 10  9 , es:


es: A) 990 B) 900 C) 1000
A) 30° B) 36° C) 40° D) 90 E) 180 CEPRU 2016–II
D) 35° E) 34° CEPRU 2016–II
12. Si se cumple que: (x y)(2x y)g ,
06. A partir de la gráfica:
 xy 
calcule el valor de 11 2 .
 xy 
120 A) 1 B) 2 C) 3
2x  y D) 4 E) 5

13. Si α representa la medida de la treintava


xy parte de 1° y β representa la veinteava
3  2
parte de 1g, calcule
10  9
A) 1/10 B) 1/3 C) 1/15
Determine el valor que adopta “y”. D) 1/7 E) 3
A) 142° B) 146° C) 148°
D) 150° E) 152° 14. En un nuevo sistema de medición angular,
la unidad de medida es el grado especial
 (1E) que se obtiene de dividir el ángulo de
25  50g  rad
3 una vuelta en 140 partes iguales. Calcule el
07. Calcule: E 
 7E  2  2g
64  40g  rad valor de
6 7E  6g
A) 1 B) 2 C) 3 A) 13/9 B) 11/10 C) 9/10
D) 4 E) 5 D) 7/5 E) 9/13

Trigonometría 11
Prof: Breyner Oroya Cruz
78g 20 22. La diferencia de los números de grados
15. Calcule el valor de:  centesimales y sexagesimales de un mismo
300m 120'
ángulo es al producto de los mismos como
A) 6 B) 7 C) 16
el número de radianes que contiene el
D) 36 E) 4
ángulo dividido por 6 . Calcule
16. La suma de las medidas de dos ángulos es aproximadamente, el valor de dicho ángulo
4080’ y su diferencia es 40g. halle la medida en radianes.
del mayor ángulo en radianes. 2 2 3
A) B) C)
13 17 45 10 20 15
A) B) C)
45 45 13 3 2
D) E)
45 11 30 15
D) E)
17 45
23. Determine la medida en el sistema
17. ¿Cuántos minutos centesimales se deben sexagesimal del ángulo, si se cumple que:
adicionar a 32400 segundos sexagesimales 3S  2C  42
3 A) 50° B) 52° C) 53°
para obtener radianes? D) 54° E) 55°
50
A) 100 B) 200 C) 50
D) 150 E) 10 24. Sabiendo que “S” y “C” son lo conocido tal
que: C  nn  2 ; S  nn  2
18. Señale el equivalente de 712' en el Señale la medida circular del ángulo.
sistema centesimal.   
A) B) C)
A) 7 g10m B) 8 g C) 8g20m 3 4 5
D) 7g50m E) 9 g  
D) E)
10 20

19. Si se cumple: rad  x y ' z''
64 25. Siendo S, C y R lo conocido para un mismo
Calcule el complemento de (x y z) en 20R  C  S
ángulo, reducir: M 
radianes. 20R  C  S
4 3 9 A) 5 B) 10 C) 15
A) B) C) D) 20 E) 25
9 10 20
17  26. Si S, C y R lo convencional para un mismo
D) E)
36 40 ángulo. Halle R, si S y C son raíces de la
ecuación 3x 2  19x  30  0
20. Un ángulo mide a ' y b m en los sistemas   
sexagesimal y centesimal, respectivamente. A) B) C)
3 6 20
ab  2a2  b2
Si  208 , calcule la medida  
ba D) E)
30 60
en radianes.
   27. Sean S°, Cg y R rad las medidas de un
A) B) C)
100 180 360 ángulo “α” en los sistemas sexagesimal,
  centesimal y radial respectivamente, halle
D) E) el menor valor posible de R, si S es un
200 540
numero par y C es un numero múltiplo de 3.
21. ¿Qué ángulo forman las manecillas de un   
A) rad B) rad C) rad
reloj de pared a las dos horas con diez 4 3 6
minutos?  3
A) 2° B) 3° C) 4° D) rad E) rad
20 10
D) 5° E) 6°
28. Un ángulo es tal que los números que
indican su medida en grados

Trigonometría 12
Prof: Breyner Oroya Cruz
sexagesimales (S), grados centesimales
(C) y radiales (R) respectivamente cumples Problemas Dirigidos
con la condición.
01. B 02. B 03. D 04. A 05. E 06. E
S R CS RC
  1 07. E 08. D 09. B 10. A 11. D 12. C
180   190   200
13. E 14. C 15. B 16. C 17. A 18. D
Hallar la medida de dicho ángulo en
radianes. 19. E 20. C 21. D 22. D 23. E 24. B
  25. B 26. D 27. D 28. A 29. A 30. B
A)  rad B) rad C) rad
20 2 Problemas Propuestos
 01. B 02. C 03. E 04. A 05. E 06. E
D) rad E) 2 rad
40 07. A 08. B 09. C 10. C 11. B 12. E
13. C 14. A 15. E 16. A 17. B 18. B
19. D 20. D 21. D 22. D 23. D 24. C
29. Se sabe que: C  C  C  ...  4
25. B 26. E 27. E 28. A 29. C 30. A
Calcular “S”, donde S y C son las medidas
sexagesimales y centesimales de un mismo
ángulo.
A) 9 B) 4,5 C) 18
D) 27 E) 20

30. Los números que indican la medida de un


ángulo en minutos sexagesimales y
centesimales son M y N respectivamente, y
los números que indican la medida de otro
ángulo en grados centesimales y
centesimales son S y C respectivamente. Si
M  2S 18 y N  2C  20 , entonces la
diferencia positiva de las medidas de dichos
ángulos en grados centesimales, es:
A) 39 B) 60 C) 40
D) 23 E) 41 CEPRU 2016–I

Trigonometría 13
RAZÓN TRIGONOMÉTRICA TEOREMA DE PITÁGORAS
Se denomina razón trigonométrica a la relación “En todo triangulo rectángulo la suma de los
que se establece entre los lados de un triángulo cuadrados de sus catetos es igual al cuadrado
rectángulo respecto a uno de sus ángulos de la hipotenusa”.
agudos. Las razones trigonométricas en total
son seis y estas son: Seno, Coseno, Tangente,
c
Cotangente, Secante, Cosecante. a

c ab c  ab b
a c a ; c b

c  a2  b2
2

 a,b,c 
b Triángulos Rectángulos Pitagóricos
Se denomina de esta manera a aquellos
A los resultados así obtenidos se les asigna un triángulos rectángulos cuya medida de sus
nombre asociado a uno de los ángulos agudos lados esta expresada por números enteros.
del triángulo. Así en el gráfico; para θ tenemos:
a : Cateto Opuesto (CO) Los lados de todo triangulo pitagórico tienen la
b : Cateto Adyacente (CA) siguiente forma:
c : Hipotenusa (H)
B
Luego se definen:
CO a H c m y n 
sen    csc    m2  n2
H c CO a 2mn
mn
CA b H c
cos    sec   
H c CA b A C
CO a CA b m2  n2
tan    cot    Algunos triángulos pitagóricos son:
CA b CO a

Importante: 13
5 17 8
Las razones trigonométricas de un ángulo;
dependerán de la medida de dicho ángulo y no
de los lados del triángulo rectángulo en que se
12 15
ubique. Por ejemplo:

C 29 61
PQ 20 11
sen  
AQ
MN
M sen   21 60
AN
Q BC
sen  
AC 41 85
40 84
Iguales

A P N B
9 13

Trigonometría 14
Prof: Breyner Oroya Cruz
RAZONES TRIGONOMETRICAS DE ÁNGULOS PROPIEDADES DE RAZONES TRIGONOMETRICAS DE
NOTABLES ÁNGULOS NOTABLES
Son aquellas razones de triángulos rectángulos I. R.T. Reciprocas: Se nota claramente, de las
en los cuales conociendo las medidas de sus definiciones de las razones trigonométricas
ángulos agudos se pude establecer la de un ángulo agudo, que existen tres
proporción en la que se encuentran los lados parejas que son una la recíproca inversa de
de dicho triángulo. la otra, por lo que su producto es siempre
igual a 1 (uno). Estas parejas son las
siguientes:
45º 60º
2 2
1 1 c 
a
45º 30º
1 3 
b
sen .csc   1 cos .sec   1 tan .cot   1
75º 53º
4 5
6 2 3
II. R.T. de ángulos complementarios: Cuando se
15º calculan las razones trigonométricas de los
37º
2 ángulos agudos de un triángulo
6 2 4
rectángulo, se puede notar que existen
ciertas parejas de éstas que toman el
mismo valor. Esta característica la vamos a
74º 82º indicar de la siguiente manera:
25 5 2
7 1 Si :    son agudos; Si :    son agudos;
tales que :     90º tales que :
16º 8º
Entonces : sen   cos
24 7
sen   cos tan   cot 
tan   cot  sec   csc 
sec   csc  Entonces :     90º
5 10
1 1

53º /2 37º /2 R.T.()  Co  R.T() R.T.()


1
2 3 donde :     90 Co  R.T.()

30 60 45 37 53 16 74 Observación:


1. Para ángulos agudos “x” e “y” se cumple:
sen
sen x  cos y 

tan x  cot y   x  y  90º
cos
sec x  csc y 

tan
2. Para ángulos agudos “x” e “y” se cumple:
cot sen x.csc y  1

cos x.sec y  1  x  y
sec tan x.cot y  1 

csc

Trigonometría 15
Prof: Breyner Oroya Cruz
Área de una Region Triangular
El área de cualquier región triangular está dada  (1)2  (2)2  x 2
x
por el semiproducto de dos de sus lados 1 5  x2
multiplicado por el seno del ángulo que forma
dichos lados.  x  5
Así tenemos: 2

Para determinar la medida del ángulo θ,


a calculemos una razón trigonométrica con los
S catetos de longitudes 1 y 2.
 Por decir:
1
b tan      26º 30' (Aproximadamente)
2
1
S a.b.sen Como     90º    63º 30'
2

2. La longitud de un lado y la medida de un


Ángulo Mitad
Se genera de la prolongación de uno de los ángulo agudo
catetos del triángulo rectángulo.
A) Conociendo la longitud de la hipotenusa y
un ángulo agudo.

c a
a
/2  a.sen
c b

 
tan  csc   cot  cot  csc   cot  a.cos 
2 2

RESOLUCIÓN DE TRIÁNGULOS RECTÁNGULOS B) Conociendo un ángulo agudo y la longitud


del cateto opuesto a dicho ángulo.
Las aplicaciones de la trigonometría en campo
como topografía y navegación requieren
resolver triángulos rectángulos. La expresión
“Resolver un triángulo” significa encontrar la
longitud de cada lado y la medida de cada a.csc 
a
ángulo del triángulo.

En esta sección veremos que podemos
a.cot 
resolver cualquier triangulo si se nos da:

1. Las longitudes de dos lados C) Conociendo un ángulo agudo y la longitud


del cateto adyacente a dicho ángulo.
Ejemplo:
Resolver un triángulo rectángulo sabiendo que
sus catetos miden 1 y 2 respectivamente.

Resolución: a.sec 
a.tan
Para calcular x, aplicamos el teorema de
Pitágoras: 
a

Trigonometría 16
Prof: Breyner Oroya Cruz
x 2  12x  20  0
Problemas Resueltos x  2  x  2 No cumple
x  10  x  10 Si cumple
Problema 01: Señale el valor de “x”,
Si: sen2x.csc 40º  1
Los lados del triángulo rectángulo son positivos,
siendo el único valor admisible x = 10.
Solución 01: Dada la ecuación:
sen2x.csc 40º  1 Problema 05: Si el cuadrado de la suma del
Luego, los ángulos deben de ser iguales: cateto “a” y la hipotenusa “b” de un
2x  40º  x  20º triángulo rectángulo (recto en “B”)
es igual a 9 veces su producto.
Problema 02: Hallar el menor valor positivo de Hallar: “ senA  csc A ”
“x”. Si: sen5x  cosx
Solución 05: Del enunciado obtenemos:
Solución 02: Dada la ecuación: sen5x  cosx
Luego, los ángulos deben sumar 90º, entonces: A
5x  x  90º  6x  90º (a  b)2  9ab
x  15º c b
a  2ab  b2  9ab
2

 a 2  b2  7ab
Problema 03: Reducir:
sen10º sen20º sen80º C
E   ...  B a
cos80º cos70º cos10º
Trabajando con la pregunta, tenemos:
Solución 03: Recordemos que: a b
sen x sen A  csc A  
 1  x  y  90º b a
cos y
a2  b2
Trabajando en la pregunta, tenemos: sen A  csc A 
ab
sen10º sen20º sen80º
E   ...  7ab
cos80º cos70º cos10º sen A  csc A 
ab
8 terminos
sen A  csc A  7
E  1  1  ...  1
8 terminos

E8 Problema 06: Calcular:


E  sen2 30º  tan37º
Problema 04: De la figura, calcular “x”
Solución 06: Reemplazando valores:
2
 1 3 1 3
E       1
2 4 4 4
x3
x 5
Problema 07: Reducir:
sen30º  tan37º
E
x2 sec 2 45º  tan2 60º

Solución 04: Por el teorema de Pitágoras Solución 07: Del enunciado:


tenemos:
(x  5)2  (x  2)2  (x  3)2
x 2  10x  25  x 2  4x  4  x 2  6x  9

Trigonometría 17
Prof: Breyner Oroya Cruz
sen30º  tan37º Solución 10: Trabajando en el gráfico, tenemos:
E
sec 2 45º  tan2 60º
A B
1 3 5

2 4 5 1 1
E  4  
2
2  3
2
5 4 5 4
1 M

2a a 2
Problema 08: Calcular:
E  tan18º 30' cot18º 30' a 2 45º
D C
N a 2
a a
Solución 08: Reemplazando valores:
E  tan18º 30' cot18º 30' 3a
Nos piden: cot    cot   3
 37º   37º  a
E  tan    cot  2 
 2   
E  csc 37º  cot 37º  csc 37º  cot 37º Problema 11: Hallar “x” en función de “d” y “θ”
 5  10
E  2csc 37º  2   
3 3
x
Problema 09: Calcular el área de la región
sombreada: d

15º

Solución 11: Trabajando en el gráfico, tenemos:


4u 6u
x cos   dsen 
Solución 09: Del grafico tenemos: 
x
d
15º

4u S 
30º 
15º
4u 6u dsen 
Despejando “x”: x   x  dtan 
1 1 cos 
S  (4)(6)sen30º  12   6u2
2 2

Problema 10: En la figura ABCD es un cuadrado,


M y N son puntos medios.
Determine "cot θ”
A B

M

D N C

Trigonometría 18
Prof: Breyner Oroya Cruz
07. En un triángulo rectángulo de lados
Problemas Dirigidos respectivo a, b y c, recto en B donde se
cumple: a  b  3c . Calcule senA .
A) 3/5 B) 4/5 C) 7/9
Tema: R.T. de Ángulos Agudos D) 4/7 E) 8/11
01. En la figura adjunta:
08. En un triángulo ABC (C = 90°), se cumple
2n  3 B
2  cot  
5 2
que: tan(A) 
 A
4  cot  
2n  2 2
Calcular: tan   cot 
sec 2 (B)  cot(A)
A) 169/60 B) 169/90 C) 169/50 Calcular: R 
D) 90/169 E) 60/169 4  csc 2 (A)
A) 1 B) 1/2 C) 1/3
D) – 1/3 E) 2
24 
02. Si: tan x  ; x  0; ;
7 2 Tema: R.T. de Triángulos Notables
8 09. Calcule el valor de A  B , si:
Halle: sen x  cos x 
25 A  4sen30  tan2 45
A) 1 B) 2 C) 3
D) 5 E) 4 B  sec 60 2csc 30
A) 3 B) 4 C) 5
20 D) 6 E) 7
03. En un triángulo rectángulo ABC, tan A  ,
21 10. Calcular el valor de “x” en:
y la hipotenusa mide 58 cm. Hallar el
x cos60  tan45
perímetro del triángulo.  csc 53
A) 156 cm B) 116 cm C) 136 cm x cos60  tan45
D) 140 cm E) 145 cm A) 10 B) 12 C) 16
D) 18 E) 20
04. Si: (tan )cot   2 , “α” es ángulo agudo.
  
Calcular: E  2sec .csc  tan    sen  
A) 9 B) 8 C) 7 11. Si: cos    6 3
D) 6 E) 5 2
1  sec  
4
05. En el triángulo ABC, recto en B, sus lados
Entonces el valor de la expresión:
son: a, b y c respectivamente, expresar en
sen  tan 
csc 2 A  csc 2 C E , (θ es un ángulo agudo)
términos de sus lados: P  sen  tan 
cot A  cot C
A) – 11 B) 12 C) – 9
A)
ab
B)
b2
C)
b2 D) 8 E) – 10 CEPRU 2017–I
b2 a c
b2 b 12. En la figura, hallar “ PQ ”
D) E)
ac ac A
4
06. En un triángulo rectángulo ACB, recto en C. Q
Calcular el lado “b” en términos de “a”.
3
Si: sec(B).sec(A)  tan(A) 
4
4a 3a 5a 16 45
A) B) C) C B
38 P
3 4 4
4a 4a A) 2 2 B) 5 2 C) 6 2
D) E)
5 3 D) 8 2 E) 10 2

Trigonometría 19
Prof: Breyner Oroya Cruz
13. Del gráfico, hallar: AP , siendo PC  10 A) 2 B) 2 1 C) 2 2  1

B D) 2 1 E) 2 2  2

P Tema: R.T. Reciprocas y Complementarias


17. Hallar el valor de “m”, si: m  3n ,
cot(m  n  30).tan(2m  3n  40)  1
23 A) 45° B) 20° C) 15°
37 D) 30° E) 35°
A C
A) 12 B) 14 C) 15 18. Si se cumple que:
D) 16 E) 18 cos(x  y  10).sec(3x  2y  20)  1
Además: x  y  50
14. Del gráfico, hallar: tan
Calcular el valor de: x – y
B A) 4° B) 3° C) 6°
D) 7° E) 5°
60
4 19. Si: sen(x  y).csc(4x  y  10º )  1  0 y
sec(3x  y).cos(10)  1
P Hallar: tan(3x).sen(6x)  sen(20  x)
2 A) 2 B) 1 C) 3
D) 5 E) 4
A  C
20. Si: sen(x  y  10º )  cos(3y  4x  20º )
3 3 3
A) B) C) Además: tan(2x  y)  cot(20)
2 3 4
Hallar el valor de “x”.
3 3 A) 24° B) 25° C) 30°
D) E)
5 6 D) 20° E) 18°

15. De la figura, calcular: P  5sen .csc  21. Si: tan(5x  8)  cot(2x 2)
Calcular: M  tan(x  3)  tan(5x)
C
A) 1 B) 2 C) 3
  D) 4 E) 5

22. Si se cumple que:


tan40.tan50.sen10
53 45 sec(4).cos(  45) 
A B cos80
M
Calcule: cot()  tan(4)
A) 2 B) 2 2 C) 3 2 A) 1 B) 3 C) 3  3
D) 4 2 E) 5 2 D) 2 E) 3  3

16. Del gráfico, hallar: BC 23. Siendo α y β ángulos complementarios, tal


A n 1
que sec()  n 1 y sen() 
3
Determine el valor de “n”.
A) 0,5 B) 1,0 C) 1,5
D) 2,0 E) 2,5
O
1
24. Sabiendo que:
45 tan(3).tan(2)  1
C
B cos(2).sec(3  5)  1

Trigonometría 20
Prof: Breyner Oroya Cruz
Calcular: N  sen (    5)  tan2 (3)
2 B
D
A) 0,25 B) 0,50 C) 0,75
D) 1,00 E) 1,25

Tema: R.T. de Triángulos Rectángulos 6 
25. Calcule el perímetro de la región A C
sombreada. A) sen.csc  B) tan .cot 
C) sec .csc  D) csc .sen 
37
E) cos .cos 
3
29. Calcule la razón entre las áreas de las
 regiones triangulares ABC y ADC,
A) 3  9sen  2cos  respectivamente.
B) 3  9sen  3cos  B
C) 3  3sen  9cos 
D) 9  3sen  9cos 
E) 3  9sen  6cos 

A   C
b
26. De la figura, calcular:
a
C
D

a A) 2sen B) 2sen2 C) 2cos


b
D) 2cos2  E) 2tan
45 37
A B 30. Del gráfico mostrada se conoce que:
D
AE  ED  DC . Calcule tan .csc 
2 2 2 3 2
A) B) C) C
5 5 5
4 2 5 2 D
D) E)
5 3

27. Del gráfico, calcule “x”.  


A E B
2
A) 1/2 B) 1 C) 2
D) 1/4 E) 4

Problemas Propuestos
 1
01. Si: sen   , “θ” es agudo, calcular, “ cot  ”
x 3
A) 2sen2 B) 2cos2  C) 2tan2  A) 2 B) 2 2 C) 9 2

D) 2cot 2  E) 2csc 2  7 2
D) E)
2 4
28. Dado la figura, determine BD en términos
02. En un triángulo isósceles ABC (AB =BC) se
de α y β, si AB  1 sabe que la altura relativa al lado desigual
es el triple de dicho lado. Calcular “ sen ”
ˆ
(  BAC)

Trigonometría 21
Prof: Breyner Oroya Cruz
6 1 1 12
A) B) C) 08. Si: tan x  ; x  0;90
37 37 7 5
 x
D)
6
E)
7 Calcule tan  45  
7 4  2
A) 1/5 B) 2/5 C) 3/5
03. Con los datos de la figura, calcule: D) 5 E) 3
1
E  5cos   25sen  09. Calcule: 2 tan2 60  3 sec 2 45  3csc 37
5
A) 11 B) 13 C) 15
D) 17 E) 19
2n  1
n5 10. Si “α” es un ángulo agudo, tal que:
 cos   sen30.sec 45 , halle “α”.
2n A) 30° B) 37° C) 45°
A) 11 B) 10 C) 9 D) 53° E) 60°
D) 12 E) 8
11. Siendo: tan(4x).cot(x 48)  1
04. En un triángulo rectángulo ABC (B = 90º), Calcule: E  csc(x)  cot(x)
se sabe que: sec A  2,6 . Si el perímetro A) 1 B) 3 C) 5
del triángulo es 60 cm, ¿Cuál es su área en D) 7 E) 9
cm2?
A) 120 B) 60 C) 30 12. Si “α” es un ángulo agudo, tal que:
D) 90 E) 45 tan   (sec 37  tan37)sen2 45
05. En un triángulo ABC recto en B se cumple cos2 
Calcule: E  9sen2 
4 2
que: sen A.cosC  . Calcule: 5 cos A A) 1 B) 18/17 C) 2
9
D) 3 E) 16/17
5 1 5
A) B) C)
3 3 3 13. Del gráfico, calcule cot  .
5 2
D) E)
2 3
37 
06. Si x e y son ángulos agudos, además:
 1  1
 senx    cos y  
27 3
 4 2 3x  1
Calcule: 2 tanx  3 cot y
A) 1,5 B) 2,5 C) 0,5 2x x 1
D) 2,0 E) 4,0
A) 1 B) 1/3 C) 3
D) 1/2 E) 2
07. Del gráfico, calcule: 5 tan
C 14. Según el gráfico, calcule: tan
 B
53o
C


A 2 E 3 B 
A D
E
3 5
A) 3 B) C) A) 3/17 B) 13/41 C) 12/37
2 2
D) 14/39 E) 12/41
D) 5 E) 2

Trigonometría 22
Prof: Breyner Oroya Cruz
15. A partir del gráfico mostrado, se cumple tan10  tan20  tan30  ...  tan80
E
AD  DC . Calcule tan  cot10  cot 20  cot 30  ...  cot 80
B 3
A) 3 B) 2 C)
2
D) 1 E) 1/2

 45 37 22. Del grafico mostrado:


A D C
A) 10/3 B) 3/10 C) 3/5
3sec 50  csc 40
D) 5/3 E) 3/4 sec 50  csc 40

2
16. Del gráfico, calcule: 17(sen   cos )
Calcule: sec 2 3  tan2 4
135 A) 1 B) 2 C) 3
3 2
D) 4 E) 5

23. Si: sen(3 x 10)  cos(5 x) ; 0  x  15
A) 4 B) 3 C) 5 Calcule: tan(x).tan(2x).tan(8x).tan(7x)
D) 2 E) 1/2
A) 0 B) – 1 C) 1/2
D) 1 E) 2
17. Se cumple que:
sen(2x y).csc(x 2y)  1
24. Si: sen(x 3y).sec(2x 3y)  1
tan(x y).cot(30)  1
2 tan(6 x  3 y)  1
Calcule: 2tan(x)  cos(x y) Calcule:
cot(3 y  3 x)  cos(2 x  4 y)
A) 5/2 B) 3/2 C) 1/2
A) 1 B) 2 C) – 1
D) 2 E) 1
D) – 2 E) 1/2
18. Siendo “α” y “θ” ángulos agudos y se
25. Halle “x” en términos de “n” y “α”.
cumple:
sen(3 ).csc(60)  3 2  23
cos(10  ).sec(20)  1
Determine: 2sen(  )  1 
A) 0 B) 1 C) 2 n x
D) 3 E) 4
A) n.sen  2
B) n.cos2  C) n.tan2 
19. De las siguientes condiciones: D) n.cot 2  E) n.sec .csc 
sen(  )  cos(   )
tan(  30).cot()  1
26. Determinar CD
Donde los ángulos dados son agudos,
calcule:   
B
A) 15° B) 30° C) 10°
D) 45° E) 20° m
A
sen20  cos70 
20. Si: cos   ;   0;90
3sen2 20.csc 20 
Calcule: tan2  C D
A) 5/9 B) 1/4 C) 4/5
D) 4/9 E) 5/4 A) m.tan .sen  B) m.cot .cos 
C) m.tan .cos  D) m.tan .csc 
21. Calcule el valor de la siguiente expresión: E) m.cot .sen 

Trigonometría 23
Prof: Breyner Oroya Cruz
27. Del gráfico, hallar “x”
Problemas Dirigidos
45 01. A 02. A 03. D 04. E 05. D 06. B
07. B 08. B 09. C 10. D 11. A 12. E
13. A 14. D 15. D 16. B 17. D 18. C

19. A 20. D 21. D 22. D 23. D 24. E
m x
25. B 26. C 27. D 28. D 29. D 30. E
A)
m
B)
m
C)
m Problemas Propuestos
tan   1 cot   1 1  cot  01. B 02. A 03. D 04. A 05. C 06. A
m 07. A 08. D 09. D 10. C 11. D 12. A
D) E) m(1 tan )
1  tan  13. E 14. E 15. B 16. C 17. A 18. C
19. B 20. E 21. D 22. E 23. D 24. B
28. Calcule “ cos  ”.
25. C 26. E 27. B 28. D 29. A 30. C
a


b

a b a
A) B) C)
b a b
b a
D) E) 3
a b

29. En la figura siguiente, hallar “x” en términos


de “H”, “Φ” y “θ”.

x


H
H H
A) B)
cot   tan  tan   cot 
C) H(cot   tan ) D) H(tan   cot )
H
E)
tan   cot 

sen .cos 
30. Del grafico mostrado, halle
tan 

 
1 2
A) 1/4 B) 1/3 C) 1/2
D) 1 E) 2

Trigonometría 24
R.T. de Ángulos en Posición Normal
ÁNGULOS EN POSICIÓN NORMAL Sea α un ángulo que está en posición normal si
P(x,y) es un punto que pertenece a su lado final
Llamado también en posición canónica o entonces las razones trigonométricas de α se
stándar. Es aquél ángulo trigonométrico cuyo definen de las siguiente manera:
vértice (V) coincide con el origen del sistema
cartesiano y su lado inicial (L.I.) coincide con el y
eje "x" positivo. P(x o ,yo ) yo
Cuando un ángulo, está en posición normal, el
lado final (L.F.) puede estar en uno de los r

cuadrantes, en cuyo caso se dice que éste
pertenece a tal cuadrante. '
xo x
y
L.F. Del gráfico :
r  (xo )  (yo ) : Radio vector
2 2

 : es un ángulo en
  ' : Se denomina ángulo de referencia
posición normal
L.I. Se define:
V   IIC ;   0
x yo xo xo
sen   cos   tan  
r r yo
y
Del gráfico : r r yo
csc   sec   cot  
L.I.  : es un ángulo en yo xo xo
V
x posición normal
L.F.    IIIC ;   0 Signo de las R.T. en los Cuadrantes
Dependiendo del cuadrante al que pertenezca
un ángulo en posición normal, sus R.T. pueden
Ejemplos: ser positivas o negativas. Es así como se
obtiene el cuadro adjunto.
y y

x x
IIC IC
200 420
 seno  todas son
()  () 
cosecante  positivas

 tangente  coseno
y y ()  () 
 cotangente secante
IIIC IVC
120 310
x x Ejemplos:
 tan(100)  ( )
 cos(280)  ( )
 sec(81)  ( )
 sen( 200)  ( )

Trigonometría 25
Prof: Breyner Oroya Cruz
ÁNGULOS CUADRANTALES ÁNGULOS COTERMINALES
Son aquellos ángulos en posición normal cuyo Son aquellos ángulos trigonométricos que
lado final pertenece a alguno de los semiejes tienen el mismo vértice, el mismo lado inicial y
del sistema de coordenadas rectangulares. el mismo lado final.
y
Lado Final
90º Lado Inicial
180º
x 
270º

 
Cuadrantales  90º.k ó  rad .k ; k  Z Propiedad Nº 1:
2 

    360º.k     400g.k
R.T. de Ángulos Cuadrantales
kZ     2 rad.k
0 90 180 270 360  y  : son coterminales

sen
cos Propiedad Nº 2:
tan y
cot P(xo ;yo )
sec 
csc
 x
Caso Particular:
 R.T( )  R.T()
90º  rad
2 R.T( )  R.T(  360º k),k  Z
IIC IC  y  : son coterminales
0 ó 360º
180º   rad R.T. de Ángulos Negativos
0 ó 2 rad
En este punto vamos a comparar las razones
IIIC IVC trigonométricas de un ángulo α (  0) con las
razones trigonométricas de – α.
3
270º 
2 y
P(x o ;yo )
Caso General: r yo

(4k  1)  x
o
2
 x
 yo
r
(2k  1) 2k P(xo; yo )

sen (  )  sen( ) cot (  )   cot (  )


cos (  )  cos( ) sec (  )  sec ( )

(4k  3) tan (  )   tan(  ) csc (  )   csc ( )
2

Trigonometría 26
Prof: Breyner Oroya Cruz
Problemas Resueltos IIC IC
csc () csc ()
Problema 01: Del siguiente grafico calcular:
tan ()
E  10 sen   12cot 

tan ()
y
IIIC IVC
x
Por lo tanto “θ” pertence al IIC.

 Problema 04: Para el ángulo α, de la figura, es


(1; 3) cierto que cot   2,4 . Calcular
valor de “x”.

Solución 01: De la gráfica, tenemos: y


y
x
1 x 
P(x 2;x 5)
r
3

r Solución 04: Trabajando en el gráfico, tenemos:
(1; 3)
y
Con la coordenada de dato calculamos “r”. x 2
r 2  12  (3)2  r  10 r x
x 5 r 
Reemplazamos las definiciones:
E  10 sen   12cot 
 3   1  12
E  10    12    3  4  1 Del dato: cot   2,4   …(1)
 10   3  5
x2
Del gráfico: cot   …(2)
Problema 02: Indicar el signo resultante de la x5
siguiente operación. De (1) = (2), tenemos:
E  sen130º.cos230º.tan330º 12 x  2 12x  60  5x  10

5 x  5 x  10
Solución 02: Trabajando en la pregunta:
IIC IIIC IVC

E  sen130º.cos230º.tan330º Problema 05: Cuantos ángulos positivos y


cuadrantales menores que 1234º
E  (  ).( ).( )  (  )
existen.

Problema 03: Indicar el cuadrante al que Solución 05: Los ángulos cuadrantales tienen la
pertenece la medida angular “θ”. forma: 90ºk , k  Z
Si: tan   0  csc   0 Del enunciado, tenemos:
0º  90º k  1234º
Solución 03: Del enunciado: 0  k  13,71...
tan   0 csc   0
Como k solo puede tomar valores enteros,
tan  ( ) csc  (  ) entonces se deduce que k puede tomar 13
Haciendo una representación grafica, tenemos: posibles valores, con lo cual existen 13 ángulos
cuadrantales.

Trigonometría 27
Prof: Breyner Oroya Cruz
Problema 06: Si   IIIC , es positivo y menor que Nos piden:
una vuelta. ¿En qué cuadrante R  sen   sen   cos   cos 
esta 2 / 3 ? R  2 sen   2cos 
R  2sen( 270º )  2cos( 270º )
Solución 06: Si   IIIC
R  2 sen 270º 2cos 270º
 R  2( 1)  2(0)  2
180º    270º  60º   90º
3
2 Problema 09: Simplifique:
120º   180º
3 sen() cos( ) tan( )
B  
Como 2 / 3 está entre 120º y 180º, entonces sen( ) cos  tan( )
pertenece al: II Cuadrante.
Solución 09: Trabajando en la pregunta:
Problema 07: A partir de la grafica, calcule:
sen() cos( ) tan()
K  sen  tan   cos  B  
sen( ) cos  tan( )
sen() cos() tan()
y B  
sen() cos   tan()
B  1  1  1  1

60º
Problema 10: Hallar el valor de:
x
tan360º
R  (cos270º )sen 90º   (sec180º )cot 270º
cos0º
Solución 07: Del grafico se observa que 60º y θ
son ángulos coterminales, donde se
Solución 10: Trabajando en la pregunta:
cumple que: R.T.()  R.T.(60º )
tan360º
Reemplazando en la pregunta, tenemos: R  (cos270º )sen 90º   (sec180º )cot 270º
K  sen  tan   cos 
cos0º
(0)
K  sen 60º.tan 60º  cos 60º R  (0)(1)   ( 1)(0)  1
(1)
 3  3   1 
K          2
 2  1   2  Problema 11: Si: 3tan   2cos 
Además: 0º      360º
Problema 08: Si α y θ son suplementarios y Calcule: M  cos   sen 
conterminales tal que:
  400º;500º
Solución 11: Del enunciado se observa que:
Calcule: R  sen   sen   cos   cos  3 tan   2cos   tan   cos   0
   0º,180º,360º,...   90º,270º,...
Solución 08: Del enunciado tenemos:
    180º (Suplementarios) De la condición:
0º      360º
    360º k (Coterminales)
   90º ;   180º
   90º 180º k
Reemplazando lo obtenido, tenemos:
Reemplazando en el dato, tenemos:
M  cos   sen 
400º    500º
M  cos180º  sen90º  ( 1)  (1)  0
400º  90º 180º k  500º
310º  180º k  410º
1,72...  k  2,27...  k  2
   450º ;   270º
Además:
sen(  )  sen( )
R.T.(  )  R.T.(  ) 
cos(  )  cos( )

Trigonometría 28
Prof: Breyner Oroya Cruz
06. Si los puntos A(2; 5) , B y C(8;0) son
Problemas Dirigidos colineales de tal manera que:
AB 2
 ,
BC 3
Tema: Introducción a la Geometría Analítica calcule las distancias del punto P(7; 15) al
01. Sean los puntos A(4;3) y B, si el punto punto B.
medio del segmento AB es M(1; 2) . A) 9 B) 10 C) 11
D) 12 E) 13
Determinar la suma de coordenadas del
punto B.
A) 13 B) – 12 C) – 13 Tema: R.T. de Ángulos en Posición Normal
D) – 26 E) 12 07. Determinar que par de ángulos pertenece al
mismo cuadrante
02. Señale las coordenadas del punto P   200º ;   80g ;   3 rad;   510g
ubicado en el eje de abscisas que equidista A) α y θ B) α y φ C) β y φ
de A(1;5) y B(7;3) D) θ y β E) θ y φ
7  8  4 
A)  ;0  B)  ;0  C)  ;0  08. A que cuadrante pertenece “x” si:
3  3  3 
tanx senx
 11   11  A) I C B) II C C) III C
D)  ;0  E)  ;0 
2  4  D) IV C E) todos

03. Calcule las coordenadas del punto C, 09. Determinar el signo en cada caso:
si AB  BC R  sen 130º.cos210º
F  cos225º  sen 190º
y
D  tan250º  cot 310º
C A) – ; – ; – B) – ; – ; + C) + ; +; +
A(2;3) D) – ; + ; – E) + ; + ; –

53 10. Si tan   0 y sec   3


x Calcular M  csc   cot 
B
2
A) (7;4) B) (9;4) C) (9;5) A) B) 2 C)  2
2
D) (8;6) E) (10;5)
2
D)  E) 1
04. La base de un triángulo isósceles tiene por 2
extremos los puntos A(2; 1) y B(1;2) .
11. Si tan   2 y sen   0
Los lados iguales miden cada uno 17 .
Calcular: 3sen   cos 
Calcule las coordenadas del vértice
opuesto a la base. A) 2 5 B) 5 C) 2 5
A) (1; 1) y (3;3) D)  5 E) 0
B) (2; 2) y (1;1)
C) (2; 2) y (3;3) 12. Si θ es un ángulo en posición normal para
D) (1; 1) y (2;2) el cual tan   5 y cos   cos  , hallar
E) (3; 3) y (2;2)
26(cos   sen )
A) 3 B) – 3 C) 6
05. Calcule la distancia del baricentro del
D) 2 E) 1
triángulo ABC al origen de coordenadas, si:
A(3; 5) , B(1;7) y A(7;4)
13. A partir de la figura, calcular el valor de
A) 1 B) 2 C) 3 tan   sec 
D) 2 E) 5

Trigonometría 29
Prof: Breyner Oroya Cruz
y A) 16560° B) 16650° C) 16740°
8 D) 16830° E) 16920°
5 x
3  18. Calcular el valor de la expresión:
(cos60)sen270  (csc 270)sec 60
A) 1 B) 2 C) 3
(18; 8)
D) 4 E) 5
A) 7/3 B) 7/2 C) – 7/2
D) – 5/3 E) 5/3 19. A qué cuadrante pertenece el ángulo “θ” si
se cumple que:
14. Los puntos A(a b;b) y B(b;a b) sen   sen360  cot   cot 90
pertenecen al lado terminal del ángulo en A) IC B) IIC C) IIIC
posición normal α. D) IVC E) IC y IIC
Calcule csc 2   tan2  si b  0
20. Simplificar la siguiente expresión:
A) 5 B) 6 C) 7
D) 8 E) 9 (a b)2 sen3 (90)  (a  b)2 cos5 (180)
E
asen(270)  bcos2 (90)
15. Si: OP  OQ y AM  MQ , calcule tan A) 2a B) – 2a C) 4a
D) – 4a E) – 4b
y
M Q 21. Si se cumple que α y θ son positivos y
menores a una vuelta que cumplen:
A(6;2 3)
 3 
1  sen  sen  1  csc   sen  
 2 
Calcule cos(2)  tan(6)
60  O A) – 2 B) – 1 C) 0
P x D) 1 E) 2

22. Si se cumple que:


 3 2  3 2 sen(x)  sec(y)  0 ; 0  x  y  2
A) ( 3  2) B)  C) 
 2   3  x xy
    Calcule tan    2cos  
 3 2   3 2  2  2 
D)   E)   A) – 2 B) – 1 C) 0
 4 
  5  D) 1 E) 2

16. Considerando los datos de la figura, calcule 23. Si: 0  x  2


el valor de la expresión 5 3sec   sen Calcule:
x x
y E  sen(x)  1  cos2    sen  
2 3
B(8;b) A) 4 B) 5 C) 3
A(a;6)
4 5 D) 1 E) 2
3 5

 Tema: R.T. de Ángulos Coterminales


x 24. Se tiene dos ángulos que se diferencian en
 un múltiplo de 360°. Se sabe que el
cuádruple del menor es a la suma del
A) 15 B) 14 C) – 18 ángulo menor más el triple del mayor de los
D) 16 E) – 16 ángulos, como 4 es a 5. Hallar el menor de
los ángulos, si se sabe que está
Tema: R.T. de Ángulos Cuadrantales comprendido entre 1080° y 3240°.
17. Determine la suma de todos los ángulos A) 1280° B) 2160° C) 3200°
cuadrantales comprendidos entre 1000° y D) 3210° E) 3240°
2000°.

Trigonometría 30
Prof: Breyner Oroya Cruz
25. Se tienen dos ángulos coterminales tales 02. Si M es un punto medio del segmento de
que el mayor es al menor como 23 es a 2. extremos A(3; 1) y B(1;3) , calcule el
Su suma está comprendida entre 2820° y radio vector de M.
3100°. ¿Cuál es la medida del mayor?
A) 2540° B) 2760° C) 2820° A) 1 B) 2 C) 3
D) 2420° E) 3000° D) 2 E) 5

26. Los ángulos α y β son coterminales y la 03. Se tiene el cuadrado ABCD donde:
suma de sus medidas es 200°. Si la “α” A(2;6) , B(4;3) y C(1; 3) . Calcule las
toma su mayor valor negativo, halle la coordenadas del punto D.
medida de “β”. A) (4;0) B) (5;0) C) (6;0)
A) 280° B) 240° C) 260°
D) 220° E) 200° D) (7;0) E) (3;0)

27. Siendo “α” y “θ” ángulos coterminales, 04. Se traza un segmento desde A(1;1) hasta
hallar: B(3;5) . ¿Hasta qué punto será necesario
E  sen .cos .tan(  ).sec .csc 
AC BC
A) 0 B) 1 C) – 1 prolongar “C” para qué  ?
D) 2 E) – 2 6 5
Señale la suma de coordenadas de “C”.
28. En la figura: A) 35 B) 38 C) 42
D) 23 E) 27
y
(3;4) 05. Del gráfico, ABCD es un cuadrado y la
ordenada del punto D es 6. Calcule las
 coordenadas del punto C.

x y
C

El valor de: 5cos   cos(  ) , es:


A) – 1 B) – 2 C) 0 B
D) 2 E) 1 CEPRU 2017–I D

29. Los ángulos α y β son coterminales 37


3 A x
sen .cot   0 y sen  . Calcular el
13 A) (8;12) B) (8;14) C) (10;12)
valor de sec 2   tan  D) (10;14) E) (12;12)
A) 17/4 B) 19/4 C) 9/2
D) 21/4 E) 11/2 06. De acuerdo con el grafico se cumple que:
5 tan   2  0 . Determine las coordenadas
30. Si α, θ y 10° dos ángulos coterminales, del punto F si AM  MB .
calcule:
E  2sen(  20)  tan(  35)  cos(  ) y
B(11;8)
A) 0 B) 1 C) 3
D) 4 E) 5
M
Problemas Propuestos

01. Calcule la distancia entre los puntos A y B A(1;0) F x
si:
A) (10;0) B) (11;0) C) 12;0)
A(1 n;2  m) y B(4  n;6  m)
A) 1 B) 3 C) 5 D) (15;0) E) (16;0)

D) n2  m2 E) 2 n2  m2

Trigonometría 31
Prof: Breyner Oroya Cruz
07. Del grafico mostrado, calcule “ m.r ”, si A) – 5 B) – 4 C) – 5/2
5 D) – 2 E) – 3/2
sec   
4
13. De acuerdo con el grafico, calcule
y 13(2sen   3sec )

x y
r  (3;2)
(4;m)

A) – 12 B) – 15 C) – 18
D) – 20 E) – 24  x

08. Halle el valor de sec   tan  A) 13 B) 15 C) 17


D) 19 E) 21
Si 15csc   17  0 y  IIIC .
A) – 2 B) – 3 C) – 4
14. Si se cumple que
D) – 5 E) – 6
15sen2  14sen   8  0 donde α es un
9csc x ángulo en posición normal del tercer
09. Si  sen 20  cos70  1 , además cuadrante, calcule el valor de cot .cos 
81
cot x  0 , calcule el valor de A) – 2,4 B) – 2,5 C) – 2,3
D) – 2,1 E) – 2,2
2 3 cos x  3 cot x
A) – 6 B) – 1 C) 1 15. Si AB  BC , calcule 12 tan   1
D) 3 E) 6
y
C
10. De acuerdo con el grafico, calcule:
tan   cot  . B

y 37 A
(3;  a)

(a;1) 
 x

x A) – 40 B) – 42 C) – 44
D) – 46 E) – 48
4 3 2 3 3
A)  B)  C) 
3 3 3 16. De la figura mostrada, hallar el valor de:
D) – 2 E) – 4 E  sen   cos 

cos240  2sec 240 y


11. Calcule el valor de: (4;3)
cos120
A) 7 B) 9 C) – 7 
D) – 9 E) – 11

 x
12. Del gráfico, calcule sec .csc   1 , si
AC  AB
y A) 7/5 B) 3/5 C) 1
D) – 1/5 E) – 7/5
C A B(2;1)
17. Halle la suma de los ángulos cuadrantales
comprendidos entre – 1000° y 1200°.
 A) 1800° B) 1950° C) 2150°
 D) 2200° E) 2250°
x
18. Calcule el valor de:

Trigonometría 32
Prof: Breyner Oroya Cruz
sen270  cos90  tan0 26. Los ángulos coterminales α y β están en
cos 450  cot 270  sec180 relación de 2 a 7. Si el ángulo α se
A) – 2 B) – 1 C) 1/2 encuentra entre 90° y 180°, el valor de
D) 1 E) 2   2 , es:
A) 316° B) 226° C) 236°
f(270 )  f(0 ) D) 216° E) 306° CEPRU 2016–II
19. Si f(x)  sen(cos x) , calcule:
f(360 )
27. Si α y β son ángulos coterminales, calcular
A) 2 B) – 1 C) – 2 el valor de la expresión:
D) 0 E) 1
 
2 tan .cot   3 sec .cos   4 cos  
20. Si a  b  4  2 
A) 10 B) 5 C) 1
a2sen90  b2 cos180 D) 3 E) 9
Calcule:
acos360  b sen270
A) 1 B) 2 C) 3 28. Si α y θ son suplementarios y coterminales
D) 4 E) 5 tal que   400;500 , calcule:
K  sen   sen   cos   cos 
21. Si ,   0;360 son ángulos
A) – 2 B) – 1 C) 0
cuadrantales que cumplen D) 1 E) 2
(sen   1)2  (cos   1)2  0
Calcule sen(  ) 29. Si α y 50° son ángulos y coterminales tal
A) – 1 B) 0 C) 1/2 que (  10)  300;400 , calcule:
D) 1 E) – 1/2 E  2sen(  20)  3 tan(  10)
A) 0 B) 1 C) 3
22. Si α y β son ángulos cuadrantales, de modo
D) 4 E) – 2
que 0      2 , además
sen   sen  30. Los ángulos α y β son coterminales y
sen .cos2   1 , calcule:
cos   cos  (2;a) con a  3 es un punto del lado
A) 0 B) – 1 C) 1 13
D) 1/2 E) 2 terminal de β. Si cot   tan    ,
6
23. Siendo A, B y C ángulos cuadrantales calcule csc 2   cot  .
diferentes, positivos y menores o iguales a A) 47/12 B) 7/4 C) 25/8
360°, además se cumple: D) 19/4 E) 31/12
1  cos A  cos A  1  1  senB
cscB  2  tanC  1
Determine el valor de A  B  C .
A) 240° B) 810° C) 120° Problemas Dirigidos
D) 360° E) 180° 01. B 02. B 03. B 04. C 05. E 06. E
07. B 08. C 09. B 10. B 11. B 12. C
24. ¿Cuál es el mayor ángulo negativo que es 13. E 14. C 15. C 16. D 17. D 18. C
coterminal con el menor cuadrantal
positivo? 19. A 20. E 21. B 22. B 23. D 24. B
A) – 270° B) – 180 ° C) – 60° 25. B 26. A 27. A 28. B 29. B 30. C
D) – 360° E) – 450° Problemas Propuestos
01. C 02. B 03. B 04. B 05. B 06. E
25. Los ángulos α y β son terminales y para
07. B 08. C 09. A 10. A 11. B 12. E
ellos es cierto que 0    2 y 6    8
13. C 14. D 15. B 16. D 17. E 18. D
. Si   2  2535 , calcular    .
19. E 20. D 21. D 22. B 23. B 24. A
A) 1330° B) 1280° C) 1380°
D) 1220° E) 1480° 25. A 26. D 27. E 28. E 29. D 30. D

Trigonometría 33
2. Línea Trigonométrica del Coseno:
CIRCUNFERENCIA TRIGONOMÉTRICA
y
Se llama circunferencia trigonométrica a
aquella circunferencia cuyo centro coincide con
el origen del sistema cartesiano y su radio es 
igual a la unidad del sistema. En el gráfico cos 
adjunto tenemos:
y x

() C.T.

 x
 cos  : Existe   1  cos   1
R 1
()
3. Línea Trigonométrica del Tangente:

LINEAS TRIGONOMÉTRICAS 
tan

Son segmentos de medida positiva, negativa o


nula; que van a representar los valores x
numéricos de las razones trigonométricas de un
arco, ángulo o número real, siempre que esté
definido.
C.T.
1. Línea Trigonométrica del Seno:
y


  
tan  : Existe    (2n  1) ;n  
 2 
sen
  tan   
x
4. Línea Trigonométrica del Cotangente:

C.T.

sen  : Existe   1  sen   1

Trigonometría 34
Prof: Breyner Oroya Cruz
cot 
y csc  : Existe    n ;n  
C.T.   sec   1   1  sec   

Formas Generales de Arcos Referentes


x En ocasiones habrá la necesidad de
generalizar la ubicación de los ángulos
cuadrantales, para lo cual se puede tomar en
cuenta las siguientes gráficas.
y
 C.T.

n

cot  : Existe    n ;n  


x
  cot   

5. Línea Trigonométrica del Secante:


y 2n
C.T. (2n  1)

y  
(4n  1) 
x  2
sec 

 
   (2n  1) 
sec  : Existe    (2n  1) ;n    2
 2 
C.T.
  sec   1   1  sec     
(4n  3) 
 2
6. Línea Trigonométrica del Cosecante:
y
y C.T.

C.T.

x x

 n 
csc    
2

Trigonometría 35
Prof: Breyner Oroya Cruz
II. Graficando los datos en la C.T., tenemos:
Problemas Resueltos 
 1,57
2
Problema 01: Indique el producto de los valores
mínimos y máximo de la expresión:
Q  4  3cos2   2sen3 ,   

Solución 01: Recordemos que la extensión del   3,14 2  6,28


seno y coseno para cualquier sen6
sen4
ángulo está dado por:
sen5 6
1  cos   1 1  sen  1
4
Trabajando en cada una, tenemos:
1  cos   1 e2 1  sen   1 e3 3 5
 4,71
0  cos2   1 3 1  sen3  1 ( 2) 2
0  3 cos2   3...(i) 2  2sen3  2...(ii) sen 6 > sen 4 > sen 5 (Verdadero)
Sumando (i) y (ii), tenemos: III. Graficando los datos en la C.T., tenemos:
2  3cos2   2sen3  5 
 1,57
2  4  3cos2   2sen3  9 2
Qmín Qm áx 1
Q
cos1
Nos piden:
Qmáx  Qmín  18

Problema 02: Determine la veracidad (V) o   3,14 2  6,28


falsedad (F) de c/u de las
siguientes proposiciones: 6
cos6
I. sen 2 > sen 1 > sen 3
II. sen 6 > sen 4 > sen 5
cos5
III. cos 6 < cos 1 < cos 5
IV. cos 2 < cos 4 < cos 3 3 5
 4,71
2
Solución 02: Trabajando con cada proposición, cos 6 < cos 1 < cos 5 (Falso)
tenemos.
IV. Graficando los datos en la C.T., tenemos:
I. Graficando los datos en la C.T., tenemos:

 1,57
 2 2
 1,57
2 2 cos 2
1

3 cos3
sen2 sen1
3   3,14 2  6,28
sen3
  3,14 2  6,28
cos 4
4

3
 4,71
2
3
 4,71 cos 2 < cos 4 < cos 3 (Falso)
2
sen 2 > sen 1 > sen 3 (Verdadero)

Trigonometría 36
Prof: Breyner Oroya Cruz
Problema 03: Halle los valores de: cos(x  30º) 1  sen   0
Si: x  0;30º 5x  1
1   0 6
6
Solución 03: Como: 6  5x  1  0 1
0  x  30º 30º  5  5x  1 5
30º  x  30º  60º 1 x 
1
Haciendo una representación gráfica, en la 5
circunferencia trigonométrica, tenemos: 1
 x  1;
5
90º 60º
x  30º
Problema 05: Señale el Rf si se tiene que:
C.T. 30º 
x  ;  y además: f : y  3senx  2
180º 6

Solución 05: Ubicamos el arco “x” en la C.T.,


luego calculamos la variación de
cos(x  30º) sen x.
270º

1 3 Del grafico se observa que:  x  ; 
6
2 2
Sabemos que:
De la gráfica, tenemos:
y
1 3
 cos(x  30º )  x 1
2 2 
1 3 6
 cos(x  30º )  ; senx
2 2
x
Problema 04: ¿Qué valores puede tomar “x” para 0
 0
que se cumpla:
x  2 x 1
sen    siendo “θ” un
3 2 C.T.
arco del tercer cuadrante?

Solución 04: Trabajando con la condición,


tenemos:
x  2 x  1 5x  1 0  sen x  1
sen    
3 2 6 0  3 sen x  3
Como:  IIIC
2  3 sen x  2  5
y
90º
Por lo tanto:
Ranf  2;5
C.T.

180º 0

sen

1
270º

De la gráfica, tenemos:

Trigonometría 37
Prof: Breyner Oroya Cruz
y
Problemas Dirigidos
Tema: Líneas Trigonométricas
01. Indicar el mayor valor en las siguientes
expresiones: x
A) sen20 B) sen70 C) sen100
D) sen300 E) sen230 

02. Indicar verdadero (V) o falso (F) según


corresponda: sen cos (1  sen )
A)  B)
I. sen20  cos20 2 2
II. cos190  cos300 sen cos  sen
C)  D) 
III. sen100  cos350 2 2
A) VVV B) FFF C) FFV cos 
D) VFF E) VVF E)
2
03. Se define el valor absoluto de un número
06. Calcular el área de la región sombreada.
real “x”, como:
  x;x  0 y
x 
  x;x  0
Según esto, reducir:
A  sen3  sen2  sen3  sen2

B  cos3  cos2  cos3  cos2 x
A  2 sen3 A  2sen2
A) B)
B  2cos3 B  2cos2
A  2sen3 A  2sen3 C.T.
C) D)
B  2cos2 B  2cos3
1  cos  1  sen 
A  2sen2 A) B)
E) sen  cos 
B  2cos3
1  sec  1  cos 
C) D)
04. En una circunferencia trigonométrica las cos  sen 
coordenadas del extremo de un arco θ del E) sen
 1
segundo cuadrante son  x;  . Determine
 2 07. Hallar las coordenadas del punto E.
<
las coordenadas del extremo de arco dado
y
por    .

 2 2 2 5 P
A)  ;  B)  ;  
 2 2  3 3 C.T.
  
 3 1 3 4
C)  ;  D)  ;  
 2 2  2 5 x

 5 12 
E)  ;  
 13 13  H E

05. Calcular el área de la región sombreada, a


partir de la circunferencia trigonométrica. A) (sen
; cos ) B) (sen;cos )
C) (sen;cos ) D) ( cos 
; sen)
E) ( cos ; sen)

Trigonometría 38
Prof: Breyner Oroya Cruz
08. Encuentre el conjunto de valores para x que A)  4;4 B)  4;3 C)  4;2
tanx
pertenezca al intervalo  0 y que D)  4;1 E)  4;0
1  tan2 x
cumplan con la condición.
13. Si : 45    135 y A  sen   B
  
A) 0; B) 0;  ;  Hallar el valor de: (A  B)(A  B)
2 4 2
1 1
  3  3 A)  B) C) 1
C) 0;  ; D) 0;  ; 2 2
4 2 4 4 4
2 2
3 D)  E)
E) 0; 2 2
4
14. Calcular el máximo valor de:
09. ¿Cuál de las siguientes razones M  sen2  cos2   2(sen   3cos )
trigonométricas es negativa y creciente A) 2 B) 4 C) 6
para los ángulos comprendidos entre 460° D) 8 E) 10
y 470°?
A) cosecante B) tangente 15. Determine la suma del máximo y mínimo
C) coseno D) cotangente 3  sen
valor que adopta la expresión:
E) seno 4  sen
22 21 23
10. En la circunferencia trigonométrica A) B) C)
mostrada, calcule MQ si el punto M es de 15 16 15
tangencia. 25 15
D) E)
y 16 13
Q
16. Si   IIC , halle la variación de la expresión
M E  (cos   1)(cos   2)

A) 1;1 B)  2;0 C)  0;2

P D)  2;2 E) 0;2
x
17. Si se cumple que 170    280 , halle el
4sen(  60)  1
menor valor que adopta
5
A) – 3 B) – 2 C) – 1
A) tan B) cot 
D) 0 E) 1
C)  cot  D) cos   cot 
E) sen  cot     
18. Si  ; tan 
además,     1,
3  12 
Tema: Variaciones y Extensiones calcule la medida del ángulo 2α.
11. Determine la extensión de la siguiente
sen   2 A)  B)  C) 3 
expresión: E  3 2 4
3
D) 2 E) 5
 1 1  1  2 3 6
A)   ;  B)  1;   C)  1;  
 3 3  3  3
1 3
 1  1 19. Si   sen  1 ; 0    , calcule la
D)  1;   E)  1;  2 2
 2  3 suma del máximo y mínimo valor de
 
12. Hallar la variación de “m” para que sea sen    
 6
m 1
posible la relación: cos   A) – 1 B) 0 C) 1/2
3 D) 1 E) 3/2

Trigonometría 39
Prof: Breyner Oroya Cruz
20. Determine el máximo valor que adopta: 3sen  3sen 
A) 3sen B) C)
  cos   2 4
E  3 sen  
 3  sen 
D) 2sen E)
A) 1/3 B) 1/2 C) 2/3 4
D) 3/2 E) 1
06. Calcule el área de la región sombreada.
Problemas Propuestos y

 
01. Si: 0      señale verdaderas (V) o
2
falsas (F) las siguientes proposiciones:
I. sen   sen 
x
II. cos   cos 
III. sen   sen 
IV. cos   cos  C.T.
A) VVVV B) FVVF C) FVFF
1 1
D) VFFF E) FVFV A) (sen .cos ) B) (sen .cos )
2 4
02. Señale el mayor valor: 1 1
A) cos10 B) cos120 C) cos200 C) (sen   cos ) D) (sen   cos )
2 4
D) cos240 E) cos300
1
E) (sen   cos )
03. Señale verdadero (V) o falso (F) en las 2
siguientes proposiciones:
I. sen2  sen3 07. Calcular el área de la región triangular
BMB’.
II. cos5  cos6
y
III. sen3  sen4 B
IV. cos 4  cos5
A) VFFF B) VFVF C) VFFV
D) VVVF E) FFVV
A' A
04. Cuando el ángulo “x” aumenta de 90° a x
180°. ¿Cuál de las siguientes afirmaciones
es cierta?
A) El seno aumenta C.T. M
B' 
B) El coseno aumenta
C) El cosecante aumenta
D) La secante disminuye A) sen B)  sen C) cos
E) La cotangente aumenta D)  cos  E) 1

05. Del gráfico, calcule MN en términos de θ. 08. En la C.T. mostrada, hallar el área de la
y región sombreada en función de “θ”.
y

 M 

x
x
N
C.T.
C.T.

Trigonometría 40
Prof: Breyner Oroya Cruz
A) 0,5cos B) 0,5tan  C)  sec  A)  1;4 B)  1;5 C)  2;2
D)  tan  E) 0,5sen.cos  D)  3;5 E)  5;1

09. Calcule el área de la región sombreada. 13. Calcule la extensión de la expresión:


y E  2  3cos2 
A)  1;2 B)  1;3 C)  5;1
D)  1;1 E)  2;2


x
14. Si   0;  , halle la extensión de la
expresión: M  3  4sen 
A) 3;7 B) 4;7 C) 3;7
C.T.
D)  4;7 E)  1;7

sen  sen  15. Calcule el valor de k, si 2sen   3k  1


A) B)
2(1  cos  ) 2(1  cos  )
 1  1  1
cos  sen  A)  1;  B)  1;  C)  1; 
C) D)  3  3  3
2(1  cos  ) 2(1  sen  )
D)  3;1 E)  2;2
sen 
E)
2(1  cos  )
16. Si:    , además: K  2sen2  3cos2 
Calcule: Kmáx .Kmín
2
10. Calcule la ordenada del punto P. Si:   A) – 8 B) – 6 C) – 4
3
y D) 4 E) 6

  2 
17. Si:    ;  , halle la expresión:
6 3 
2sen   1
E
x 3
2  2  2 4
P A)  ; 3  B)  ;1 C)  ; 
3  3  3 3 
C.T. 2 3 
2 
D)  ; 2  E)  ; 
3   3 2 
A) 1  2 B) 3 2 C) 3 3
D) 2  5 E) 3 4 18. Calcule el mínimo valor de la expresión:
K  cos (cos   1)
11. ¿Cuál de las siguientes igualdades es
1 1 1
posible? A)  B)  C)
5 4 2 2
A) sen   B) cos   2 1
2 D) E) 1
 4
C) sen   D) cos  
3
3  2sen
 19. Señale la variación de: E 
E) sen   1  sen
4
3 5 1
A)  ;  B)  ;  C)  ; 
12. Calcule la extensión de la expresión: 2 2 2
R  3sen   2 D) 2;  E) 3; 

Trigonometría 41
Prof: Breyner Oroya Cruz
20. Señale la extensión de:
R  2sen(2cos )  1
A) 2sen1  1;2 B) 2cos2  1;2
C) 2sen1  1;3 D)  1;3
E) 1;3

Problemas Dirigidos
01. C 02. C 03. E 04. C 05. C
06. D 07. A 08. C 09. B 10. C
11. B 12. C 13. A 14. E 15. A
16. E 17. C 18. D 19. B 20. D
Problemas Propuestos
01. C 02. A 03. B 04. C 05. B
06. B 07. C 08. B 09. B 10. B
11. E 12. B 13. A 14. A 15. A
16. B 17. B 18. A 19. B 20. D

Trigonometría 42
El presente tema tiene como objeto aprender a B) Ángulos cuya medida es mayor a 360º
expresar en términos de las R.T.s En este caso se procede de la siguiente
correspondientes a un ángulo de cualquier manera.
magnitud, positivo o negativo; para un mejor Si a un ángulo positivo “β” mayor que una
análisis se hará el tratamiento por casos: vuelta lo dividimos entre 360º nos da como
cociente “n” y residuo “α”. Es decir:
A) Ángulos cuya medida están 90º;360º  360º
   n  360º  
En este caso, el ángulo original “  ” se  n
descompone como la suma o resta de un Las R.T. de “β” y las R.T. de “α” son iguales,
ángulo cuadrantal (90º; 180º; 270º ó 360º) con por tanto:
un ángulo que sea agudo; para luego aplicar R.T.()  R.T.(n  360º   ) ; n  
Esta propiedad la veremos en dos partes:
R.T.()  R.T.( )
  180º   
RT    RT() Ejemplos:
  360º   
RT     
RT  90º     CO  RT()  sen750º
  270º   
   750º 360º
 750º  2  360º  30º
30º 2
Donde el signo (±) que deberá anteponerse al
resultado dependerá del cuadrante al que sen750º  sen(2  360º  30º )
pertenezca el ángulo original “  ”. 1
sen750º  sen30º 
2
Ejemplos:
 tan10000º
IIC
10000º 360º
 sen100º  sen(90º 10º)   cos10º
280º 27
IIC
10000º  27  360º  280º
 sec123º  sec(90º 33º)   csc 33º
tan1000º  tan(27  360º  280º )  tan280º
IIIC tan1000º  tan(360º 80º )   tan80º
 tan250º  tan(270º 20)   cot 20º
IVC  sen(10   )  sen(5  2  )  sen 
 cos345º  cos(270º 75)  sen75º
 tan(35   )  tan(34     )  tan(   )
IIC

 csc172º  csc(180º 8)   csc 8º tan(35   )   tan 

IIIC
 cos 4377  cos(4376   )
 tan190º  tan(180º 10)   tan10º cos 4377  cos   1
IVC

 sec 333º  sec(270º 63)   csc 63º 


 sen43
IC
7
Dividimos sin el π:
 cot 400º  cot(360º 40)   cot 40º
43 7 43 1
 6
1 6 7 7

Trigonometría 43
Prof: Breyner Oroya Cruz
  1  cos( 143º )  cos143º
sen 43  sen  6   
7  7 cos( 143º )  cos(90º  53º )
    cos( 143º )   sen53º
sen 43  sen  6    sen
7  7  7
cos( 143º )  
4
5
315
 tan 
7  tan( 2013º )   tan2013º
Dividimos sin el π: tan( 2013º )   tan(213º )
315 8 315 3 tan( 2013º )   tan(180º 33º )
  39 
3 39 8 8 tan( 2013º )   tan33º
315  
3  3 
tan   tan  39     tan  39  8 
7  
8   Ángulos Relacionados:
315  3 
  tan  38   
8 
tan
7  Caso Nº 1:
315  3  3 sen x   seny
  tan     tan 
8 
tan x  y  180º cos x   cos y
7  8
 tan x   tan y

C) Ángulos de Medida Negativa Caso Nº 2:
Se procede de la siguiente manera: sen x   cos y
sen (  )  sen( ) cot(  )   cot( ) 
x  y  270º sec x   csc y
cos (  )  cos( ) sec (  )  sec( )  tan x   cot y

tan (  )   tan( ) csc(  )  csc( )
Caso Nº 3:
 sen x  seny
Podemos deducir que: 
x  y  360º cos x   cos y
 El signo negativo del ángulo sale y  tan x   tan y
antecede a la R.T. y se mantiene en las 
R.T.s Seno, Tangente, Contangente y
Cosecante (Funciones Impares). Ejemplo:
 El signo negativo del ángulo se cambia por  2 3 4
el signo positivo en las R.T.s Coseno y M  cos  cos  cos  cos
7 7 7 7
Secante (Funciones Pares).
5 6
 cos  cos
Ejemplos: 7 7
 sen( 45º )   sen45º
En esta expresión note que:
sen(45º )  
2  6  6
    cos   cos
2 7 7 7 7
2 5  2 5
 cot ( 166º )   cot164º     cos   cos
7 7 7 7
cot ( 166º )   cot(180º 16º ) 3 4 3 4
    cos   cos
cot ( 166º )   cot16º 7 7 7 7
24
cot ( 166º )   Luego:
7
6 5 3 4
M   cos  cos  cos  cos
 csc( 330º )   csc 330º 7 7 7 7
csc( 330º )   csc(360º 30º )
5 6
 cos  cos
7 7
csc( 330º )  csc 30º
csc( 330º )  2 Reduciendo, quedaría M = 0.

Trigonometría 44
Prof: Breyner Oroya Cruz
 7   
sen   sen  
Problemas Dirigidos  12    12 
   7 
cos   cos  
Tema: R.T. de Ángulos Positivos Menores a 360º  12   12 
01. Reducir la siguiente expresión: A) – 2 B) – 1 C) 0
sen(90  x).tan(360  x).cos(180  x) D) 1 E) 2
cos(270  x).cos(180  x)
08. En qué tipo de triangulo ABC se cumple:
A) – 2 B) – 1 C) 0
D) 1 E) 2 sen(2A 2B 3C)  sen(2A 3B 2C)
A) Equilátero B) Isósceles
02. Simplificar: C) Rectángulo D) Acutángulo
E) Obtusángulo
   3 
sen(   ).cos(2  ).cot     .sen   
2   2  09. Si “α” es un ángulo en posición normal del
 
sen().tan(2  ).cos(  ).tan     12
2  cuarto cuadrante, tal que sen    ,
13
A) sen() B)  sen() C) cos() calcular:
D)  cos() E) tan() tan(180  )  sen(  180)  cos(90  )
12 5 12
03. Calcular: E  sen150
 .cot 225.tan220 A)  B)  C)
5 12 5
3 3 3 5
A) B) C) D) E) 1
6 4 3 12
D) 1 E) 2
10. Con los datos de la figura, halle el valor de
04. Obtenga el valor de M, si:

 5   4  la expresión 17 cos( )  2sen  
sen   .tan   6
M  6   3 
cos 60 y
A) 2 B) 3 C) 3
x
D) 2 E) 1

2k  1  3  k
05. Si: tan(   )  y cot     , (1; 4)
3  2  2
A) 1 B) 2 C) 3
 21 
calcule el valor de cot    D) 4 E) 5
 2 
A) – 7 B) 0 C) 7 Tema: R.T. de Ángulos Positivos Mayores a 360º
1 1 11. Calcular el valor de:
D) E) 
7 7  15 
P  cos1560  tan  
 4 
06. Calcule el valor de:
A) 1,0 B) – 1,0 C) 0,5
  5   5  D) – 0,5 E) 1,5
tan     cot    4 
2 4   
 5  12. Dados:
cos  
 4  P  tan 400  cos810
A) 2 2 B)  2 C) 2 Q  cot 760.sen 450
R  tan1125.sec 720
D) 2 2 E) 4
Indique a alternativa correcta.
07. Calcule el valor de la siguiente expresión: A) P  Q  R B) P  R  Q
C) Q  P  R D) Q  R  P
E) P  Q  R

Trigonometría 45
Prof: Breyner Oroya Cruz
13. Reducir la siguiente expresión: 20. Calcule la suma de los ángulos positivos
sen(3780)  cos(7470)  csc(1350 ) menores que 630°, siendo el seno de cada
E uno de ellos igual a coseno de 960°
tan(2025).sec(900)
A) 1110° B) 1140° C) 1125°
A) 1 B) – 1 C) 0,5 D) 1130° E) 1120°
D) – 0,5 E) 1,5

14. Si “x” es un ángulo agudo, tal que: Tema: R.T. de Ángulos Negativos
tan(2018).tan(x 2018)  1 21. Reduzca la siguiente expresión:
sen(x).csc(x)  cos(x).sec( x)  tan(x).cot( x)
Hallar x.
A) 12° B) 14° C) 16° A) 1 B) 2 C) 0
D) 18° E) 20° D) – 1 E) – 2

 22. Simplificar la siguiente expresión:



15. Hallar el valor de cot (2k  1)  ; k  E  sen(37).cos(60).tan(45)
 4
A) 0,1 B) 0,2 C) 0,3
A) 1 B) (1)k C) (1)k 1 D) 0,4 E) 0,5
D) (1) k
2 E) – 1
23. Simplificar la siguiente expresión:
16. Simplificar: sen(300) cos(200) tan(400)
E  
   5  sen( 30) cos( 20) tan( 40)
cot   x  .csc(x 4).sen   x
 2   2  A)  3 B)  2 C) – 1
E
 3   3   13  D) 2 E) 3
cos  x   
2   2   2
.tan x .sec x
    
A) sen(x) B) cos(x) C) tan(x) 24. Si se “x” una ángulo agudos, se cumple:
D)  cos(x) E) cot(x) sen(x)  2cos( x)  3cos(180  x)
Determine el valor de: tan( x)
17. Al simplificar: A) 5 B) – 5 C) – 1/5
 135  D) 1/5 E) 1
sen   x  .tan(123  x)
 2 
25. Reducir la siguiente expresión:
 135 
cos   x E  tan(1).tan(2).tan(3)...tan(89)
 2 
Se obtiene: A)  tan(1) B)  tan89 (1) C)  tan(89)
A) – 1/2 B) 1 C) – 1 D) 1 E) – 1
D) 1/2 E) 2 CEPRU 2017–I
26. Si se cumple que:
18. Hallar el valor de: 5sen(2017).cos(2018)  Acos(B )
 89   22  2  41  Determine: A + B
cos   .tan    sen   A) 38 B) 39 C) 40
 6   3   4 
A) – 1 B) 1/2 C) – 1/2 D) 41 E) 42
D) 1 E) 2
sen( 510)
27. Si:  sen( ) ;  0;2
19. Si α y β son ángulos coterminales, cos(840)
simplificar: Calcule cos(4)
cot(3  3  45)  cos .sec  A) – 1,0 B) 0 C) – 0,5
M
 8  8  90  D) 1,0 E) 0,5
sen  
 2 
28. Si: sen20  a , calcule: cos(2410)
A) 2 B) 2 2 C) 3 2
1 a a
D) 2 2 E)  2 A) B) C)
1 a 2
1 a 2
1  a2
D) a E) – a

Trigonometría 46
Prof: Breyner Oroya Cruz
29. Al reducir la expresión: 07. Si A y B son ángulos complementarios,
sen( 1085).tan( 2525) simplifique la expresión:
E , se obtiene:
cos( 1805).tan( 3245) sen(A  2B).tan(2 A  3B)
A)  tan5 B) tan5 C) cot 5 cos(2 A  B).tan(4 A  3B)
D) sen5 E) sen5 CEPRU 2017–II A) – 2 B) – 1 C) + 1
D) + 2 E) + 3
30. Señale el valor que toma:
08. Calcule el valor de:
 2018   2019   2020 
sen    .cos   .tan        7   5   11 
 2   3   6  tan    tan    tan    tan  
 12   12   12   12 
A) – 1 B) 1 C) 2
D) – 2 E) 0 A) – 4 B) 4 C) – 6
D) 6 E) 0

Problemas Propuestos 09. De la figura, calcule: tan

01. Simplificar: y
tan(180  x).cos(90  x) (2;8)
R
cot(270  x).sen(360  x)
A) 1 B) 0 C) – 1
D) 1/2 E) – 1/2 x

02. Señale el valor de la siguiente expresión:
A) – 2 B) – 4 C) 0
2sen(100  x) 3 tan(240  x)
E  D) 4 E) 1/4
sen(80  x) cot(130  x)
A) 1 B) 2 C) 3 10. De acuerdo con el grafico, reduzca la
D) 4 E) 5 tan   cot 
expresión:
tan 
03. ¿Cuál es el valor de “m” para que se
cumpla la siguiente igualdad? y
 
m.tan   x   cot(   x)  3cot(x)
2  
A) – 4 B) – 2 C) 2
D) 4 E) 1 x

04. Si se cumple que:
   3 
tan   x   cot   x   3 A) tan2  B)  tan  C) 0
2   2 
D) cot 2  E)  cot 2 
Calcule: tan x  cot x
2 2

A) 5 B) 6 C) 7
D) 9 E) 11 11. Calcule: cos1200.sen765
2 2 2
A) B) C) 
tan300  2tan120 2 4 2
05. Calcule el valor de:
tan150 2 2
A) – 9 B) – 3 C) 3 D)  E) 
4 8
D) 9 E) 3 3
 17 
06. Hallar el valor de la siguiente expresión: csc 840  cot  
12. Simplificar:  3 
sen310º  cos310º  sec 250º  sen200º
E cos2580
sen130º  cos50º  cos180º
2 3
A) 2 B) –1 C) –2 A) B) 3 C) 2
D) 1 E) 0 3
D) 4 E) 3

Trigonometría 47
Prof: Breyner Oroya Cruz
13. Siendo: cos100  n , determine sen1000 A) – 1/4 B) – 1/2 C) 0
en términos de “n”. D) 1/2 E) 1/4
A) n2  1 B) 1 n2 C)  1 n2
(4n 1)
D)  1  n2 E) – n 20. Si: tan   cot   2 y     ,
2
n , determine el valor de: csc .cos 
14. Simplifique:
A) 2 B) 3 C) 3
 5   7   9 
tan     .sen     .sen    D) 6 E) 2
E  2   2   2 
cos(5   ).cos(7   ).cot(9   )
21. Calcule: R  tan(60).sec(30)
A) – 2 B) – 1 C) 0
D) 1 E) 2 A) 6 B) – 6 C) 2
2 3
15. Simplifique la siguiente expresión: D) – 2 E) 
3
sen2 x  cos2 x
E
 
37  sen( 20)  3 sen20
cos   x   cos  35  x  22. Siendo: tan  
 2  2sen160
A) sen x  cos x B) sen x  cos x ¿Cuál es el valor agudo de θ?
C) cos x  sen x D) sen x  cos x A) 37° B) 45° C) 53°
D) 30° E) 60°
E) cosx
23. Calcule:
7
16. Halle “θ”, tal que:    4 y 4cos(120)  3cot(315)  4sec2 (225)
2 A) 1 B) 2 C) 3
 D) – 4 E) – 2
tan    cot  
7
24. Calcule: sen(2400).cos(1200)
51 26 53
A) B) C)
14 7 14 3 3 3
A)  B) C)
27 55 2 4 2
D) E)
7 14 3 3
D)  E) 
4 6
 1749   327  
17. Simplifique: 4 tan    5 cot  
 4   4  25. Reducir:
A) – 1 B) 3 C) 5 a2 cos(0)  6ab tan(360)  b2sec(360)
D) 9 E) 11 2abcos( 540)  a2sen(90)  b2sen(270)
 2001  ab
sen   A) B) a  b C) a  b
 6  ab
18. Si:  cot  , halle
 2002    2003  ab
tan   .cos   D) E) a2  b2
 3   4  ab
el valor de cos(  ).sen(4  ) .
26. Si: sen(x)  2cos(x)  2sen(x) ; donde “x”
6 3 2
A) B) C) es un ángulo agudo.
5 2 2
Calcular: M  sec(x)  csc(x)
6 6
D) E) 5 5 13
3 4 A) B)  C)
2 2 6
 (2n  1)  13 5
sen    D)  E) 
 2   tan(2n   )  2 6 5
19. Si ,
cos(n   )
n . Calcular: sen .cos 

Trigonometría 48
Prof: Breyner Oroya Cruz
cos(20  x) tan(41  x)
27. Reducir: E  
cos(  x)  3  Problemas Dirigidos
cot  x 
 2  01. D 02. B 03. A 04. E 05. E 06. D
A) – 1 B) – 2 C) 0 07. C 08. B 09. A 10. E 11. C 12. D
D) 1 E) 2 13. A 14. B 15. B 16. C 17. C 18. A
19. B 20. A 21. D 22. C 23. E 24. B
28. Si   60 , entonces el valor de: 25. E 26. D 27. D 28. E 29. A 30. E
 39  Problemas Propuestos
csc     .sec( 85   )
W  2  01. A 02. E 03. B 04. C 05. B 06. D
 73  07. 08. 09. 10. 11. 12.
cos(  73).tan    C E E C D A
 2  13. D 14. D 15. C 16. A 17. A 18. A
A) 8 3 B) 4 3 C)  3 19. D 20. E 21. D 22. B 23. C 24. D
D) 8 E) 3 25. A 26. D 27. E 28. A 29. C 30. B

29. Si:      , calcule:


sen(cos   cos )  1
E  tan(  )
cos(tan   tan )  1
A) 1 B) – 1 C) 1/2
D) – 1/2 E) 2

30. De acuerdo a la figura y si tan2   9 ,


calcular 10(sen   sen )  sen(  )

 x

A) – 3 B) 1 C) – 1
D) 3 E) 2

Trigonometría 49
A.3. Identidades Por Cociente
IDENTIDADES TRIGONOMÉTRICAS sen x   
tan x  x   (2n  1) / n  
Una identidad trigonométrica es una igualdad cos x  2 
que contiene expresiones trigonométricas que cos x
se cumplen para todo valor admisible del cot x  x   n / n  
sen x
ángulo.

A) Identidades Fundamentales B) Identidades Auxiliares


Las identidades trigonométricas  tan x  cot x  sec x.csc x
fundamentales sirven de base para la  sec 2 x  csc 2 x  sec 2 x.csc 2 x
demostración de otras identidades más  sen4 x  cos4 x  1  2sen2 x.cos2 x
complejas.
 sen6 x  cos6 x  1  3sen2 x.cos2 x
Se clasifican en:
 (1 senx  cos x)2  2(1 senx).(1  cos x)
A.1. Identidades Pitagóricas  (senx  cosx  1)(senx  cosx  1)  2senx.cosx
sen x  1  cos x sen x 1 cos x cos x 1 senx
2 2

sen2 x  cos2 x  1    
1  cos x sen x 1  sen x cos x
cos x  1  sen x
2 2

x   Si: a sen x  bcos x  a2  b2


Entonces:
sec 2 x  tan2 x  1
sec 2 x  tan2 x  1 2 sen x 
a
 cos x 
b
tan x  sec x  1
2
a b
2 2
a  b2
2

    Si: sec x  tanx  k


 x   (2n  1) / n  
 2  Entonces:
1   
csc x  cot x  1
2 2
sec x  tan x  x   (2n  1) ;n  
csc 2 x  cot 2 x  1 2 k  2 
cot x  csc x  1
2

 Si: csc x  cot x  m


 x   n / n  
Entonces:
1
csc x  cot x  x   n;n  
A.2. Identidades Reciprocas m
1
sen x.csc x  1  csc x  Recordar:
sen x
x   n / n   Verso de “x”: ver (x)  1  cos(x)

1 Converso de “x”: cov(x)  1  sen(x)


cos x.sec x  1  sec x 
cos x Ex secante de “x”: ex sed(x)  sec(x)  1
  
x   (2n  1) / n  
 2  Sugerencia:
1 A continuación te proponemos algunas guías o
tan x.cot x  1  cot x  sugerencias que te servirán para desarrollar
tan x
ejercicios, estas son:
    Escoger el miembro más complicado de la
 x   n / n  
 2  identidad.
 Colocar el miembro escogido en términos
de senos y cosenos.

Trigonometría 50
Prof: Breyner Oroya Cruz
 Colocar el miembro escogido en términos Simplificaciones:
de senos y cosenos.
 Hacer uso de identidades algebraicas, 1  cos x sen x
según sea el caso. Problema 03: Simplificar: E  
sen x 1  cos x
 Cuando haya potencias puede ser útiles
hacer factorizaciones.
 De las identidades fundamentales se Solución 03: Efectuando la diferencia de
podrán deducir otras. fracciones
Los ejercicios sobre identidades 1 cos2 x

trigonométricas, son de 4 tipos: (1  cos x)(1  cos x)  (sen x)(sen x)


 Demostraciones. E
(sen x)(1  cos x)
 Simplificaciones.
 Condicionales. sen2 x  sen2 x
E 0
 Eliminación de ángulos. (sen x)(1  cos x)

Demostraciones: Condicionales
Demostrar una identidad consiste en que
Dada una o varias condiciones, se pide hallar
ambos miembros de la igualdad propuesta son
una relación en términos de dicha o dichas
equivalentes. Para lograr dicho objetivo se
condiciones.
siguen los siguientes pasos:
 Se escoge el miembro “más complicado”.
1
 Se lleva a Senos y Cosenos (por lo Problema 04: Si: sen x  cos x 
general). 2
 Se utilizan las identidades fundamentales y Hallar: senx.cos x
las diferentes operaciones algebraicas.
2
 1
Problemas Resueltos Solución 04: Del dato: (sen x  cos x)   
2

2
Problema 01: Demostrar: sen x  cos x  2sen x.cos x 
2 2 1
sec x(1 sen2 x)csc x  cot x 4
1
1  2sen x.cos x 
Solución 01: Se escoge el 1er miembro: 4
sec x(1  sen2 x)csc x  3
sen x.cos x  
Se lleva a senos y cosenos: 8
1 1
(cos2 x) 
cos x sen x Eliminación de Ángulos
1 La idea central es eliminar todas las
Se efectúa: cos x   expresiones algebraicas, y que al final se den
sen x relaciones independientes de la variable.
cot x  cot x L.q.q.d
Problema 05: Eliminar “x” a partir de:
Problema 02: Demostrar: senx  a  cosx  b
sec x  tanx  11 sec x  tanx  2tanx
Solución 05: Elevando al cuadrado, tenemos:
Solución 02: Se escoge el 1er miembro: sen x  a  sen2 x  a 2 ...(*)
sec x  tan x  11  sec x  tan x   cos x  b  cos2 x  b 2 ...(**)
sec x  (tan x  1)sec x  (tan x  1)  De (*) + (**), tenemos:
Se efectúa: sen2 x  cos2 x  a2  b 2
(sec x)2  (tan x  1)2  1  a2  b2
(1  tan x)  (tan x  2 tan x  1) 
2 2

1  tan2 x  tan2 x  2 tan x  1


2 tan x  2 tan x L.q.q.d

Trigonometría 51
Prof: Breyner Oroya Cruz
09. Simplifica la expresión:
Problemas Dirigidos tan2 (tan4   3 sec 2 )
1
1  sec 3 
Tema: Identidades Fundamentales A)  sec 3  B) sec 3  C)  csc 3 
01. Reducir: D) csc 3
E) sen  3

E  sen2.sec   cos 
A) sec  B) csc  C) tan Tema: Identidades Auxiliares
D) cot  E) 1 2
10. Reducir: E   1  sen 
tan   cot 
02. Reducir: A) 0 B) cos  C) 1
1 1 1 D) sen 
E   E) tan
1  cos2  csc 2   1 1  sen2
A) tan2  B) tan2  C) csc 2  1
11. Simplificar: E   tan   csc 
csc   cot 
D) cot 2
E) sen 2
A) sec .csc  B) sen .cos 
 sen x  tan x  C) sec   csc  D) sen   cos 
03. Reducir: M    .cot x
 cot x  csc x  E) sec   csc 
A) 1 B) sen x C) cosx
12. Si: sec 2 .csc 2   7 , calcule:
D) 2 E) 0
sec 4   2sec 2 .csc 2   csc 4 
04. Reducir: A) 49 B) 42 C) 36
(2 tan   cot )2  (tan   2cot )2 D) 25 E) 48
E
tan2   cot 2  cos x
A) 2 B) 10 C) 5 13. Simplificar: E  tan x 
1  sen x
D) 3 E) 9
A) sec x B) csc x C) sen x
1 1 D) cos x E) tan x
05. Si:   A  B.tanC x
1  sen x csc x  1 14. Determine el valor de “k” para que la
Entonces el valor de F  A  B  C , es: k  sen x 1  cos x
siguiente expresión  ,
A) – 2 B) 0 C) – 1 1  cos x sen x
D) 2 E) 1 CEPRU 2017–II sea independiente de “x”.
A) 0 B) 1 C) – 1
06. Siendo θ un ángulo perteneciente al D) 2 E) – 2
segundo cuadrante, reducir la siguiente
expresión: 15. Si: tan   cot   3 , calcule el valor de la
E  cos  (sec   1)(sec   1) sen4  cos 4 
expresión 
A) tan B)  tan  C) 1 cos2  sen2
D) sen  E) sen A) 5 B) 3 C) 6
D) 4 E) 1/2
07. Simplificar la expresión: 16. Halle el valor de la expresión:
tan2   cot 2  E  (1  sen   cos )2 .(1  sen   cos )2
 cot 2 
cot 2   tan2   2(sen4   cos4 )
A) cot .sec  B) sec .cot  A) 4 B) 1 C) 0
C) sec 2 .cot 2  D) sec 2 .cot 2  D) 3 E) 2

E) tan2 .tan2  2  2cos x


17. La expresión 1 , es
sen x  cos x  1
08. Determine el menor valor entero que puede equivalente a:
tomar P, si se sabe que α es un ángulo A) sec x  cosx B) sec x  tanx
agudo y P  12 tan2   4 sec 2   20 C) sec x  tanx D) csc x  cot x
A) 26 B) 25 C) 24
E) csc x  cot x
D) 23 E) 15

Trigonometría 52
Prof: Breyner Oroya Cruz
Tema: Identidades Condicionales 26. Eliminar “x” a partir de:
18. Si: sen  cos   m , m  1, calcular el sen4 (x)  cos4 (x)  a sen6 (x)  cos6 (x)  b
valor de: tan   cot   sec   csc  A) 3a  2b  1 B) 3a  2b  1
m 1 m 1 C) 2a  3b  1 D) 2a  3b  1
A) B) C) m2  1
2 2 E) 3a  2b
2 2
D) E) 27. Eliminar “θ” de:
m 1 m 1
a  tan()  cot()  b  sec()  csc()
19. Si: sec 3   sec   a y tan3   tan   b A) a2  2a  b B) a2  2a  b2
Calcule el valor de: acos   bcot  C) a  a  b
2
D) a2  a  b
A) – 3 B) – 2 C) 1 E) a  2a  b
2 2
D) 2 E) 3
28. Eliminar “α” del sistema dado:
20. Si: cos   sec   4 ,
acsc()  2cos()  csc()
Calcule: (sec   cos )2
b  sen()  cos()  1
2
A) 10 B) 12 C) 14
D) 16 E) 18 A) a  b  1  2 a B) a  b  1  2 b

21. Sea x un ángulo agudo tal que C) a2  b2  1 D) a  b 1


E) a  b  1
sec 2 x 2sec x.tanx  tan2x  25
Hallar: 8cosx  senx
29. Determine la relación que elimina el arco de
A) 2 B) 1 C) 4 “θ”, si:
D) 3 E) 5
m  (tan   cot   3)2  n  (sec .csc   1)2
22. Si: sec 3x  sec x  2 A) m  n 1 B) m n2

Halle: 2cos x  tan x  3


2 C) n m4 D) 2 m  n  3
A) 1 B) tan2 x C)  tan2 x E) n m4
D) 2 E) – 1
30. Determine la relación independiente de “ϕ”
y “θ”, si:
23. Calcule: tan(x)  tan ()2
x  p sen().cos()
sec 2 ().sen2 (x)  cos2 (x) y  p sen().sen()
tan2 () 
1  sen(x).cos(x)
z  pcos()
Considere: tan(x)  0
A) x2  y2  z2  p2
A) – 2 B) – 1 C) 0
D) 1 E) 2 B) x2  y2  z2  1 p2
C) x2  y2  z2  p2
24. Si: senx  sen2 x  sen3 x  1 , halle el valor
D) x2  y2  z2  p2
de: sen3 x  csc x
A) 1 B) 2 C) 3 E) x2  y2  z2  p2
D) 4 E) 5
Problemas Propuestos
Tema: Eliminación de Variables
25. Eliminar “x” a partir de: 01. Reduzca la siguiente expresión:
tan(x)  cot(x)  a tan(x)  cot(x)  b 1 1
  csc 2 
A) a2  b2  3 B) a2  b2  3 cos2  cot 2 
C) a  b  4
2 2
D) a2  b2  4 A)  tan2  B) tan2  C) cot 2 
E) a  b  8
2 2 D)  cot 
2
E) 2 tan

Trigonometría 53
Prof: Breyner Oroya Cruz
02. Reducir: A) 1/6 B) 5/6 C) 7/3
tan .cos   sen .cot   cos(90º ) D) 5/3 E) 7/6 CEPRU 2016–II
A) cos  B)  cos  C) 0
11. Halle “x” agudo que cumple:
D) sen E) sen 
1
(sec x.csc x tan x)sen x 
03. Siendo: tan(x)  3 . Determine el valor de: 2
A) 30° B) 37° C) 45°
E  sec 2 (2x)  tan2 (2x) D) 53° E) 60°
A) 1 B) 2 C) 3
D) 4 E) 5 12. Calcule el equivalente de la expresión:
csc 2   2
04. Del gráfico, calcule: sen x  csc   cos 
2 2 2
 sec .csc 
cot   1
A) tan   1 B) 1  cot  C) 1  tan 
cosx D) cot   1 E) 1  tan 
cos 
13. Reducir:
cot  E  (sec2 .csc2   sec 2 )sen   csc 
A) 4 B) 5 C) 2 A) 2csc  B) 1  csc  C) 2sec 
D) 3 E) 6 D) 1  sec  E) sec   csc 

05. Reduzca la siguiente expresión:


4
sen2  (1  cos  )2 14. Siendo: sen   cos  
3
sec   1
Calcular: R  tan   cot 
A) 2cos B) 2tan C) 2cos  A) 2 B) 3 C) 6
D) 2sen E) 2sen 
D) 2 3 E) 6
06. Calcule el valor de la expresión:
sen2 x.tan2 x  tan2 x  cos2 x.cot 2 x  cot 2 x
tan3 x  cot 3 x
15. Al simplificar la expresión:
A) – 1 B) – 5 C) – 2 csc 2 x.csc 2 x  3
D) – 3 E) – 4 se obtiene:
A) tan x  cot x B) tan x  cot x
07. Calcule el valor de la siguiente expresión:
C) 2tan x D) 2cot x
sec 2 .csc 2   2
E) tan2 x  cot 2 x
tan2   cot 2 
A) 1/2 B) – 1 C) 1 16. Calcule el valor de la siguiente expresión:
D) 2 E) – 2
sen4   cos4   3
08. Si: sec x  tan x  m ; sen6   cos6   5
Hallar: K  sec x  tan x A) 3/2 B) 3/4 C) 2/3
A) m B) m–1 C) 2m–1 D) 1/2 E) 1/4
D) 3m–1 E) m2
sen
09. De la igualdad: 17. Reduzca la expresión:  cot 
1  cos 
2sen2 A) csc  B)  sec  C) sec 
 sen2   (cos   M)N
1  cos 
D) tan E)  csc 
Calcule: M  N
A) 2 B) 4 C) 3
18. Halle el valor de P.
D) 5 E) 1
(1  sen   cos )2 (1  sen )
P  2cos 
10. Si: 5senx  12cosx  13 , entonces el valor cos2 
1 A) 1 B) 2 C) 3
de: E  sec x  es: D) 4 E) 5
12

Trigonometría 54
Prof: Breyner Oroya Cruz
19. Si: tan   cot   3 , calcule el valor de: A) a  b(1 a2 ) B) a  b(1 a2 )
tan   cot  C) 2a  b(1  a ) 2
D) 2a  b(1  a2 )
A)  5 B) 7 C) 5
E) 2b  a(1  b2 )
D)  5 E) 2
28. Elimine la variable angular “θ” a partir de las
1 sec   tan 
20. Si: csc   sen   , calcule: siguientes condiciones:
3 cos   cot 
sen   cos   m  sen3  cos3   n
A) 2 B) 1/2 C) 4
D) 1/4 E) 3 A) m  n3  2n B) 3m  m3  2n
C) 3m  n  2
2
D) 2m2  3m  n
tan x cot x
21. Si:  ; halle el valor de la E) 3m  n  2n 3
2 3
3sen6 x  cos6 x 29. Cuál es el valor de “k” para que la siguiente
expresión:
4sen6 x  3cos6 x expresión sea independiente de “θ”.
51 50 27 k(csc   cot )(1 cos )  sen 
A) B) C) A) – 1 B) 0 C) 1
113 113 113
D) 2 E) 3
29 63
D) E)
113 113 30. Sean:
m  1  csc   csc 2   csc 3   ...
22. Si: 7sen  1  cot 2  obtener:
n  1  sec   sec 2   sec 3   ...
cot 6   cot 4   cot 2  Entonces se cumple que:
A) 6 B) 8 C) 9 2 2
D) 10 E) 11  m  1  n  1
A)     1
 m   n 
23. Si: cos2   1  cot  , calcule el valor de:  m   n 
2 2

sen   sen   1
2 6 B)     1
 m  1  n  1
A) – 15 B) 15 C) 2 2 2
D) – 2 E) 7  m   n 
C)     1
 m  1  n  1
24. Si: 1 sen   b(1 cos ) 2 2
 m  1  n  1
Calcule el equivalente de: D)     1
 m   n 
(csc   cot   1)2 ,   0;90 2 2
 2m   2n 
A) b/2 B) b C) 2b E)     1
D) 3b E) 4b  m  1  n  1

25. Halle el mínimo valor de: Problemas Dirigidos


sec 4 x  csc 4 x;x  01. A 02. D 03. B 04. C 05. E 06. D
A) 4 B) 6 C) 8 07. D 08. B 09. A 10. B 11. A 12. A
D) 10 E) 16 UNI 1996–I 13. A 14. A 15. C 16. E 17. C 18. D
19. E 20. B 21. C 22. E 23. B 24. B
26. Elimine la variable angular “θ” de la
siguientes condiciones: 25. C 26. A 27. B 28. A 29. C 30. A
sec .csc   x  sec   csc   y Problemas Propuestos
A) y2  2x  x2 B) x 2  2  y 2 01. D 02. A 03. A 04. C 05. A 06. A
07. C 08. B 09. C 10. E 11. E 12. E
C) y2  2x  x2 D) x2  2y  y2
13. A 14. C 15. A 16. C 17. A 18. B
E) y  x  2
2 2
19. A 20. E 21. A 22. A 23. C 24. C
25. C 26. A 27. D 28. B 29. C 30. C
27. Elimine “x” de:
sen   cos   a  sec   csc   b

Trigonometría 55
Las razones trigonométricas de ángulos Propiedades:
compuestos se generan a partir de la suma o I.
diferencia de dos o más ángulos conocidos o
de sus razones trigonométricas, los cuales nos sen(x  y).sen(x  y)  sen2 x  sen2 y
dan como resultado una gran variedad de cos(x  y).cos(x  y)  cos2 x  sen2 y
ángulos con sus respectivas razones
trigonométricas.
II.
R.T. de la Suma de Dos Ángulos sen(x  y)
tan x  tan y 
cos x.cos y
sen(x  y)  sen x.cos y  cos x.sen y sen(x  y)
tan x  tan y 
cos(x  y)  cos x.cos y  sen x.sen y cos x.cos y
tan x  tan y
tan(x  y)  III.
1  tan x.tan y
Si :L  asen x  bcos x ; a,b,x 

R.T. de la Diferencia de Dos Ángulos Lmax  a2  b2 Donde : a  b : constantes


Lmin   a  b
2 2
x : variables
sen(x  y)  sen x.cos y  cos x.sen y
cos(x  y)  cos x.cos y  sen x.sen y IV.
tan x  tan y Si :K  a sen x  bcos x ; a,b,x  
tan(x  y) 
1  tan x.tan y
 K  a2  b2 .sen(x  )
Donde :
Ejemplos:
Expresar las siguienes razones de angulos a2  b2
compuestos en su forma expandida: b

 sen(1  2)  a
 sen(  ) 
 cos(13  17)  V.
 cos(15  25)  tan(x  y)  tan x  tan y  tan x.tan y.tan(x  y)
 tan(x  y)  tan x  tan y  tan x.tan y.tan(x  y)
 tan(40  20) 

 VI.
 tan(36  24) 
 Si: x  y  z   ó n ; n 
tan x  tan y  tan z  tan x.tan y.tan z
Observación: cot x.cot y  cot y.cot z  cot z.cot x  1
1 1
cot(x  y)  cot(x  y) 
tan(x  y) tan(x  y) VII.
1 1  
sec(x  y)  sec(x  y)  Si: x  y  z  ó (2n  1) ; n 
cos(x  y) cos(x  y) 2 2
1 1 cot x  cot y  cot z  cot x.cot y.cot z
csc(x  y)  csc(x  y) 
sen(x  y) sen(x  y) tan x.tan y  tan y.tan z  tan z.tan x  1

Trigonometría 56
Prof: Breyner Oroya Cruz
R  tan13º  tan32º  tan13º.tan32º
Problemas Resueltos R  tan13º  tan32º  tan13º.tan32º.1
R  tan13º  tan32º  tan13º.tan32º. tan 45º
Problema 01: Calcular: R  tan 45º  1
E  (sen17º  cos13º )2  (cos17º  sen13º )2
Problema 05: Determine el valor de: cos69º
Solución 01: Efectuando el cuadrado, en la
pregunta, tenemos: Solución 05: Como 69º no es un ángulo
E  sen217º  cos2 13º  2cos13º.sen17º  conocido, tenemos que formar
cos2 17º  sen213º  2cos17º.sen13º dicho ángulo a partir de los
ángulos conocidos relacionándolos
E  1  1  2 sen(17º 13º )  2  2 sen30º en suma o resta; es decir:
1 cos 69º  cos(53º 16º )
E  2  2  3
2 cos 69º  cos53º.cos16º  sen53º.sen16º
3 24 4 7 72 28
Problema 02: Hallar: cos 69º  .  .  
P  cos80º 2sen70º.sen10º 5 25 5 25 125 125
72  28 44
cos 69º  
Solución 02: De la pregunta, tenemos: 125 125
E  cos(70º 10º )  2sen70º.sen10º
E  cos70º.cos10º  sen70º.sen10º Problema 06: Hallar Dominio y Rango:
f(x)  3senx  4cosx
 2sen70.sen10º
E  cos70º.cos10º  sen70º.sen10º Solución 06: Por simple inspección:
E  cos(70º 10º )  cos60º 
1 Dominio: x  R
2 Rango:
3 4 
Problema 03: Escribir el equivalente de: y  5  sen x  cos x 
5 5 
tan60º  cot 75º
E y  5(sen37º.sen x  cos37º.cos x)
1  tan60º.cot 75º
y  5cos(x  37º )
La extensión de coseno es:
Solución 03: Recordemos que: cot 75º  tan15º
 1  cos(x  37º)  1 5
Reemplazando en la pregunta y dándole la
forma de un ángulo compuesto, tenemos: 5  5cos(x  37º)  5
tan60º  cot 75º 5 y 5
E
1  tan60º.cot 75º  y   5;5
tan60º  tan15º
E
1  tan60º.tan15º Problema 07: Siendo:
E  tan(60º 15º ) cos20º  a , cos25º  2 b
6 2 Obtener tg 25º en términos de “a” y “b”
E  tan75º  2 3
6 2
Solución 07: Trabajando con el dato, tenemos:
sen 20º  a
Problema 04: Calcular el valor de:
R  tan13º  tan32º  tan13º tan32º sen(45º 25º )  a
1 1
.cos 25º  .sen 25º  a
Solución 04: Recordemos que: 2 2b
2
tan(x  y)  tanx  tany  tanx.tany.tan(x  y) 1
b .sen 25º  a
2
Trabajando en la pregunta, dándole la forma de sen 25º  2(b  a)
la propiedad, tenemos:

Trigonometría 57
Prof: Breyner Oroya Cruz
Nos piden: a.sen  b.cos  c
sen25º 2(b  a) b  a a.tan   b  c.sec 
tan25º   
cos25º 2b b a2 .tan2   b2  2ab tan   c 2 .(1  tan2 )
(a2  c 2 )tan2   (2ab)tan   (b2  c 2 )  0
Problema 08: Simplificar:
De la ecuación, tenemos:
E  sen2 (   )  2sen(   ).sen .cos  
2ab b2  c 2
sen2  cos2.sen2  cos2.sen2 tan   tan   tan .tan  
a2  c 2 a2  c 2
Nos piden:
Solución 08: Formando un binomio cuadrado,
2ab
tenemos:
tan   tan  a2  c 2
E  sen(  )  cos .sen  sen  (1  Cos  )
2 2 2 tan(  )  
1  tan .tan  b2  c 2
1 2
E  sen2.cos2   sen2 .sen2 a  c2
E  sen2(cos2   sen2 ) 2ab 2ab
tan(  )  
E  sen 
2 a2  b2 b2  a2

Problema 11: A partir de la gráfica, calcular: tan x


Problema 09: Siendo:
sen   sen   sen   0
a 3a
cos   cos   cos   0
Calcular: E  cos(  )  cos(  )  cos(  ) 2a
x
Solución 09: Despejando convenientemente
a
una de las variables, tenemos:
cos   cos   cos 
sen   sen   sen  Solución 11: Trabajando en la gráfica, tenemos:
Elevando al cuadrado:
a 3a
cos2  cos2  2cos.cos  cos2 
 

sen2  sen2  2sen.sen  sen2 2a
1  1  2.cos(  )  1
x
1 a
cos(  )  
2 
Por analogía:
1 1 A partir de la gráfica, tenemos:
cos(  )   ;cos(  )   2 1
2 2 x     ; tan   ; tan  
Nos piden: 3 4
E  cos(  )  cos(  )  cos(   ) Nos piden:
tan x  tan(   )
3
E tan   tan 
2 tan x 
1  tan .tan 
Problema 10: Siendo “tan α” y “tan β” las raíces 2 1

11
de la ecuación: tan x  3 4  12
asen()  bcos()  c 1 
2 1 10
Hallar: tan(  ) 3 4 12
11 12 11
tan x    1,1
Solución 10: Trabajando con el dato, tenemos: 10  12 10

Trigonometría 58
Prof: Breyner Oroya Cruz
Hallar el valor de: cot(  )
Problemas Dirigidos A) 5/7 B) 7/24 C) 5/24
D) 24/7 E) 24/5
Tema: R.T. Fundamentales de Ángulos Compuestos 09. Si: tan(5  3)  5 , tan(5  3)  2
01. Simplifique la siguiente expresión:
Calcular: K  tan(10)  cot(6)
3
sen(30  )  sen 7 3 26
2 A)  B) C)
1  sen2  9 11 9
25
sec  D) E) 3
A) 2sec  B) C) 2csc  9
2
csc  sen(45   x)
D) E) sec  10. Si: tan(x)  cot(x) 
5
y M
2 2 sen(135   x)
02. Determine el valor aproximado de la Calcule M2
siguiente expresión: 125sen(21)  1 A) 2 B) 9 C) 16
A) 43 B) 44 C) 45 D) 25 E) 36
D) 46 E) 47 11. En la figura:
03. Si: 2sen(x 45)  senx.sec 2 x
3
Calcule tan3 x  cot 3 x
A) – 2 B) 2 C) 4
D) 8 E) 16 2

04. Simplificar: 
 1
cos 60º cos15º sen60º sen15º
E x
sen30º cos15º  cos30º sen15º
El valor de x, es:
A) cot 40º B) 2 C) 1 A) 5 B) 2 C) 1
D) tan15º E) 3 D) 4 E) 3 CEPRU 2017–II

05. Si: cos(  )  2cos(  ) 3 sen50  cos50


12. Calcule: E 
Calcule cot().cot() sen25  cos25
A) 3 B) 1/4 C) 2 A)  2 B) 2 C) – 1
D) 1/3 E) 4
D) 1 E) 3
24 25
06. Si: cot    , IIC y sec    , 13. Si:
7 7
 IIIC Calcular el valor de cos(  ) k(sen80  sen10)  2(cos25  3 cos65)
Entonces, el valor de k es:
625 336 650
A) B) C) A) 3 B) 1 C) – 1
336 625 333 D) 0 E) 2 CEPRU 2017–I
333 125
D) E) 2sen57  cos 27
650 336 14. Hallar el valor de:
2sen72  cos 27
07. Escribir el equivalente de:
1 1
tan60º  tan15º A) B) C) 3
E 3 2
1  tan60º.tan15º
D) 1 E) 2
A) 2  3 B) 2  3 C) 1  3
15. Calcule el valor de:
D) 1  3 E) 1
sec 28.sec17  2 tan28.tan17
3 3 A) 1 B) 2 C) 2
08. Si: cos   y sen  
5 5 D) 3 E) 3

Trigonometría 59
Prof: Breyner Oroya Cruz
Tema: R.T. Auxiliares de Ángulos Compuestos Calcule: E  tanB(tanC tanA)
16. Cuál es la extensión de: A) 5/9 B) 16/5 C) 3/3
K  5sen  12cos   6 D) 9/4 E) 9/16
A) 5;12 B) 13;13 C) 19;7
tan50  tan10  tan40
25. Evaluar:
D) 5;19 E) 6;13 cot 260
A) – 2 B) – 1 C) 1
D) 2 E) 3
E  3 sen  cos   1
17. Siendo:
R  3sen  4cos   1 26. Siendo ABCD un cuadrado, calcule tan
Determine Emáx  Rmín
A) – 2 B) – 1 C) 0 A B
53
D) 1 E) 2

18. Si: E  sen(x 45)  2 cos(x 45)


Determine: 3 Emáx  1
A) 1 B) 2 C) 3 
D) 4 E) 5 D C
19. Calcule:
E  cos(x 45).cos(x 45)  sen2 (x) 23 15 2
A) B) C)
A) 0,5 B) 1,0 C) 1,5 15 23 3
D) 2,0 E) 2,5 3 4
D) E)
2 3
20. Determinar el valor de:
sen2 55  sen85.sen25 27. En un triángulo ABC se cumple que:
A) 1 B) 1/2 C) – 1/4 tan(B)  tan(C)  2 3  2
D) 1/3 E) 1/4
tan(B).tan(C)  3cot(A)  2 3
21. Si: M  tan10  tan50  3 tan10.tan50 Calcule tan(A)
Determine: (M  tan10)(M  tan20) A) 1/3 B) 1/2 C) 3/4
A) 1 B) 2 C) 4 D) 1 E) 2
D) 8 E) 16
28. Simplificar:
22. Halle la medida del ángulo α si se cumple  sen(50  x) 
que: P  tan x  tan 40
 cos50 .cos x 
tan   tan45  tan50.cot 85  cot 85
A) 1 B) tan x C) cot x
Además   0;90 D) 2 E) 3/2
A) 38° B) 48° C) 15°
D) 35° E) 50° UNMSM 2009–I 29. Reduzca la siguiente expresión:
 sen1 
23. Siendo:   tan1  cot 2
m  tan x  tan2x  tan3x  cos 2.cos1 
n  tan2x.tan3x(tan x  2 tan5x) A) tan2 B) cot 2 C) 1
D) 2 E) tan1.cot 2
p  2 tan2x  tan5x
Halle: “ m  n  p ” 30. Si se cumple:
A) 2 tan5x B) 3 tan5x C) 4 tan5x (1 tan1)(1 tan2)...(1  tan45)  2x
D) 5 tan5x E) 6 tan5x Entonces el valor de “x” será:
A) 22 B) 23 C) 24
24. En un triángulo ABC se cumple: D) 25 E) 26
tan A tanB tanC
 
2 3 4

Trigonometría 60
Prof: Breyner Oroya Cruz
09. Si: tan(45  )  4 , calcule tan()
Problemas Propuestos A) – 4/5 B) 3/5 C) 4/5
D) – 3/5 E) – 2/5
01. Calcule el valor de la siguiente expresión:
10. Si: tan(  )  2 y tan(  )  3 ,
3
sen(23)  calcule tan(2)
10
A) 1 B) – 1 C) 5
3 2 3 13 D) 1/5 E) – 1/5
A) B) C)
5 5 10
11. En el gráfico:
2 3
D) E)
10 10
 
x
02. Si: 2sen(45  )  5sen(37  )
Calcule: tan()
A) 1/5 B) 3/4 C) 2/5 3 4 2
D) 4/3 E) 1/2 El valor de “x”, es:
A) 3 3 B) 2 3 C) 6 3
03. Reduzca la siguiente expresión:
sen(x  y) D) 3 6 E) 3 2 CEPRU 2016–II
 cot x.tan y
sen x.cos y
12. Si α y β son ángulos agudos, tal que
A) – 1 B) tanx.cot y C) 1
tan   sen2 y cot   sec 2   tan2  ,
D) tany.cot x E) cot x.cot y
entonces el valor de sen(  ) ,es:
04. Hallar el máximo valor de: 2
A) 2 B) 3 C)
E  sen(30º x)  cos(60º x) 2
A) 1 B) 2/3 C) 4/3 3 1
D) 5/3 E) 1/7 D) E) CEPRU 2015–I
2 2
05. Si: cosx.cosy  2senx.seny
13. Si: ksen50  3sen20  3 cos20
cos(x  y)
Calcule: Determine “k”.
cos(x  y)
A) 3 3 B) 2 3 C) 4 3
A) 1 B) 1/3 C) 3
D) 1/2 E) 2 D) 2 2 E) 3 2

1 4sen17  3cos17
06. Si: cos x  , calcule el valor de la 14. Reducir: E 
2 sen 40  3 cos 40
cos(x  y)  sen x.sen y  cos2 x A) 1,0 B) 1,5 C) 2,0
expresión: D) 2,5 E) 3,0
cos y  cos x
A) 1/2 B) 1 C) 2 15. Determine el valor que toma:
D) 1/3 E) 3
3 cos12  2sen 48
07. Calcule el valor de la siguiente expresión: sen78
tan20  cot 80 A) 1 B) 1/2 C) – 1
1  tan20.cot 80 D) – 1/2 E) 3

A)
3
B)
3
C)
1 16. Cuál es la extensión de la siguiente
4 3 2 expresión: E  2 2sen   cos   1
D) 1 E) 3 A)  1;5 B)  2;4 C)  3;3
08. Si: tan .tanx  3 y tan(  x)  5 , calcule D)  4;2 E)  5;1
tan   tan x
A) 10 B) – 6 C) – 10
D) 6 E) 12

Trigonometría 61
Prof: Breyner Oroya Cruz
17. El máximo valor de la expresión sen(A)  m sen(B).sen(C)
E  sen   ncos   1 es 9, determine el cos(A)  m cos(B).cos(C)
mínimo valor de E. Calcule: tan (A)
A) 0 B) – 2 C) – 5 A) m + 1 B) m – 1 C) m + 2
D) – 7 E) – 10 D) m – 2 E) 1 – m

18. Reducir: sen(  ).sen(  )  cos2  


27. Si:       , calcule el valor de:
A) sen2  B) 2cos2  C) sen2 4
D) cos2  E) cos2   cos2  tan2  tan2 1

cot 2 cot 2.cot 2
19. Determine el valor de:
A) 1/4 B) 1/2 C) 3/4
sen50  2 sen210  cos2 40 D) 1 E) 2
A) 3 B) 1 C) 2
D) 5 E) 4 
28. Si:        cot   1 cot   3
2
20. Hallar el valor de: M  100cos7.cos67
Determine el valor de cot 
A) 33 B) 35 C) 39
D) 41 E) 43 A) 5 B) 1 C) 4
D) 2 E) 3
21. Calcule el valor que debe tomar “k” para
que la igualdad se cumpla: 29. Encuentre el equivalente de:
tan20  tan25  k tan20.tan25  k sen40
 cot 80
A) 2/3 B) 1/2 C) 1 cos30.cos10
D) 3/2 E) 2 A) 1 B) 3 C) 3
22. Calcule: 3 3
D) E)
K  3 tan10  3 tan20  3 tan10.tan20 2 3
A) 1 B) 2 C) 3
D) 3 E) 3 3 
30. Si: x  y  , calcule el valor de:
4
23. Evalúe expresión:
sen(x  y) tan(x  y)
tan20.tan10 
3 tan10  tan20.tan2 10  cos x.cos y cot x.cot y
3
3 3 3
3 A) B) C)
A) tan10 B) tan20 C) 2 3 4
3
D) 1 E) 3
D) 3 E) cot10

24. En un triángulo ABC se cumple que: Problemas Dirigidos


6 tan A  3 tanB  2tanC 01. B 02. C 03. B 04. C 05. A 06. B
Calcule: tan A  2tanB  3 tanC 07. A 08. D 09. D 10. B 11. E 12. A
A) 12 B) 13 C) 14 13. E 14. C 15. B 16. C 17. B 18. D
D) 15 E) 16 19. A 20. E 21. C 22. E 23. B 24. D
25. Calcule el valor de la siguiente expresión: 25. C 26. A 27. D 28. A 29. C 30. B
tan40  tan60  tan80 Problemas Propuestos
tan40.tan80 01. B 02. C 03. C 04. A 05. C 06. A
3 07. B 08. C 09. D 10. B 11. C 12. C
A) 3 B)  3 C) 13. B 14. D 15. A 16. B 17. D 18. D
3
D) 2 E) – 1 19. C 20. C 21. C 22. D 23. C 24. C
25. A 26. A 27. D 28. D 29. E 30. D
26. Las medidas de los ángulos de un triángulo
ABC, verifican las igualdades:

Trigonometría 62
3. Triangulo del Ángulo Doble
IDENTIDADES DEL ANGULO DOBLE
1. Identidades Fundamentales 1  tan2 
2 tan

sen(2)  2.sen().cos() 2
1  tan2 
cos2   sen2 Del triángulo rectángulo:
 2 tan 
cos(2)  2cos2   1 1  tan2 
sen2  cos 2 
 1  tan2  1  tan2 
1  2 sen 
2

2 tan() 4. Identidades Adicionales


tan(2) 
1  tan2 () * sec 2  1  tan2.cot 
* sec 2  1  tan2.tan 
Ejemplos: * 8sen4   3  4cos 2  cos 4
 sen(40) 
* 8cos4   3  4cos 2  cos 4
 cos(20) 
3  cos 4
* sen4   cos4  
 tan(80)  4
5  3cos 4
 sen(6 )  *sen   cos  
6 6

8
 sen(10 ) 
 cos(8 )  IDENTIDADES DEL ANGULO MITAD
 cos(12 ) 

 tan(4 ) 
1. Identidades Fundamentales
Se obtiene a partir de las fórmulas de
degradación.
 tan(9 ) 
 1  cos 
sen 
2 2
PROPIEDADES  1  cos 
cos 
2. Degradacion de Cuadrados 2 2
 1  cos 
tan 
2sen2 ()  1  cos(2) 2 1  cos 
2cos2 ()  1  cos(2) Dónde: (  ) Depende del cuadrante al cual
pertenece “θ/2”
Ejemplos:
Ejemplos:
 2sen2 (10) 
37
 2cos2 (20)   sen( )
2
 2sen2 (3 ) 
45
  cos( )
 2cos2 ( )  2
2
53
 2sen2 (5)   tan( )
2

Trigonometría 63
Prof: Breyner Oroya Cruz
2. Triangulos Notables Ejemplos:
 sen(6) 
 sen(9 ) 
71,5o
10 1  cos(9) 
 cos(2 )
18,5o
 tan(12) 
3

 tan(6 ) 

2 67,5o
2 2
2. Degradación de Cubos
o
22,5
2 2 4 sen3 ()  3 sen()  sen(3)
4cos3 ()  3cos()  cos(3)

63,5o Ejemplos:
5
1
 4 sen3 (9) 
 4 sen3 (6 ) 
o
26,5
2  4 cos3 (7) 
 4 cos3 (10 ) 
3. Formulas Racionalizadas
3. Identidades Especiales del Angulo Triple

tan( )  csc()  cot()
2
sen(3)
  2cos(2)  1
cot( )  csc()  cot() sen()
2
cos(3)
 2cos(2)  1
cos()
Ejemplos:
tan(3) 2cos(2)  1
 tan(20)  
tan() 2cos(2)  1
 cot(10) 
 tan(2)  4 sen().sen(60  ).sen(60  )  sen(3)
 csc(8 )  cot(8)  cos().cos(60  ).cos(60  )  cos(3)
 csc(6 )  cot(6)  tan().tan(60  ).tan(60  )  tan(3)
 tan( )  cot( ) 
 cot(3 )  tan(3)  Ejemplos:
sen(6)
 
IDENTIDADES DEL ANGULO TRIPLE sen(2)
cos(33)
 
1. Formulas Fundamentales del Ángulos Triples cos(11)
tan(15)
 
sen3  3 sen   4sen3  tan(5)
cos3  4cos3   3cos   4sen(10).sen(50).sen(70) 
3 tan   tan3   4cos(20).cos(40).cos(80) 
tan3   tan(12).tan(48).tan(72) 
1  3 tan2 

Trigonometría 64
Prof: Breyner Oroya Cruz
Problema 04: Si: sec 2   sec 2   n
Problemas Resueltos tan  tan 
Calcule: K  
sen2 sen2
Problema 01: Hallar el valor de:
E  4 2 sen(2230').cos(2230') Solución 04: Por las propiedades del ángulo
UNSSAC 2008 – II 2tan x
doble: sen2x 
1  tan2 x
Solución 01: Trabajando con la pregunta, dándole Reemplazando en la pregunta, tenemos:
la forma de un ángulo doble,
tan  tan  1  tan2  1  tan2 
tenemos: K   
sen 2 (22º 30 ')
2 tan  2 tan  2 2
E  2 2  2sen(22º 30')cos(22º 30') 1  tan  1  tan 
2 2

 1  Pero: 1  tan2 x  sec 2 x


E  2 2 .sen (45º )  2 2   2
 2 sec 2   sec 2 
K
2
Problema 02: Reducir: n
Por dato: K 
E  senx.cos5 x  sen5 x.cos x 2

Solución 02: Trabajando con la pregunta, Problema 05: Sabiendo que:


factorizando los términos 1
comunes, tenemos: cos   y 270    360
6
E  sen x.cos x(cos4 x  sen4 x)

1 Determine: cos  
E .2sen x.cos x(cos4 x  sen4 x) 2
2
1
cos 2x 1
Solución 05: Averiguamos el cuadrante al que
E .sen2x.(cos2 x  sen2 x)(cos2 x  sen2 x) pertenece (θ/2).
2

1 1
sen4x
sen 4x 270    360  135   180
E .sen2x.cos 2x  .2sen2x cos 2x  2
2 4 4  
 IIC  cos    ( )
2 2
4 Trabajando con la pregunta, tenemos:
Problema 03: Calcular: tan2 , si: cos  
5 1
1
 1  cos  6
Solución 03: Construyendo un triángulo cos   
2 2 2
rectángulo con el dato, tenemos:
7
5  7
4
cos    3 cos  6 
2 2 12
5 
4
Trabajando con la pregunta, tenemos: Problema 06: Simplificar:
3 3 R  csc 2x  csc 4x  csc 8x  cot 8x
2
2 tan  4
tan2    2 Solución 06: Trabajando con la pregunta,
1  tan2  3
2
1
9
1   16 tenemos:
4
R  csc 2x  csc 4x  csc 8x  cot 8x
3 3 cot 4x
3  16 24
tan2  2  2   R  csc 2x  csc 4x  cot 4x
16  9 7 27 7 cot 2x
16 16 R  csc 2x  cot 2x  cot x
cot x

Trigonometría 65
Prof: Breyner Oroya Cruz
Problema 07: Señale el equivalente de: Solución 11: Trabajando con la pregunta
C  (sec 20º  tan20º )sen35º tenemos:
cos3
R  cos 
Solución 07: Recordemos que: cos 2  2cos2   2
sec 20º  csc70º ; tan20º  cot70º cos3
R  cos 
Reemplazando en la pregunta tenemos: 2cos2   1  2cos2   2
cot 35º
cos3
C  (csc 70º  cot 70º)sen35º R  cos 
4cos2   3
cos35º 4cos3   3cos 
C  sen35º  cos35º R  cos 
sen35º 4cos2   3
cos  (4cos2   3)
sen   1 R  cos   0
Problema 08: Reducir: 4cos2   3
cos 

Solución 08: Trabajando con la pregunta,  3x 


Problema 12: Siendo: tan  45º  3
tenemos:  2 
sen  1  cos  1 cos  Calcule el valor de “sen3x”
  
1  cos  sen  sen  sen 
Solución 12: Trabajando con la pregunta
sen  
 csc   cot   tan   tenemos:
1  cos  2  3x   90º 3x 
tan  45º    3  tan  3
 2   2 
Problema 09: Determine el valor de:
1  cos(90º 3x) 1  sen3x
A  tan2.tan   tan2.tan  tan2.tan  3 9
1  cos(90º 3x) 1  sen3x
Si: tan2.cot   tan2.cot   tan2.cot   7
1  sen3x  9  9sen3x  8  10 sen3x
Solución 09: De la condición: 4
sen3x  
tan2.cot   tan2.cot   tan2.cot   7 5
sec 2  1  sec 2  1  sec 2  1  7
sec 2  sec 2  sec 2  4 Problema 13: Hallar sen 2  1
En la expresión: E
Si: sen  
A  tan2.tan   tan2.tan   tan2.tan  E2  1
A  sec 2  1  sec 2  1  sec 2  1 4 sen  .sen(60º  ).sen(60º )
Dónde: E 
A  sec 2  sec 2  sec 2  3  1 sen .tan 45º  2sen .cos 2
4
Solución 13: Hallando E:
3 sen 3 
sen20º sen 20º
Problema 10: Calcule: M   4 sen .sen(60º ).sen(60º )
4 3 E
sen  tan 45º 2sen  cos 2
Solución 10: Trabajando con la pregunta, sen3

tenemos: sen   2sen  cos 2
sen20º sen3 20º 3 sen20º  4 sen3 20º sen3 sen3
M     1
4 3 12 sen  (1  2cos 2) sen3
sen3(20º ) sen60º 3 Entonces:
M  
12 12 24 E 1 1
sen   2  
E  1 12  1 2
Problema 11: Simplificar la expresión: Nos piden:
cos3 2
R  cos   1 1 5
cos2  2cos2   2 sen 2   1     1   1 
2 4 4

Trigonometría 66
Prof: Breyner Oroya Cruz
A) 0,25 B) 0,50 C) 0,35
Problemas Dirigidos D) 0,75 E) 0,60

x
Tema: Ángulo Doble 08. Si: tan    n , donde x   , entonces
2
01. Calcular el valor de “ sen2 ”
cuál de las siguientes alternativas es la
3 correcta.
Si: sen   cos  
5 1  n2 2n
A) – 22/25 B) 22/25 C) 24/25 A) sen(x)  ; cos(x) 
1  n2 1  n2
D) 28/25 E) 25/28
1  n2 2n
B) sen(x)  ; cos(x) 
02. Evaluar: 1  n2 1  n2
E  cos0.cos20.cos 40.cos60.cos80 2n 1  n2
A) 1 B) 1/2 C) 1/4 C) sen(x)  ; cos(x) 
1 n2
1  n2
D) 1/8 E) 1/16
2n 1  n2
D) sen(x)  ; cos(x) 
cos2 x(1  sen2 x)  cos4 x 1 n2
1  n2
03. Simplificar: T 
1  cos2 2x 1  n2 2n
E) sen(x)  ; cos(x) 
A) 1 B) 2 C) 1/2 1  n2 1  n2
D) – 2 E) – 1/2
4 tan x  4 tan3 x
 09. Halle el equivalente de:
04. Encontrar el valor exacto de: “ sec( ) ” 1  2 tan2 x  tan4 x
8
A) sen2x B) cos2x C) sen4x
A) 42 2 B) 42 2 D) cos4x E) sen x
C) 22 2 D) 22 2
10. Se cumple que:
E) 24 2 1  sen2018  1  sen2018  acosb
o
05. Reducir:  a
Determine el valor de: tan  2b  
 2
K  2  2  2.cos 4x ;0º  x  45º
3 4
A) 2 B) sen x C) 2senx A) B) C) 1
4 3
D) cos x E) 2cos x
3
D) 3 E)
3
06. Si: 3tan2 ()  10tan()  3  0 , halle el
valor de cot(2) Tema: Ángulo Mitad
5 3 3 1
A)  B) C)  11. Si: cos    0    90
3 5 5 6
5 3 
D) E) Calcular: cos  
3 2 2
5 1 1
07. Con la información de la figura, halle A) B) C)
6 6 12
tan2 ()
5 7
D) E)
12 12
5
12 3
12. Si: sen()   y  ;2
13 2
 3
 
Calcule 13 sen  
2

Trigonometría 67
Prof: Breyner Oroya Cruz
A) 1 B) 2 C) 3 19. Al simplificar la expresión:
D) 4 E) 5 sec 50  tan50
F
cot 70
13. Si: sec x  3 , 90  x  180 ;
Se obtiene:
x A) – 2 B) 2 C) 1
Hallar: “ tan( ) ”
2 D) – 1 E) 0 CEPRU 2017–II
2 2
A) 2 B) C)  20. Si:
2 2
csc 2  csc 2  csc 2  cot 2  cot 2  cot 2
D)  2 E) 2
tan3   tan3   tan3 
Hallar: E 
14. El valor de: tan .tan.tan 
A) 1 B) 2 C) 3
(cosa  cosb)2  (sena  senb)2
D) – 3 E) 9
a b
En función de sen   es:
 2  Tema: Angulo Triple
a b a b 21. Si cos(3)  acos()  bcosc () es una
A) 2 sen   B) 4 sen2  
 2   2  identidad, determine: a  b  c
a b a b A) 2 B) 3 C) 4
C) sen   D) sen2   D) – 3 E) – 4
 2   2 
a b   3 
E) 2sen2   22. Si: sen     ; calcule sen  
 2 
6 3 5 2
137 127 117
2 1  sen A)  B)  C) 
15. Al simplificar la expresión , 145 135 125
1  sen  cos 
107 117
5 D) E)
2    se obtiene: 125 125
2
   23. Simplifique la expresión:
A) sen   B) csc   C)  csc  
2 2 2 csc().sen(4)  sec().cos(4)  sen().tan()
  A) cos B) 2cos C) 3cos
D)  tan   E)  cot  
2 2 D) 4cos E) 5cos

24. La expresión es equivalente a:


x
cot(x).tan( ) 3 6sen10  1
16. Reducir: N  2
1
1  cos(x) A) 2cos10 B) 2sen10 C) 2
A) cos x B) csc x C) sen x D) 2cos10 E) 2sen10
D) sec x E) 1
25. Calcule el valor de:
   25 3 cot110(cos60  2cos20)
17. Si: tan( ).cot    y “α” es un ángulo
 2  12 3 2 2
A)  B)  C)
agudo, halle 5cot() 2 3 3
A) 10 B) 11 C) 12 3 5
D) E)
D) 13 E) 14 2 3

18. Halle el valor de la expresión: sen310  cos3 20


2 26. Calcular el valor de:
cot 2 (6 ) cot(45  3 )  cot(45  3 ) sen10  cos20
A) 1 B) 2 C) 3 3 3 4
D) 4 E) 5 A) B) C)
2 4 3

Trigonometría 68
Prof: Breyner Oroya Cruz
2 1 3 2
D) E) D) E)
3 3 2 2

27. Determinar el valor de la expresión: 05. Si sen4  n , calcule cos8(  ) en


sen60 términos de “n”.
2cos 40 
sen20 A) 2n2  1 B) 1 2n2 C) n2  1
A) 0 B) – 1 C) 1 D) 2n  1
2
E) 1  n2
1 3
D) E)
2 2 06. Calcule el valor de la expresión:
2 tan(6730 ')
28. Determine el equivalente de: 1  tan2 (6730 ')
(1  2cos12)(1  2cos36) A) 1 B) 2 1 C) – 1
(1  2cos12)(1  2cos36)
2 1
A) tan12.cot18 B) cot 6.tan54 D) E) 
2 2
C) tan54 D) tan18.cot12
E) tan6.cot 54 07. Del gráfico, calcule el valor de “x”.

29. Simplificar la siguiente expresión:


1  sen(50). 1  sen(10). 1  sen(70) 5

2 3 6
A) B) C)
4 4 4
 4
3 6 
D) E) x
2 2
A) 18 B) 12 C) 9
tan x tan74 D) 6 E) 3
30. Hallar “x”, si: 
tan14 tan44
A) 45° B) 42° C) 39° 08. De la siguiente identidad
D) 21° E) 35°  1  tan2  
2  sen2  sen(M ) , calcule M.
 1  tan  
2

Problemas Propuestos A) 2 B) 1 C) 6
D) 4 E) 8
2 1
01. Si: tan(x)  , calcule sen(2x)  09. Si α es un ángul agudo tal que:
3 13 1
A) 2/13 B) 1 C) 12/13 a 2
D) 2 E) 5/13 tan    
b
Calcule: absen(2)  bcos(2)
02. Si: csc(2x)  1,5 , calcule tan2 (x)  csc 2 (x)
A) a B) 2ab C) 1
A) 3 B) 9 C) 7
D) b E) a + b
D) 6 E) 8

cos(2) 1  sen24  1  sen24


03. Si  3sen() , calcule cot() 10. Reducir: E 
cos()  sen() cos12
A) 1/2 B) 1/4 C) 1 A) 2 B) – 2 C) 2tan12
D) 2 E) 4 D) 2 tan12 E) tan24

1 1
04. Si cos4 x  sen4 x  , calcule cos(4x) 11. Si: cos2   ; 90    180
6 25
2 1 1 
A)  B)  C)  Calcular sen  
3 3 2 2

Trigonometría 69
Prof: Breyner Oroya Cruz
A) 0,5 B) 0,6 C) 0,7
D) 0,8 E) 0,9 2
2
 
A B
1  cos24
1 A) 2 B) 4 C) 8
12. Reducir: H 
2
D) 12 E) 16
2
A) cos6 B) sen6 C) sen3 19. Reducir: E  sec 80  tan80
D) cos3 E) cos12 A) tan40 B) cot 40 C) tan5
D) cot 5 E) sen5
  
13. Señale el valor de: 2sen  
 24  20. Si 2cot(2)  tan()  4 , calcule tan() .
A) 1/2 B) 4 C) 1/8
A) 2 3 B) 2 2 3 D) 2 E) 1/4

C) 2 2 3 D) 2 2 3
 2
21. Si sen    , halle el valor de sen .
E) 2 3 3 3
11 2 17 2 19 2
A) B) C)
5 27 27 27
14. Si sen    , además 180    270 ,
3 8 2 23 2
D) E)
 9 27
calcule tan  
2
A)  6 B)  5 C)  3 22. Calcule: E  3cos 40  4cos3 40
1 1 3
D) 
5
E) 
3 A) B)  C)
5 3 2 2 2
3
D)  E) 1
1 2
15. Si sen   ;   0;90
3
    1
Calcule tan  45   23. Si cos   x  
 2 6  3
1 1 1 Halle el valor de: sen(3x)
A) B) C)
3 2 2 26 23 21
A) B)  C)
1 1 27 27 25
D) E)
5 6 26 23
D)  E)
27 27
  x 
16. Reducir E  cscx  tan    tan x 24. Si 2 tan3   3 tan2   6 tan   1  0
  2 
A) 1 B) – 1 C) 1/2 Calcule el valor de tan3
D) – 1/2 E) 2 A) 0,5 B) 1,0 C) 1,5
D) 2,0 E) 2,5
17. Calcule el equivalente de la siguiente
expresión: (cot   tan )sen2 2.cos2 2sen3 (x)  sen(3 x)
25. Simplifique:
2  cos(2 x)
A) cos 4 B) sen4 C) cos2 4
A) sen(x) B) sen(2x) C) sen(3x)
D) cos8 E) cos2
D) cos(x) E) cos(3x)

18. Del gráfico, calcule cot   , si AB  8
2

Trigonometría 70
Prof: Breyner Oroya Cruz
sen 20º  3 cos3 20º
3
26. Calcular: R 
sen 20º  3 cos 20º Problemas Dirigidos
A) 3/4 B) 4/3 C) 2/3 01. A 02. E 03. C 04. A 05. C 06. D
D) 3/2 E) 3 07. A 08. C 09. C 10. A 11. E 12. B
13. A 14. B 15. C 16. A 17. C 18. D
sen(3x) cos(3x)
27. Si   2k cos(2 x) , calcule 19. C 20. C 21. C 22. C 23. C 24. E
sen(x) cos(x)
25. A 26. B 27. B 28. E 29. C 30. B
k.
A) 0 B) 1 C) 2 Problemas Propuestos
D) 3 E) 4 01. B 02. E 03. D 04. A 05. A 06. C
07. B 08. D 09. D 10. A 11. B 12. B
sen( ) sen() sen() 13. C 14. B 15. B 16. A 17. B 18. E
28. Si   a
sen(3 ) sen(3) sen(3) 19. D 20. E 21. C 22. A 23. B 24. A
tan( ) tan() tan( ) 25. A 26. A 27. C 28. A 29. A 30. C
Exprese   , en
tan(3 ) tan(3) tan(3)
términos de “a”.
A) 3  2a B) 3  a C) 2  3a
D) 3  2a E) 2  3a

29. Simplifique la siguiente expresión:


1  sen50. 1  sen10. 1  cos 20
6 3 2
A) B) C)
4 2 3
6
D) E) 3
2

30. Calcular el valor de la expresión:


E  tan20.tan40.tan60.tan80
A) 1 B) 2 C) 3
D) 3 E) 2

Trigonometría 71
Identidades de Transformacion 1. Factorizaciones Condicionadas
Trigonometrica de Suma o Diferencia a A BC  
Si:
Producto A B C
* sen A  senB  senC  4cos .cos .cos
2 2 2
 A B  A B A B C
sen A  senB  2sen   .cos   * cos A  cosB  cosC  4sen .sen .sen  1
 2   2  2 2 2
 A B
sen A  senB  2sen   .cos 
 A B
 Si: A  B  C  2
 2   2  A B C
* sen A  senB  senC  4 sen .sen .sen
 A B  A B 2 2 2
cos A  cosB  2cos   .cos  
 2   2  A B C
* cos A  cosB  cosC  4cos .cos .cos  1
A B  A B 2 2 2
cos A  cosB  2sen   .sen  
 2   2 
2. Productos Trigonométricos Notables
Ejemplos:
 sen(16)  sen(14)   2 n 2n  1
sen( ).sen( )......sen( )
 sen(19 )  sen(5 )  2n  1 2n  1 2n  1 2n
 sen(20)  sen(30)   2 n 1
cos( ).cos( )......cos( )
 sen(12 )  sen(2 )  2n  1 2n  1 2n  1 2n
 2 n
 cos(7)  cos(5)  tan( ).tan( )......tan( )  2n  1
2n  1 2n  1 2n  1
 cos(6 )  cos(4 ) 
 cos(50)  cos(20) 
 cos(3 )  cos( ) 
Ejemplos:
 2 3
 sen( ).sen( ).sen( ) 
7 7 7
Identidades de Transformacion  2 3 4
 cos( ).cos( ).cos( ).cos( ) 
Trigonometrica de Producto en Suma o 9 9 9 9
 2
Diferencia  tan( ).tan( ) 
5 5

2sen A.cosB  sen(A  B)  sen(A  B)


3. Casos Especiales
2cos A.cosB  cos(A  B)  cos(A  B)
 3  (2n  1)  1
2sen A.senB  cos(A  B)  cos(A  B) cos( )  cos( )  ...  cos  
2n  1 2n  1  2n  1  2
Ejemplos: 2
)  cos(
4  2n 
)  ...  cos 
1
cos( 
 2sen(8).cos(2)  2n  1 2n  1  2n  1  2
 2sen(8 ).cos(4) 
 2sen(30).cos(50)  Ejemplos:
 2cos(25).cos(15)   3 5
 cos( )  cos( )  cos( ) 
 2cos(2 ).cos(4)  7 7 7
2 4 6 8
 2sen(7).sen(5)   cos( ).cos( ).cos( ).cos( ) 
9 9 9 9
 2sen(40).sen(20) 

Trigonometría 72
Prof: Breyner Oroya Cruz
C  sen(40º  x)  sen(34º  x)
Problemas Resueltos  40º  x  34º  x 
C  2sen 
 40º  x  34º  x 
 .cos  
 2   2 
Problema 01: Simplificar: 3
sen80º  sen40º C  2sen37º.cos(3º  x)  2. .cos(3º  x)
R 5
cos80º  cos 40º 6
C  cos(3º  x)
5
Solución 01: Transformando a producto, máx 1

tenemos: 6
Cmáx 
sen80º  sen 40º 5
R
cos80º  cos 40º
 80º  40º   80º  40º  Problema 05: Reducir:
2sen   .cos  
 2   2  2sen 4x.cos3x  sen7x
R R
 80º  40º   80º  40º  2sen5x.cos 4x  sen9x
2cos   .cos  
 2   2 
2sen60º.cos 20º Solución 05: Transformando de producto a
R  tan60º  3 suma, tenemos:
2cos 60º.cos 20º
2sen 4x.cos3x  sen7x
R
Problema 02: Simplificar: 2sen5x.cos 4x  sen9x
sen x  sen3x  sen5x sen(4x  3x)  sen(4x  3x)  sen7x
L R
cos x  cos3x  cos5x sen(5x  4x)  sen(5x  4x)  sen9x
sen x
Solución 02: Agrupando convenientemente en R 1
la expresión: sen x

sen x  sen3x  sen5x


L Problema 06: Simplificar:
cos x  cos3x  cos5x L  sen4x.cosx  senx.cos6x  sen2x.cos5x
2 sen3x.cos 2x  sen3x
L
2cos3x.cos 2x  cos3x Solución 06: Multiplicando por 2 a la expresión
sen3x (2cos 2x  1) y luego transformamos.
L  2L  2sen 4x.cos x  2sen x.cos 6x 
cos3x (2cos 2x  1)
sen3x 2sen2x.cos5x
L  tan3x 2L  sen5x  sen3x  sen7x  sen5x 
cos3x
sen7x  sen3x
Problema 03: Transformar a producto:
2L  2sen7x  L  sen7x
L  cos2x  cos6x  cos10x  cos14x

Solución 03: Agrupando convenientemente: Problema 07: Transforma a suma o diferencia:


C  4cos7x.cos3x.cos x
L  cos14x  cos 2x  cos10x  cos 6x
L  2cos8x.cos 6x  2cos8x.cos 2x Solución 07: Ordenando:
L  2cos8x(cos 6x  cos 2x) C  2(2cos7x.cos3x)cos x
L  2cos8x(2cos 4x.cos 2x) C  2(cos10x  cos 4x)cos x
L  4cos8x.cos 4x.cos 2x C  2cos10x.cos x  2cos 4x.cos x
C  cos11x  cos9x  cos5x  cos3x
Problema 04: Señale el valor máximo de:
C  sen(40º x)  sen(34º x) Problema 08: Indique en qué tipo de triangulo
ABC, se cumple:
Solución 04: Transformando a producto, senA.senB  cosC
tenemos:

Trigonometría 73
Prof: Breyner Oroya Cruz
Solución 08: Multiplicando por 2 a la condición y Problema 11: Calcular el valor de:
transformando: E  (1  cos 40º )(1  cos80º )(1  cos 20º )
2sen A.senB  2cosC
cos(A  B)  cos(A  B)  2cosC Solución 11: Trabajando con la pregunta,
Pero: tenemos:
A  B  C  180º  A  B  180º C E  (1  cos 40º )(1  cos80º )(1  cos 20º )
Reemplazando tenemos:
cos(A  B)  cos(180º C)  2cosC E  (2sen2 20º )(2sen2 40º )(2cos 2 10º )
cos(A  B)  (  cosC)  2cosC E  2 2 sen20º sen 40º cos10º
cos(A  B)  cosC  2cosC E  2 2 sen20º sen 40º sen80º
cos(A  B)  cosC E  2 2 sen20º sen(60º 40º )sen(60  20º )
Igualando los ángulos:
sen3(20º ) 2
A  B  C  A  B  C  A  90º E2 2  sen60º
4 2
Es un triángulo rectángulo.
2 3 6
E  
1 2 2 4
Problema 09: Si: tan  tan7  ,
2
Halle el valor de: E  sec 8 cos6  1 Problema 12: Reducir:
L  sen2º  sen4º  sen6º ...  sen178º
Solución 09: Trabajando con la condición,
tenemos: Solución 12: Notamos que los ángulos se hallan
1 sen  sen7 1 en progresión aritmética, así que
tan  tan7    podemos usar la fórmula de
2 cos  cos7 2
sumatorias de senos, para ello
2sen  sen7 1 cos 6  cos8 1
   debemos reconocer primero:
2cos  cos7 2 cos8  cos 6 2 L  sen2º  sen 4º  sen6º ...  sen178º
2cos 6  2cos8  cos8  cos 6
cos 6  3cos8 P  2º r  2º
Nos piden: U  178º 178  2
n  1  89
E  sec 8 cos 6  1 2
E  sec 8 (3 cos8 )  1 Luego:
E4  nr 
sen  
L  2  .sen  P  U 
 2 
Problema 10: Simplificar la expresión: r  
sen  
2cos .cos   cos(   ) 2
M  tan2 (   )
2sen .sen   cos(   )  89  2º 
sen  
L  2  .sen  2º 178º 
Solución 10: Trabajando con la pregunta,  
 2º   2 
sen  
tenemos: 2
2cos .cos   cos(   )
M  tan2 (   )
1
sen89º cos1º
2sen .sen   cos(  ) L .sen90º 
sen1º sen1º
cos(   )  cos(  )  cos(   )
M L  cot1º
cos(   )  cos(  )  cos(   )
 tan2 (   )
cos(   )
M  tan2 (   )
cos(   )
M  1  tan2 (   )  sec 2 (  )

Trigonometría 74
Prof: Breyner Oroya Cruz
B
Problemas Dirigidos
Tema: Transformaciones de Suma a Producto
sen9  sen5 60
01. La expresión: , es igual a: A C
cos9  cos5
A) 2 B) 3 C) 1
A) tan2 B) tan C) tan3
D) cot  E) cot 2 1 1
D) E)
2 2
02. Simplificar: R  cos100  cos 40  cos20
A) 1 B) 2 C) – 1 08. Si: sen(8)  4sen(2)
D) 0 E) – 2 Determine: 5tan(3)  3tan(5)
A) – 5 B) – 3 C) 0
1  cos14  cos 42  cos56
03. Si: 1 D) 1 E) 5
K cos A cosB cosC
Calcular: K  A  B  C sen(5) 3 tan(a )
09. Si:  , además: 5
A) 62 B) 49 C) 58 sen(3) 2 tan(b )
D) 64 E) 60
Determine: a  b
5 A) 3 B) 5 C) 7
04. Si: tan2 (2)  D) 9 E) 11
8
sen(6 )  sen(2 ) 10. Calcular el valor de la siguiente expresión:
Calcular:
sen(6 )  sen(2 ) 2sen(  50)
, si   0;  y
5 3 7 cos 40
A) B) C)
16 4 8 (sen70  2sen50)sen   cos .cos70
3 1 A) 4cos50 B) 3sen40 C) 2sen50
D) E)
16 4 D) 2cos50 E) 2sen40

05. Si:     45 y Tema: Transformaciones de Producto a Suma


E  (sen   sen)2  (cos   cos )2 11. Simplificar la expresión
3cos10  6sen20.sen10
Hallar el valor de E  2
3 3 3 2
A) – 2 B) 2 C)  2 A) B) C)
2 2 2
D) 2 E) 1  2
D) 2 E) 3
06. En un triángulo ABC, transformar a
producto: sen(A)  sen(B)  sen(C) 12. Halle el valor de la expresión:
2sen40.cos20  sen20
A) 4sen(A).sen(B).sen(C) E
2cos35.cos10  cos25
A B C
B) 4sen( ).cos( ).cos( ) 2 3 6
2 2 2 A) B) C)
C) 4cos A.cosB.cosC 4 4 2
A B C 6 2
D) 4cos( ).cos( ).cos( ) D) E)
2 2 2 3 6
A B C
E) 2sen( ).sen( ).sen( ) 13. Reducir:
2 2 2 E  sen(3x).cos(4x)  sen(3x).cos(2x)
 sen(6x).cos(x)
07. Con los datos de la figura mostrada,
determine el valor de la expresión: A) sen(x) B) sen(x) C) 0
tan A  sec A.senB D) cos(x) E)  cos(x)
2cosC  sec A.cosB

Trigonometría 75
Prof: Breyner Oroya Cruz
14. Hallar cot(x) , si: cos(4 x) cos(2 x) cos(2 x)
A) B) C)
sen(2x).sen(5x)  sen(x).cos(4x)  cos(x).cos(6x) cos(x) cos(x) sen(x)
A) 1 B) 1/2 C) 1/4 cos(4 x) sen(4 x)
D) 4 E) 2 D) E)
sen(x) cos(x)
15. En un triángulo ABC, reducir:
02. Si: 2  15 , calcule el valor de:
sen2 (A)  sen2 (B)
E , si: A  B  45 (sen   sen3)(2cos5  2cos3)
sen(2A)  sen(2B)
3 5 3
tan(C) tan(C) 1 A)  B)  C) 
A) B)  C) 2 7 3
2 2 2
D) tan(C) E)  tan(C) 3 5
D) E)
6 7
16. Simplificar la expresión:
03. Reducir: cos10º  cos70º  cos50
3sen20  2sen2 20
 cos60 A) 1 B) 2 C) 0
2sen70 D) – 1 E) – 2
A) cos10 B) cos20 C) sen10
D) sen20 E) cos 40 3 
04. Si: 0   ; al simplificar la
2 2
sen(a) cos(a) 3
17. Si: 
sen(b) cos(b)
expresión sen(3)  sen(2) sec( 
),
2
Halle sen(a)  sen(a 2b) se obtiene Asen(B  C) ; entonces el
A) – 1 B) 0 C) 1 valor de (A  2B  3C) es:
D) – 2 E) 2 A) 8 B) 10 C) 12
D) 14 E) 16
18. Siendo “α” un ángulo agudo, calcule “α” si:
cot(6)  csc(8)  cot(16) sen2  sen3  sen4
A) 5° B) 6° C) 7° 05. Si:  mtan(n )
cos2  cos3  cos 4
D) 8° E) 9° Determine m  n
A) 1 B) 2 C) 3
19. Si:   85;135 . Calcule los valores de la D) 4 E) 5
expresión: E  sen(  35).cos(  5)
06. El valor máximo de la función f, definida
 1  1   1  por:
A) 0;  B)   ;0  C)   ;0 
 2  2   4  f(x)  sen(x 80)  sen(x 40) , es:
 1 1  1 1 2 3
D)   ;  E)   ;  A) B) C) 1
 2 2  4 2 2 3
D) 2 E) 3 CEPRU 2015–I
20. Simplificar la siguiente expresión:
sen2 20  sen2 40  sen2 60  sen2 80 07. Si se cumple:
A)
3
B)
3
C)
9 sen40  sen20  a  cos40  cos20  b
4 2 4 a2  b2
9 15 Calcule:
D) E) a2  b2
2 4
3 1 1
A)  B)  C)
2 2 2
Problemas Propuestos 3
D) E) – 2
2
sen(5x)  sen(3x)
01. Reducir: M 
sen(2x)

Trigonometría 76
Prof: Breyner Oroya Cruz
3 16. Si sen136  a , simplifique la expresión
08. Si:       , transforme a producto la
2 2sen34.sen54  cos20  1 en términos
expresión: sen2 ()  sen2 ()  cos2 () de “a”.
A) 4sen()sen()sen() A) a 2 B) 1  a 2 C) a2  2
B) 2sen()sen()sen() D) 2a2  2 E) 2  a 2
C) 2cos()cos()cos()
17. Si A  B  C   , entonces el valor de:
D) 4cos()cos()cos()
A  B C  B  CA 
E) 2sen()sen()cos() sen   cos    cos   sen  
2  2  2  2 
09. Si sen(7 x)  2sen(5 x) C  A B 
 cos   sen  
2 tan(6 x)  tan(x) 2  2 
Determine:
tan(6x)  2 tan(x) es:
A) 3 B) 5 C) 7 A) cos(A) B) cos(B) C)  cos(C)
D) 9 E) 11 D)  cos(A) E) cos(C) CEPRU 2016–II

cos(5) 2 1 18. Si 90  x  180 , calcule el valor de “x” en


10. Si: 
cos() 2 1 la siguiente igualdad:
Determine: cot 2 (3)  2tan2 (2)  1 sen40.senx  cos40.cosx  2cos20.cosx
A) 1 B) 2 C) 3 A) 110° B) 120° C) 130°
D) 2 2 E) 3 2 D) 140° E) 150°

11. Calcule: (cos13  sen4)2


19. Al simplificar 1 , se
sen13º  cos41º 2cos283º.cos36º 1  sen17
A) 0 B) 0,5 C) 1,0 obtiene:
D) 1,5 E) 2,0 A) sen9 B) cos9 C) sen7
12. Reducir: D) cos7 E) sen5 CEPRU 2016–I
R  cos5.cos2  sen6.sen   cos4.cos 
A) cos() B) cos(2) C)  cos() 20. Si tan(5)  (2  3)cot(2)
D)  cos(2) E) 0 3 sec(3)
Calcule: 2
sec(7)
1
13. Calcule el valor de:  2sen70 A) 3 3 B) 2 3 C)  3
2sen10
A) 1/2 B) 1 C) 2 D) 3 E) 1
D) 2 E) 3 UNI 2010–II

14. Calcule el valor de:


2sen2 20  2 sen85
cos25(cos85  cos35)
Problemas Dirigidos
A) 1/2 B) 1 C) 2
01. A 02. D 03. E 04. D 05. D
D) 2 E) 2 2
06. D 07. B 08. C 09. B 10. A
15. Reduzca la siguiente expresión: 11. B 12. C 13. C 14. A 15. C
3 16. B 17. B 18. A 19. C 20. C
sen 6.sen18.sen84 
2 Problemas Propuestos
cos6 cos6 01. A 02. A 03. C 04. A 05. D
A) cos 6 B) C)
2 4 06. E 07. E 08. B 09. C 10. A
cos6 sen 4 11. A 12. E 13. B 14. C 15. C
D) E)
8 4 16. D 17. E 18. B 19. A 20. E

Trigonometría 77
1. Ley de Senos B
En todo triangulo los lados son
proporcionales a los senos de los ángulos a
opuestos y la constante de proporcionalidad es c
el diámetro de la circunferencia a dicho
triangulo. A C
b

C  A B  A C
a R tan   tan  
ab  2  ac   2 

B ab  A B ac  A C
b tan   tan  
 2   2 
c
B  A 
tan  
A ba  2 

a  2R sen A ba B  A 
a b c tan  
   2R  b  2R senB  2 
sen A senB senC
c  2R senC
4. Ley de las Proyecciones
2. Ley de Cosenos En todo triangulo cada lado es igual a la
En todo triangulo, el cuadrado de un lado suma de las proyecciones de los otros dos
es igual a la suma de los cuadrados de los lados sobre él.
otros dos menos el doble producto de estos dos B
lados por el coseno del ángulo que forman.
a
c
B

a A b C
c

a  b cosC  c cosB
A b C
b  a cosC  c cos A
c  a cosB  b cos A
b2  c 2  a2
a2  b2  c 2  2bc.cos A  cos A 
2bc
5. Área de un Triangulo
a2  c 2  b2
b  a  c  2ac.cosB  cosB 
2 2 2

2ac
a2  b2  c 2
c  a  b  2ab.cosC  cosC 
2 2 2 C
a R
2ab
B
S
3. Ley de Tangentes b
En todo triangulo la suma de dos lados es a c
su diferencia como la tangente de la
semisuma de los ángulos opuestos a dichos A
lados es proporcional a la tangente de la
semidiferencia de los mismos ángulo.

Trigonometría 78
Prof: Breyner Oroya Cruz
1 1 1
S ac.senB;S  ab.senC;S  bc.sen A
2 2 2
abc elevación
S  p.r depresión
Horizontal
4R
S  p(p  a)(p  b)(p  c)
S  2R2 sen A.senB.senC
7. Ángulos Horizontales
Dónde: Son aquellos que se forman en el plano
R : Circunradio horizontal y que para el cálculo de rumbos y
r : Inradio direcciones en problemas trigonométricos se
p : Semiperimetro utiliza la Rosa Náutica.
Nota:
Rosa Náutica
2p  a  b  c perimetro Es aquel instrumentó que considera al ángulo
trigonométrico de una vuelta dividiendo en 32
abc
p semiperimetro partes iguales, siendo la medida de cada una
2 de estas partes de 11º 15’.

Semiángulos Internos de un Triangulo

A (p  b)(p  c) A p(p  a)
sen  cos 
2 bc 2 bc
B (p  a)(p  c) B p(p  b)
sen  cos 
2 ac 2 ac
C (p  a)(p  b) C p(p  c)
sen  cos 
2 ab 2 ab

A (p  b)(p  c)
tan 
2 p(p  a)
B (p  a)(p  c) Rumbo o dirección
tan 
2 p(p  b) Es la desviación angular que sufre la rosa
náutica con respecto a dos direcciones
C (p  a)(p  b)
tan   principales, al ubicar un punto.
2 p(p  c)
N
6. Ángulos Verticales
Son aquellos ángulos que se forman en el Q
O  E
plano vertical entre una línea horizontal y una 
línea de mira (visual). N d
 S
Ángulos de Elevación: Es aquel, cuya medida se 
O E
P
realiza entre la línea visual y la línea horizontal;
pero cuando el objeto se encuentra por encima
del horizontal. S
El punto Q se encuentra en la dirección
Ángulos de Depresión: Es aquel, cuya medida se E α N y a “d” metros del punto P.
realiza entre la línea visual y la línea horizontal; El punto P se encuentra en la dirección
pero cuando el objeto se encuentra por debajo S θ O y a “d” metros del punto Q.
del horizontal.

Trigonometría 79
Prof: Breyner Oroya Cruz
 A  C
tan  
Problemas Resueltos ac
ac   2  (I)
  tan  A  C 
Problema 01: En un triángulo ABC, a = 3b,  2 
 
sen A  senB Del enunciado tenemos:
Calcular: L 
sen A  2senB A  B  C  180º  A  C  180º B
AC B (II)
Solución 01: De la ley de senos tenemos:   90º 
2 2
a b c  De: (I) y (II) tenemos:
   2R  sen A  3b
sen A senB senC  2R ac  A C  A C  B
  a  c   tan  2   tan  2   tan  90º  2 
3b

b

c 
 2R senB 
b        
sen A senB senC 
 2R
ac  A C B
Reemplazando en la pregunta tenemos:    tan    cot  
ac  2  2
3b b 4b

sen A  senB 2R 2R 2R
L   Problema 04: En un triángulo ABC, Simplificar:
sen A  2senB 3b  b  b
 2  S  (a  b)cosC  (a  c)cosB  (b  c)cosA
2R  2R  2R
L4
Solución 04: Trabajando con la pregunta,
tenemos:
Problema 02: En un triángulo ABC, a = 3; b = 4 S  acosC  bcosC  acosB  c cosB
y Ĉ  60º , calcular su perímetro.  bcos A  c cos A
Agrupando convenientemente, tenemos:
Solución 02: Haciendo una representación,
gráfica, tenemos: S  acosC  c cos A  bcosC  c cosB 
b a

 acosB  bcos A  a  b  c  p
B c

a3 Problema 05: En un triángulo ABC, Simplificar:


c
S  (a  b)cosC  (a  c)cosB  (b  c)cosA
60º
A b4 C
Solución 05: Trabajando con la pregunta,
tenemos:
Utilizando la ley de cosenos tenemos: S  acosC  bcosC  acosB  c cosB
c 2  32  42  2(3)(4)(cos 60º )  bcos A  c cos A
c  9  16  12
2 Agrupando convenientemente, tenemos:
S  a cosC  c cos A  b cosC  c cosB 
c 2  13  c  13
b a
El perímetro del triángulo está dado por:  a cosB  b cos A
2p  a  b  c  3  4  13 c

S  a  b  c  2p
2p  7  13

Problema 03: Dado un triángulo ABC, reducir la Problema 06: En un triángulo ABC de lados a, b
siguiente expresión: y c respectivamente. Calcule:
ac A C C B
E  b.sen2    c.sen2  
   tan   2 2
ac  2 

Solución 03: Sabemos que: Solución 06: Recordemos que:


 A  (p  b)(p  c)  B  (p  a)(p  c)
sen2    ;sen2   
2 bc 2 ac
Reemplazando en la pregunta, tenemos:

Trigonometría 80
Prof: Breyner Oroya Cruz
(p  a)(p  b) (p  a)(p  c) Por lo tanto el menor ángulo formado por ellos
E  b c mide 30º.
ab ac
(p  a)(p  b) (p  a)(p  c)
E  Problema 09: Un observador se encuentra
a a ubicado al sur de una torre y
(p  a) (p  a)
E
a
2p  b  c   a a visualiza el extremo superior de
aquella con un ángulo de
E pa elevación α. Asimismo otro
observador se encuentra en
dirección O α N respecto a la
Problema 07: Un nadador se dirige hacia un faro anterior y al Oeste de la torre
y lo observa con un ángulo de visualizando el extremo superior
elevación de 30º; al avanzar 10 m, de sesta con un ángulo de
el ángulo de elevación se duplica. elevación β. Calcule cot β en
Halla la altura del faro. términos de una razón
trigonométrica de α.
Solución 07: Haciendo una representación
gráfica del problema, tenemos: Solución 09: De acuerdo con el enunciado,
tenemos:
Sea QR  a
30º T
10
H
O  N
30º 60º  a
10
R E
O 
H 3 
sen60º    H  5 3m
10 2 P E
S
Problema 08: Dos personas se ubican una frente En el QRP : PR  QR.tan   a tan 
a otra en línea Este – Oeste. Si
estas se desplazan en En el PRT : TR  PR.tan   a tan2 
rectilíneamente en las direcciones QR a
En el QRT : cot   
N 70º E y O 10º N respectivamente TR a tan2 
hasta encontrarse, determine el Por lo tanto:
menor ángulo formado por las
direcciones de sus movimientos. cot   cot 2 

Solución 08: De acuerdo con el enunciado, las


personas ubicadas en A y B se
desplazan rectilíneamente hasta
encontrarse en C. Se observa que
sus direcciones forman los ángulos
de medida 150º y 30º.

O10ºN N70ºE

C
N 30º N
70º 70º 80º
10º
O E
A B

S S

Trigonometría 81
Prof: Breyner Oroya Cruz
07. En la figura adjunta se tiene:
Problemas Dirigidos BM: es mediana del triángulo ABC.
AN: mediana del triángulo ABM.
Tema: Ley de Senos Determine: cos 
01. Calcule el lado “b” en un triángulo ABC, si: C
a  4 ; Â  60 y B̂  45 60
2 3 2 6 4 3 M
A) B) C)
5 3 3
4 6 N 
D) E) 4 6
3
B A
02. En un triángulo ABC de lados AB  c , 2 7 7 7
A) B) C)
BC  a , AC  b , simplificar la expresión: 7 7 2
9sen(A) 5sen(B) 4sen(C) 17 3 7
  D) E)
a b c 17 7
A) 4 B) 2 C) 3
D) 1 E) 0 1
08. En un triángulo ABC, si: cot(B)  ,
3
03. En un triángulo ABC de lados AB  c ,
BC  a , AC  b , se cumple: b2  a2  c 2
Calcular:
bsen(B C)  acos(A C) y C  A  25 (c.cosA  a.cosC)senC c.senB

B  55  A  c 3 a 3 b 3
Calcule: tan    tan  A)  B)  C) 
 3 3 3
2  2 
D) c 3 E) a 3
A) 2 B) 2 2 C) 3 2
D) 4 2 E) 5 2 09. En un triángulo ABC de lados AB  c ,
BC  a , AC  b , se cumple:
04. En un triángulo ABC de lados AB  c ,
cos(A) cos(B) cos(C) a 2  b 2  c 2
BC  a , AC  b .   
a b c R3
Si se cumple: a2sen(B).sen(C)  bc
Siendo R circunradio.
Calcular la medida del ángulo A. Calcule: sen(A).sen(B).sen(C)
A) 90° B) 30° C) 60°
A) 1/16 B) 1/14 C) 1/12
D) 120° E) 45°
D) 1/10 E) 1/8
05. En la figura mostrada AB  DC calcule el 10. En un triángulo ABC, si:
valor de α.
W  a2 cos(2B)  2bc cos(A)  b2cos(2A)  b2
B
W
 Calcular el valor de
b2  c 2
A) 1 B) 3 C) 2
D) 5 E) 4

A
8 2
C Tema: Ley de Proyecciones
D 11. En un triángulo ABC, reducir:
A) 5° B) 10° C) 15°
acosB  bcos A
D) 20° E) 30° E
senC
Tema: Ley de Cosenos Considere R: circunradio
A) R B) 2R C) R/2
06. En un triángulo ABC, simplificar:
D) 4R E) R/4
(a b)2 (1  cosC)  (a b)2 (1  cosC) 12. En un triángulo ABC, simplificar:
E
c2 E  a(b2  c 2 )cos A  b(a2  c 2 )cosB
A) 1 B) 2 C) 1/2
D) 3 E) 1/3  c(a2  b2 )cosC

Trigonometría 82
Prof: Breyner Oroya Cruz
abc abc 18. Si un triángulo ABC:
A) B) 3abc C)
3 2 ac B  A C
 4 tan   .cot  
D) 2 E) 1 ac 2  2 
13. En un triángulo ABC, simplificar: tan(A)  tan(B)  tan(C)
Calcular: S 
 a  c.cos(B)  tan(A).tan(C)
S  sec(C)
 c  a.cos(B)  A) 3/4 B) 4/3 C) 7/6
D) 6/7 E) 2/5
A) sen(A) B) cos(A) C) tan(A)
D) cot(A) E) sec(A) 19. Si en un triángulo ABC con lados a, b y c
A
14. Con los datos de la figura, calcular el valor respectivamente, se tiene: tan    1 y
2
cos(A) cos(B) cos(C)
de:  
5 6 7 B 3 ab
tan    , determine:
C 2 4 ab
A) 50 B) 16 C) 49
D) 9 E) 25
6 5
20. En un triángulo ABC se cumple que:
 A B
A B tan    6 y a  3b
7  2 
A) 10/13 B) 1/2 C) 11/43
Determine: 19tan(B)  17tan(A)
D) 11/42 E) 10/21
A) 9 B) 10 C) 11
15. En un triángulo ABC, si AB  c , BC  a , D) 12 E) 13
AC  b , simplificar: Tema: Ángulos Verticales
C B 21. Desde el puesto de vigía de un barco que
bcos    c cos2  
2

E  2 2 tiene 48 m de altura se observa que el


(b c)cos A  (a c)cosB  (a  b)cosC ángulo de depresión de un bote es de 30º.
Calcular la distancia a la que está el barco.
A) 0,5 B) 1,0 C) 1,5
D) 2,0 E) 2,5 A) 48 B) 48 3 C) 6 3
D) 24 E) 12
Tema: Ley de Tangentes
22. Desde el pie de un poste se observa la
16. En un triángulo ABC ( AB  c , BC  a , parte alta de una torre con un ángulo de
AC  b ), el ángulo C mide 60° y los lados: elevación de 45°, el mismo punto es
a2 3 2 y b2 3  2 observado desde la parte más alta de poste
con un ángulo de elevación θ, tal que
 A B sen()  0,6 . Calcular la longitud del poste,
Determine tan  
 2  si la distancia entre el poste y la torre es 80
1 2 3 metros.
A) B) C) A) 20 m B) 30 m C) 10 m
2 2 3
D) 40 m E) 50 m
D) 2 E) 1
23. Un bombero agobiado por su situación
17. Con la información que se da en la figura, amorosa decide suicidarse tirándose de lo
evaluar la expresión: 11tan(  ) alto de su casa; pero el suicida se percata,
con un ángulo de depresión de 37°, que un
60 policía lo está observando. Si el policía
2 3 observa la alto de la casa con un ángulo de
elevación 30° y el policía se encuentra a 10
  m, calcule la estatura del suicida. Considere
( 3  1,74 ).
A) 2 3 B) 3 3 C) 4 3 A) 2,10 m B) 1,90 m C) 1,80 m
D) 5 3 E) 6 3 D) 1,70 m E) 1,65 m

Trigonometría 83
Prof: Breyner Oroya Cruz
24. Desde la parte superior e inferior del 3 km/h y 4 km/h respectivamente. Si al
segundo piso de un edificio de 4 pisos cabo de dos horas la recta que une ambos
iguales, se observa una piedra en el suelo barcos corta a la dirección Este del puerto
(a 9 m del pie del edificio) con ángulos de en “M”. Calcule “α” para que la distancia del
depresión α y θ, respectivamente. Desde la puerto al punto M sea mínima.
parte superior del edificio, la depresión A) 37° B) 45° C) 53°
angular para la piedra es β, calcule la altura D) 60° E) 74°
en metros de dicho edificio si 30. Tres ingenieros eléctricos ubicados al este,
7 sur y oeste de una torre de alta tensión
tan   tan   tan   .
4 observan la parte superior de la torre con
A) 60 B) 63 C) 72 ángulos de elevación α, β y θ
D) 76 E) 84 respectivamente; de tal manera que el
ingeniero ubicado al sur observa a sus
25. Un alumno de altura h observa un punto P, compañeros bajo un ángulo recto. Si α y β
sobre una colina en línea horizontal y se son complementarios, y tan()  0,125 ;
sabe que la distancia que hay desde el pie
del alumno hacia el punto P es 2h. Luego, entonces, al calcular tan() se obtiene:
el alumno se ubica en el punto P y observa A) 1/2 B) 3/4 C) 1
su ubicación inicial con un ángulo de D) 3/2 E) 2
depresión β; calcule csc  .
Problemas Propuestos
7 7 7
A) B) C) 01. En un triángulo ABC; m B  60 ; c  4 y
2 3 4
D) 7 E) 2 7 7sen(C)  2 ; calcula “b”
A) 3 3 B) 4 3 C) 5 3
Tema: Ángulos Horizontal
D) 7 3 E) 3,5 3
26. Un pueblo A se encuentra a 100km de un
punto P en la dirección SE (Sur-Este) y otro 02. En un triángulo ABC simplificar:
pueblo se encuentra a 60km de P en la b  a sen A  senC
dirección NE (Nor-Este). Hallar la distancia E 
b  c senB  senC
entre Ay B (en km).
A) b  c B) a  c C) a  c
A) 20 34 B) 15 34 C) 20
D) 1 E) 2
D) 20 35 E) 19 34
03. En un triángulo ABC, simplificar:
27. Andy se encuentra al Este de Viviana, si a.sen(B)  b.sen(A) b.sen(C)  c.sen(B)
E  
1 sen(A) sen(B)
Viviana se desplaza en dirección N NE y
4 c.sen(A)  a.sen(C)
Andy en la dirección NO, se encuentran en
sen(C)
el punto P. Calcule la medida del ángulo
(en rad) que forman las trayectorias (p: semiperimetro)
recorridas por dichas personas. A) p B) 2p C) 3p
  21 D) 4p E) 8p
A) B) C)
4 16 80 04. En un triángulo ABC se cumple que:
5 29 abc  16 y 4senA.senB.senC 1
D) E)
16 144 Calcular el circunradio de dicho triangulo.
A) 1 B) 2 C) 3
28. Un móvil parte de la dirección N53°O, D) 4 E) 5
recorriendo 60 km y luego cambia a la
dirección S30°O, quedando al oeste del 05. En la figura:
punto de partida. ¿Cuánto recorrió en esta A
última dirección (en km)? x
A) 24 B) 30 C) 40
D) 24 3 E) 50
30 15
29. Dos embarcaciones parten de un puerto B C
hacia el NαE y S(90°-α)E a velocidades de M

Trigonometría 84
Prof: Breyner Oroya Cruz
Si AM es mediana, la medida del ángulo D) 4R E) R/2
2x, es: 14. En un triángulo ABC, si cosC  0,1 . Calcule
A) 50° B) 30° C) 20°
D) 60° E) 40° UNSAAC 2016–I c cos A  c cosB
el valor de: E  1 
ab
06. En un triángulo ABC: a3; c2 y A) 0,1 B) 0,2 C) 0,3
B  60 ; calcular “b” D) 0,4 E) 0,5
A) 3 B) 2 C) 5 15. Dado el triángulo ABC, halle el equivalente
D) 6 E) 7 de:
a  c cosB b  acosC c  bcosA
07. En un triángulo de lados: (x) ; (x  3) y E   2
RsenB RsenC RsenA
(x  4) el ángulo medio mide 60°. Hallar
R: circunradio del triángulo ABC.
“x”.
A B C
A) 25 B) 28 C) 30 A) 8sen   .sen   .sen  
D) 37 E) 42 2 2 2
08. En un triángulo ABC se cumple:  
A  
B C
B) 8cos   .cos   .cos  
2bc 2 2 2
a  b2  c 2  C) 8sen(A).sen(B).sen(C)
3
Hallar tan(A) . D) 4sen(A).sen(B).sen(C)
E) 4cos(A).cos(B).cos(C)
3
A) 1 B) 2 C)
3 B C
16. Calcule el valor de tan  
D) 2 2 E) 3  2 
09. En un triángulo ABC se cumple: Si: B  C  120 ; AC  3  1 ; AB  2
a2  b2  c 2  10 A) 2  3 B) 2  3 C) 1  2 3
Hallar: E  bc.cos A  ac.cosB  ab.cosC D) 2 3  1 E) 2 6
A) 10 B) 20 C) 5
D) 15 E) 25 17. En un triángulo ABC, Ĉ  60 y a  3b .
10. En un triángulo ABC se tiene que el Hallar: E  tan(A  B)
semiperimetro es 8 y el producto de las A) 2 3 B) 3 3 C) 4 3
longitudes de los lados es 40. Calcular:
D) 3 E) 3 /2
cos A cosB cosC
E 2  
b  c 2  a2 c 2  a2  b2 a2  b2  c 2 18. En un triángulo ABC, se tiene que:
A) 3/5 B) 1/2 C) 3/2 a  8  b  6  c  5 , determine el valor de:
D) 1 E) 1/5
C
14 tan   csc(A  B)  cot(A  B)
11. En un triángulo ABC, reducir: 2
E  (acosC ccosA)cosC (acosB bcosA)cosB A) 2 B) 14 C) 1
A) a B) b C) c D) 7 E) 3
D) 0 E) a + b + c
19. En un triángulo ABC, calcule el cociente de
12. En un triángulo ABC se sabe que: la división de la suma y la diferencia de dos
AB  c , BC  a , AC  b de sus lados, si las tangentes de la mitad
2
de sus ángulos opuestos son 1 y 0,75
 a  b cosC  respectivamente.
  sen B
2
Reducir: 
 c  A) 15 B) 64 C) 48
A) 0 B) 1 C) 2 D) 49 E) 50
D) 1/2 E) 1/4 20. En un triángulo ABC, se cumple que:
13. En un triángulo ABC, siendo R: circunradio BC
tan  
abcosC bc cosA  cacosB c  bcos A 3  2 
Reducir: E   , calcule:
senA  senB  senC b  c cos A 5  
A
tan  
A) R B) 2R C) 3R 2

Trigonometría 85
Prof: Breyner Oroya Cruz
A) 2/5 B) 3/5 C) 4/7 27. Desde un punto situado al sur de una torre
D) 3/7 E) 1/4 se observa la parte más alta de esta con un
ángulo de elevación de 30º, y desde otro
21. Una persona colocada a 36 m de una torre,
punto situado al Este de la torres el ángulo
observa su parte más alta con un ángulo de
de elevación es de 45º. Hallar la altura de la
elevación de 45°. ¿Qué distancia habría
torre sabiendo que la distancia entre los
que alejarse para que el ángulo de
dos puntos de observación es de 10 m.
elevación sea de 37°?
A) 1 m B) 5 m C) 10 m
A) 12 m B) 6 m C) 8 m
D) 25 m E) 50 m
D) 15 m E) 16 m
22. Desde un punto en la parte superior de un 28. Un automóvil parte a las 6 de la mañana de
muro se observa lo alto de un poste con un una ciudad A y recorre 30 km en dirección
ángulo de elevación de 37º y la base del norte, luego se dirige a una ciudad B
poste con un ángulo de depresión de 16°. girando 60° en dirección noreste,
Si la altura del poste es de 25 m. ¿Cuál es recorriendo 20 km. Determine la distancia
la altura del muro (en m)? entre las ciudades A y B (en km).
A) 3 B) 5 C) 7 A) 10 19 B) 5 19 C) 20 19
D) 9 E) 12
D) 4 19 E) 30 19
23. Desde lo alto de un edificio se ve un punto
en tierra con un ángulo de depresión “α” y a 29. Un móvil recorre 3 km en la dirección
otro punto ubicado a la mitad entre el N50°O, luego recorre “x” km en la dirección
primer punto y el edificio, con un ángulo de S25°O hasta un punto, de donde se
depresión “ 90º  ”. Calcular “ cot  ” observa la posición inicial en la dirección
N40°E. La distancia de la posición inicial a
A) 2 B) 2 2 C) 1 la posición final en km, es:
2 2 A) 9(2  3) B) 3(2  3) C) 6(2  3)
D) E)
4 2
D) 6(2  3) E) 3(2  3) CEPRU 2015–I
24. Un hombre que mide 1,7 m de estatura
observa su sombre a las 4 de la tarde; 30. Desde la parte superior de un faro se
asumiendo que amanece a las 6 a.m., y observan dos puntos situados al sur y al
que el sol hace un circulo sobre el hombre. este con ángulos de depresión de 30° y 45°
¿Cuánto mide su sombra (en m)? respectivamente. Si la distancia entre
A) 1,54 B) 1,67 C) 2,00 dichos puntos es 40 m, calcule la altura del
D) 2,55 E) 2,94 faro.
A) 15 m B) 20 m C) 12 m
25. Desde un punto en tierra se ve lo alto de un D) 10 m E) 8 m
edificio de 4 pisos, con un ángulo de
elevación “α”. Si nos acercamos 1 m, el
ángulo de elevación para lo alto del tercer
piso es “β” y si nos acercamos 2 m más, el
ángulo de elevación para lo alto del
segundo piso es “θ”. Pero si nos acercamos Problemas Dirigidos
3 m más, el ángulo de elevación para lo alto 01. C 02. E 03. B 04. A 05. B 06. B
del primer piso seria de 45°. La altura de 07. A 08. B 09. A 10. A 11. A 12. B
cada piso es 3 m. Calcular: 13. E 14. D 15. A 16. B 17. D 18. B
cot   cot  19. 20. 21. 22. 23. 24.
E C D B A D B
cot   cot  25. A 26. A 27. D 28. D 29. C 30. E
A) 4/5 B) 3/5 C) 1/2
D) 3 E) 1/3 Problemas Propuestos
01. D 02. D 03. B 04. B 05. D 06. E
26. ¿Cuál es la dirección de la bisectriz del 07. D 08. D 09. C 10. E 11. A 12. B
menor ángulo formado por las direcciones
N20°E y S80°O? 13. A 14. A 15. A 16. B 17. C 18. A
A) N10°O B) N20°O C) N30°O 19. D 20. E 21. A 22. D 23. A 24. E
D) N40°O E) N50°O 25. A 26. D 27. B 28. A 29. E 30. B

Trigonometría 86
Definición de Función 2. Función Decreciente
Se dice que y es un función de x, si a cada Una función “f” es decreciente en un
valor de x le corresponde un único valor de y. intervalo x , si para todo x1,x 2  x se
La correspondencia entre estas dos variables
se expresa matemáticamente por medio de una cumple que:
ecuación denominada regla de y
correspondencia la cual se denota de la f(x1)
siguiente forma y  f(x) . y  f(x)
Variable Independiente y Dependiente
El símbolo f(x) se emplea para designar una f(x2 )
función, y se lee f de x. Si y  f(x) entonces a x
la variable se le puede asignar valores a x1 x2
voluntad dentro de los límites que depende del
x
problema particular, a esta variable se le llama
Si: x1  x2  f(x1 )  f(x2 )
variable independiente o el argumento.
A la variable y, cuyo valor queda fijado cuando
se asigna un valor a la variable independiente, 3. Función Continua
se le llama variable dependiente o la función. Sean “f” y “g” dos funciones reales definidas
en un mismo intervalo, pero cuyas graficas se
Dominio de un Función representan del siguiente modo.
Son aquellos valores que admite la variable
independiente, la cual se denota por Dom f o Df y
Rango de un Función
Son aquellos valores que admite la variable f(a) y  f(x)
dependiente, la cual se denota por Ran f o Rf.

Función Real de Variable Real


Se llama función real de variable real a toda a x
correspondencia f : x  f(x) esta ley o criterio
permiten entender que f asocia números reales y
con números reales tal que x  y f(x)  . y  g(x)

Definiciones Previas g(a)


1. Función Creciente
Una función “f” es creciente en un intervalo
x , si para todox1,x 2  x se cumple que: a x
y
f(x 2 ) y  f(x) Notara que en las cercanías de x = a, el
comportamiento de “f” es ininterrumpido y
“continuo”; mientras que en el de “g” presenta
una ruptura, un “salto”, una “discontinuidad” en
f(x1) el punto x =a.
x Una función se llama continua en un punto x =
x1 x2 a de su dominio si en las proximidades de “a”,
f(x) está próximo de f(a). Gráficamente, en el
x punto x = a no deben existir “rupturas” ni
Si: x1  x2  f(x1 )  f(x2 ) “saltos” en la curva que lo representa.

Trigonometría 87
Prof: Breyner Oroya Cruz
y 6. Función Periódica
 Una función “f” se llama periódica cuando
f(a) y  f(x) existe un número real “T” ( T  0 ), tal que
  x  Df ; se cumple:
(x  T)  Df y f(x  T)  f(x)
El número T se denomina un periodo de “f”. El
a  x menor valor positivo de “T” se llamara periodo
f continua en x  a principal o periodo mínimo o periodo de “f”.
Cumpliéndose que todo múltiplo kT, k   0
y es también periodo de “f” pero no es periodo
 y  g(x) principal o mínimo.
g(a) Las gráficas de estas funciones, muestran la
representación de un tramo a lo largo de todo
 su dominio, por ejemplo en los siguientes
esquemas:
a  x
f discontinua en x  a y
2
4. Función Par
Una función “f” se llama par si: x
“x” y “– x”  Df ; además: 4 4
f(x)  f(x);  x  Df
Su grafica es simétrica respecto al eje “y” T4

y FUNCIÓN TRIGONOMÉTRICA
F.T.  (x;y) / y  R.T.(x);x  DF.T.

Esta definición requerirá de algunos conceptos


vistos en circunferencia trigonométrica, para un
x real análisis.

Funciones Trigonométricas
5. Función Impar Función Seno
Una función “f” se llama par si: y
“x” y “– x”  Df ; además: 1 y  sen x
f(x)  f(x);  x  Df
Su grafica es simétrica al origen del mismo
cartesiano. 3/2 2
y 0 /2  x

(x1;sen x1 )
1
T  2
x

F.T.  (x;y)  2
/ y  sen x;x  
Domf  ; Ranf   1;1
Es una función impar: sen(x)   sen(x)
Es una función periódica: T  2
Es una función continua en .
Es una función creciente y decreciente.

Trigonometría 88
Prof: Breyner Oroya Cruz
Función Coseno 
F.T.  (x;y)  2
/ y  tan x;x   n;n  
y Domf   n;n  ; Ranf 
1 Es una función impar: cot(x)   cot(x)
y  cos x Es una función periódica: T  
 Es una función discontinua en .
0  3  2 x Es una función decreciente en su dominio.
2 2
(x1;cos x1) Función Secante
1
T  2 y
y  sec x

F.T.  (x;y)  2
/ y  cos x;x  
Domf  ; Ranf   1;1
1
Es una función par: cos(x)  cos(x)
Es una función periódica: T  2 0 /2  3/2 2 x
Es una función continua en . 1
Es una función creciente y decreciente.
(x1;sec x1 )
Función Tangente
T  2
y
y  tanx
(x;y)  2 / y  sec x; 
 
F.T.     
  x   (2n  1) 2  ;n  
2    
0  3 x
 
2 2 Dom f   (2n  1) ;n  ;Ran f   1;1
(x1;tan x1)  2
Es una función par: sec(x)  sec(x)
T
Es una función periódica: T  2
   
F.T.  (x;y)  2
/ y  tan x;x   (2n  1)  ;n   Es una función discontinua en .
  2  Es una función creciente y decreciente.
 
Dom f   (2n  1) ; Ran f  Función Cosecante
 2
Es una función impar: tan(x)   tan(x) y
y  csc x
Es una función periódica: T  
Es una función discontinua en .
1
Es una función creciente en su dominio.

Función Cotangente 0 /2  3/2 2 x


1
y
y  cot x
(x1;csc x1 )

T  2

0   3 2 x
2 
(x;y) 
2
/ y  csc x;

2 F.T.   
(x1;tan x1) 
 x    ;n 
n  

T Domf   n,n  ; Ranf   1;1

Trigonometría 89
Prof: Breyner Oroya Cruz
Es una función impar: csc( x)   csc(x) y
Es una función periódica: T  2 g( x )  sen(x)  1
Es una función discontinua en .
Es una función creciente y decreciente. f( x)  sen(x)
1

Trazado de Graficas Especiales


Desplazamientos 2
0  x
Desplazamiento Horizontal
Sea la función f( x ) para graficar la función
1
y  f( x  c ) es necesario desplazar la gráfica de f a
lo largo del eje x.
h( x )  sen(x)  1
1. Si c0 desplazamiento hacia la izquierda.
2. Si c0 no hay desplazamiento. Reflejos
3. Si c0 desplazamiento hacia la derecha. Reflejo en el eje X
Sea f( x)  sen(x) ; g( x )  sen  x    Sea la función f( x ) , para graficar la función
 2 
y   f( x ) es necesario reflejar la gráfica de f( x )

y con respecto al eje x.


1 f( x)  sen(x) Sea f( x )  sen x ; g( x )   sen x

y f( x)  sen(x)
3/2 2
/2 0 /2  x 1

2
1
  0 /2  3/2 x
g(x)  sen  x  
 2
Observamos que: g( x )  sen  x     cos x 1
 2 g( x)  sen(x)
Entonces la función coseno es la función seno Reflejo en el eje Y
desplazada horizontalmente en   . Sea la función f(x) para graficar la función
2
y  f(x) es necesario reflejar la gráfica de
Desplazamiento Vertical f(x) con respecto al eje Y.
Sea la función f( x ) para graficar la función
Sea f( x)  sen(x) ; g( x )  sen( x)
y  f( x )  c es necesario desplazar la gráfica de f
a lo largo del eje y. y
f( x)  sen(x)
1. Si c0 desplazamiento hacia abajo. 1

2. Si c0 no hay desplazamiento.



3. Si c0 desplazamiento hacia arriba.  /2 /2 x
Sea: f( x)  sen(x) ; g( x )  sen(x)  1;
h( x )  sen(x)  1
1 g( x)  sen(  x)

Trigonometría 90
Prof: Breyner Oroya Cruz
Dilatación y Compresión 1. Con respecto al eje X solo aquellos
intervalos en donde f( x )  0 y los tramos
Con respecto al eje Y en donde f( x )  0 se mantienen.
Sea la función f( x ) , para graficar
2. Sea la función f( x )  0 , para graficar

y  af( x) ;a 
y  f( x ) es función par; solo es necesario
a 1
graficar para x  0 ya que para x < 0 será
1. Si la gráfica de f( x ) se dilata
verticalmente. solo un reflejo de f( x ) en el eje Y.
2. Si a 1 la gráfica de f( x ) se comprime
Sea f( x )  cos x ; g( x )  cos x
verticalmente.
1
Sea f( x )  tan(x) ; g( x )  2 tan(x) ; h( x)  tan(x) y g( x)  cos x
2
1

y g( x )  2 tan(x)
f( x )  tan(x) 
0  3  2 x
h( x )  0,5 tan(x)
2 2
2
1 f( x )  cos x
1
0,5
  Sea: f( x )  sen x ; g( x )  sen x
0 x
4 2
g( x)  sen x y
f( x)  sen(x)
Con respecto al eje X 1
Sea la función f(x) , para graficar

y  f(ax);a 
 /2 /2  x
1. Si a > 1 la gráfica de f( x ) se comprime
horizontalmente.
2. Si 0 < a < 1 la gráfica de f( x ) se dilata 1
horizontalmente. Observación:
x Si f es una función cuya regla de
Sea f(x)  sen(x) ; g(x)  sen( ) ;
2 correspondencia es:
h(x)  sen(2 x) f( x)  AF.T.n (Bx  C)  D
Dónde:

x A, B, C, D constantes; n 
y
f( x)  sen(x) g(x)  sen  
2 F.T. función trigonométrica
1
Entonces:

0  2 3 x F.T. sen, cos, sec, csc tan, cot


n: par 
T
B 
1 T
h( x )  sen(2x) B
n: impar 2
T
B
Valor Absoluto
Sea la función f(x) , para graficar y  f(x) se
refleja la gráfica de f(x) .

Trigonometría 91
Prof: Breyner Oroya Cruz
0  cos 2 x  1
Problemas Resueltos 0  3 cos 2 x  3
2  2  3 cos 2 x  5
Problema 01: Señale el dominio de la función: y
2cos x  1 2y5
y  f( x) 
sen x  1
Por lo tanto:

2 cos x  1 Rf : 2;5
Solución 01: En la función: y  f( x ) 
sen x  1
Tenemos que: Problema 04: Señale el rango de la función:
senx  1  0  senx  1 y  f(x)  5 sen(x  37º )  cos x
En la C.T. no puede están en el extremo
superior. Solución 04: En la función:
 y  5 sen(x  37º )  cos x
x  (4n  1) ;n 
y  5  sen x.cos37º  sen37º.cos x   cos x
2
Por lo tanto:
   4 3 
Df :  (4n  1) ;n   y  5  .sen x  .cos x   cos x
 2   5 5 
y  4  sen x  cos x 
Problema 02: Señale el dominio de la función: Sabemos que:
y  f( x )  5 sec x  3csc x  2  sen x  cos x  2
4 2  4(sen x  cos x)  4 2
Solución 02: En la función: y

y  f(x )  5 sec x  3 csc x 4 2  y  4 2


1 1 Por lo tanto:
y  f(x )  5.  3.
cos x sen x
Rf :  4 2;4 2 
Tenemos que:  

cos x  0 → En la C.T. no puede están en el Problema 05: Si el periodo de la función


extremo superior e inferior. x
y  f(x)  2sen4  1 es 5π;
senx  0 → En la C.T. no puede están en el a
extremo izquierdo y derecho. Señala el periodo de:
n  
x ;n  y  g(x)  a sec a 1  ax    2 , si a > 0.
2  4
 n 
Df :   ;n   Solución 05: En la primera función:
2 
x
y  f(x)  2sen4  1
a
Problema 03: Señale el rango de la función:
El periodo está dado por:
y  f(x)  2sen2x  5cos2 x

Tf   a  5  a  5
1
Solución 03: En la función:
a
y  f(x)  2sen2x  5 cos 2 x Luego en la segunda función:
 2(1  cos2 x)  5 cos 2 x  
y  g(x)  5 sec 6  5x    2
 2  2cos2 x  5 cos 2 x  4
y  f(x)  2  3 cos 2 x 
El periodo está dado por: Tg 
Pero: 5

Trigonometría 92
Prof: Breyner Oroya Cruz
Problema 06: Señale el periodo mínimo de la Problema 08: Esboce la gráfica de la función f
función: definida por:
x
y  h(x)  sen  sen
x sen6 x  cos 6 x  2
f(x ) 
2 3 sen4 x  cos 4 x  1

Solución 06: En este caso, determinamos el Solución 08: En la función:


periodo de cada función:
sen6 x  cos 6 x  1
x x f(x ) 
y  h(x)  sen  sen sen4 x  cos 4 x  1
2 3
2 2
Tenemos que:
T1  T2 
 1
 
 1
  sen4 x  cos 4 x  1  0
 2 3

T1  4 ; T2  6 1  2sen2x.cos 2 x  1  0
Como son diferentes, se buscan múltiplos de  2sen2x.cos2 x  0
estos periodos hasta que coincidan. sen2x.cos2 x  0
Esto es:
sen x  0 ; cos x  0
T1  4 ; 8 ; 12 ; 16 ; 20 ... En la C.T. no puede estar en ningún extremo.
T2  6 ; 12 ; 18 ; 24 ; 30 ... n
x ;n 
Por lo tanto: Th  12 2
Reduciendo la función tenemos:
sen6 x  cos6 x  1
Problema 07: Determine la ecuación de la f( x) 
sinusoide mostrada. sen4 x  cos4 x  1
1  3 sen2 x.cos2 x  1
f( x) 
y 1  2sen2 x.cos2 x  1
3 3 sen2 x.cos2 x 3
f( x)  
2sen2 x.cos2 x 2
f( x)  1,5
Esbozando tenemos:

x y f(x )  1,5
0 2
1 3
x
Solución 07: Sea la ecuación: 0   3 2
y  f(x)  asenbx  c 2 2
Notamos en el gráfico:
2 2
Tf   b 3
3 b
Luego:
fmáx  3  a( 1)  c ac3
fmín  1  a( 1)  c  c  a  1
c  1; a  2
Por lo tanto:
y  f(x)  2sen3x  1

Trigonometría 93
Prof: Breyner Oroya Cruz
 2 3  
Problemas Dirigidos 07. Si:    ;  ; calcular la suma del
3 2
máximo y mínimo valor que toma la
Tema: Rango de una Función
expresión E  tan(cos )
01. Se tiene la función f(x)  2sen(2018 x)  3
1
Determine el rango de f. A) tan(1) B) tan(-1) C) tan (-1) 
A) 1;5 B) 1;2 C)  3;5 2
1  1
D) 3;7 E)  5;1 D)  E) – tan  
2 2

02. Se tiene la función y  2  5cos2 x , si


08. Determinar el rango de la función
x  , el rango de la función, es:
 5 
A) 3;0 B) 2; 3 C)  3;2 f(x)  sec(x)
; ;  
3 6 2
D)  3;7 E) 7; 3 A) ; 1  2; 

03. Definida la función f por: 2 3


B) ; 2  ; 
f(x)  cot 2 (x)  2csc(x)  2 3
Calcule el rango de f. C) ; 1  1; 
A) 0;  B) 1;  C) 2; 
2 3
D) ;   2; 
D) 3;  E) 4;  3

04. Determine el rango de la función f, definida 2 3


E) ;   2; 
sen(9 x)  sen(3 x) 2
por: f(x) 
cos(3 x)
A) 2;2  0 B) 2;2
Tema: Dominio de una Función
09. Halle el dominio de la función definida por:
C)  2;2 D) 1;1 f(x) 
1
   
E) 2;2  0 sen  x    sen  x  
 3  3
A)  k / k  
05. Si la función f está definida por
x  k 
f(x)  a cos 2    b sen(x) B)  /k 
2 2 
a y b constantes, calcule fmáx.   
C)  (2k  1) / k  
A) a2  4b2 B) a2  b2  a  2 
a 2  b2  k 
C) 4a2  b2 D) D)  /k 
2 3 
a  a2  4b2   
E) E)  (2k  1) / k  
2  3 

06. Señala el rango de la función: 10. Halle el dominio de la función f definida por:
f(x)  (3 tanx  2cot x)2  (tanx  cot x)2 3
f(x)  sen2 (x)  , si x  0;2
A) 5 2  2;  B) 5 2  2;  4
 
    2 4  
A) 0;    ; 
C) 10 2  2;  D) 10 2  2;   3  3 3 
 
 
E) 10( 2  1);  B)  ; 
 3 2

Trigonometría 94
Prof: Breyner Oroya Cruz
  14. Halle los puntos de discontinuidad de la
C)  ;  
3   
función f si: f( x )  tan(   2 x)  tan   2x 
2 
  2   4  5  
D)  ;    ;   (2k  1) 
3 3   3 3  A)  /k 
 4 
    4 5 
E) 0;    ;  B) k / k  
 3  3 3 
 (2k  1) 
C)  /k 
11. Determine el dominio de f, definida por:  2 
3x   k 
f(x)  tan2  5
 4  D)  / k  
4 
 3n 
A)  / n   k 
 4  E)  / k  
2 
 2(2n 1) 
B)  / n 
 3  15. Sea f la función definida por:
 (2n  3)  2sen(x)  5 cos(2 x)
C)  / n  f(x) 
 3   
tan  3x  
 3(2n  1)   5
D)  / n  Determine el dominio de la función f.
 4 
 (5k  2) 
 2n  A)   ; k 
E)  / n   30 
 3 
 (5k  2) 
B)   ; k 
12. Si f es una función real definida por:  30 
f(x)  sen(3x)  7csc(3x)  cos(x)  (10k  3) 
C)   ; k 
Hallar el complemento del dominio de f.  30 
A) 3n / n    (10k  3) 
D)   ; k 
    30 
B) (2n 1) / n  
 6   (2k  1) 
E)   ; k 
 n   2 
C)  / n  
3 
16. La función f definida por:
 n 
D)  / n   f( x)  1  csc(x)  csc(x)  1
8 
E) n / n    (2n  1)
A) 

/ n 
 2 
13. Halle el dominio de la función real f definida  n 
B)  / n  
por f(x)  sec(2x)  csc(2x)  5 4 
 (2k  1)  n  
A)  /k  C)  / n 
 2   2 
 k   (4n 1) 
B)  /k  D)  / n 
2   2 
 k   (4n 1) 
C)  /k  E)  / n 
3   2 
D)  k / k  
 k  Tema: Periodo de un Función
E)  /k  17. Calcule el periodo de:
4 

Trigonometría 95
Prof: Breyner Oroya Cruz
  II. La función y  cos(x) es decreciente
f( x )  2sen  3 x    5
 4 en 0; .
 2 III. La función y  tan(x) es decreciente en
A) B) C) 
3 3 0; /2 .
D) 2 E) 3
¿Cuáles de ellas son verdaderas?
A) Solo I B) Solo II C) Solo III
18. Cuál es el dominio de: f( x)  4 sec 2 (3x)  1 D) I y II E) II y III
  
A) B) C) 24. ¿Cuál de las siguientes proposiciones es
6 4 3 falsa para un ángulo del tercer cuadrante?
2 3 A) El coseno es menor que la secante.
D) E)
3 2 B) La tangente es positiva.
C) La cosecante es negativa.
19. Marque lo incorrecto. D) El seno es mayor que la secante.
El periodo de la función: E) La cotangente es positiva.
A) y  sen(x) es 2
25. Al evaluar las proposiciones, indicar el valor

B) y  tan(3x) es de verdad (V) o falsedad (F):
3 I. La función secante es impar.
 
C) y  sen(4x) es II. El periodo de la función cosecante es
2 2
2 III. El dominio de la función secante está
D) y  csc(3x) es
3  (2n  1) 
dada por:   / n 
E) y  sec(2x) es 4  2 
IV. El rango de la función cosecante está
20. Calcule el periodo de: dado por:  1;1
f( x )  cos(4 x)  sen(2 x) La secuencia correcta, es:
A) VFVF B) FFVV C) FVFV

A) B)  C) 2 D) VFFV E) VVFF CEPRU 2017–II
2
D) 4 E) 8 26. La ecuación de la gráfica adjunta:
y  a  bsenx . Hallar: a + b
21. Calcule el periodo de la función f si:
x  2x  y
f( x )  3 tan(x)  4 tan    5 tan  
2  3  2
A) 2 B) 4 C) 6
D) 8 E) 10
1
22. Sea la función f definida por:
cos(x)
f( x )  sen(x)
Calcule el periodo mínimo de f.
0   3
  3 2 x
A) B) C) 2 2
2 4 2
D)  E) 2 A) 2 B) 3 C) 1
D) 4 E) 6
Tema: Grafica de una Función Básica
 5 
23. Dada las proposiciones: 27. Del gráfico, calcule 2f  
I. La función y  sen(x) es creciente en  6 

0; .

Trigonometría 96
Prof: Breyner Oroya Cruz
y f(x)  A sen(Bx)  1
3 A)  3;3 B)  2;2 C)  1; 
 2
 1  1 1
D) 0;  E)   ; 
 2  2 2
2 x 02. Determine el rango de la función f definida
3
por: f(x)  sen2 (x)  6 sen(x) ; x 

3
A)  9; 2 B) 8; 1 C)  5;0
D)  5;1 E)  3;9
A) 3 B) 2 C) 4
D) 5 E) 6
03. Determine el rango de la función f definida
28. Del gráfico, calcule la regla de por: f(x) 
cos(x)
1 
1
correspondencia de la función f. cos(x)  3 2
y 3 5 5 
f(x)  Acos(Bx)  D A)  ;2 B)  ;2 C)  ;2
8 4 4 4 
3 
D)  ;2 E) 0;2
4 

04. Calcule aproximadamente el máximo valor


2 de la función f definida por:
f(x)  sen(x)  cos(x)  sen(x).cos(x)
  x
4 2 1 2 3 2 32 2
A) B) C)
A) f( x )  5 cos(4 x)  3 2 2 2
23 2 32 2
B) f( x )  3 cos(2 x)  5 D) E)
2 4
C) f( x )  5 cos(2 x)  3
05. Determine el rango de la función f definida
D) f( x )  cos(4 x)  6  3 x  3
por: f(x)  sen     ; x  ;
E) f( x )  3 cos(4 x)  5  4 2 2

1 2 2
 2 n  1 
A) 0; B) 0; C) 0;
29. El punto  ;  pertenece a la gráfica
2 2 4
 3 n  1
de la función y  cos(x) , calcular “n”. 2 3
D) ;1 E) ;1
A) 1/3 B) – 1/3 C) 1/2 2 8
D) – 1/2 E) 1/4
06. Señala el rango de la función:
30. Indique cuantas soluciones presenta la 
f(x)  tan x  4 cot x ; x  0;
ecuación: 2
3 
2cos(x)  3 tan(x)  0 , si x   ;2 A) B) 4; C) 2;
2
A) 1 B) 2 C) 3 D) 4;  E) 2; 
D) 4 E) 5
07. Calcule el rango de la función f, definida
Problemas Propuestos 
por: f(x)  sen  2
 

 x 1
01. Determine el rango de la función f definido
sen(x)  1 A) 0;1 B) 0;1 C) 0;1
por: f(x) 
sen(x)  3 D) 0; 2  E) 0;2 2 

Trigonometría 97
Prof: Breyner Oroya Cruz
08. Se define la función f(x) tal que: 13. Sea la función:
 2 f(x)  sen2 (x)  tan(x).cot(x)  cos2 (x),n 
f(  x)  2f(  x)  cos(x) ; x 
3 3 Determine los puntos de discontinuidad.
Señale el rango de la función f(x) . A) 2n B) (2n  1) C)  
 3 3  3 3  n   n 
A)   B)   D)   E)  
;  ;  2 4
 3 3   2 2 
 2 2  1 1  3x  
C)   ;  D)   ;  14. Sea la función: f(x)  csc    1
 2 2   2 2  4 2
E)  1;1 Determine su dominio.
A)  2n;n  
09. Señale el dominio de la función f, definida  n 
B)   ;n  
por: f(x) 
sen(3x)
 1 ; n 3 
sen(x)  2n 
C)  ;n  
 (4n  1)   3 
A)  2n B)  
 2   2 
D)  (2n  1) ;n  
 (4n  3) 
C)   D)  (2n 1)  3 
 2  2
 
 (2n  1) ;n  
E)  n E)
 3 

10. Señale el dominio de la función f, definida 15. Determine el dominio de la función f


1  cos(x) 1  cos(x) definida por:
por: f(x)   ; n
1  cos(x) 1  cos(x) f(x)  2tan(2x)  3cot(x)  1;k 
 (2n  1)   (4n  1)   k 
A)   B)   A)   B)  k
 4   4  4
C)  2n D)  (2n 1)  k 
 
 (2k  1) 
C) D)  
E)  n 2  2 
 (2k  1) 
E)  
11. Se define la función: f(x)  2sec(3x)  1  4 
¿Cuál es su dominio para todo n ?
 n  16. Sea la función f definida por:
A)  2n B)    csc(  cos(3x))
3 f(x)  , k
csc(x).sec(x)
 n   
C)   D)  (2n  1)  Determine el dominio de f.
6  3
 k   k   k 
  A)    B)    C)  
E)  (2n  1)  2 3 4
 6
 k   k 
D)    E)   
12. Hallar el dominio de la función f definida 6 8
por:
f(x)  1  cos(x)  cos(x)  1;k  17. Determine el periodo mínimo para la
función:
     x  
A) (2k  1)  B) (4k  1)  f(x)  2015 csc  
 2  2   2013
 2016 2014 
     
C) (2k  1)  D) (4k  1)  A) B) C) 1008
 4  4 1008 2016
E) 2k D) 2016 E) 4032

Trigonometría 98
Prof: Breyner Oroya Cruz
18. Determine el periodo mínimo para la y
función:
 f(x)
 2x   4x  P( ;a)
f(x)  sen    cos   8
 3   5 
A) 2 B) 5 C) 10
x
D) 15 E) 30
5
19. Determine el periodo mínimo para la Q( ;b)
8
función h definida por:
3  3x  4  4x  2 3 2
h(x)  cot    tan   A) B) C) 2
 4   3  2 4
A) 6 B) 8 C) 10 2
D) 12 E) 14 D)  2 E) 
2

20. Sea f una función definida por: 25. La grafica muestra una función f, cuya regla
 8x  2  2x  de correspondencia es: f(x)  A  Bsec(x)
f(x)  4sen3    3 cos  
 3   5  Calcule A – B.
a y
Periodo mínimo tiene la forma ;
2
2
entonces, el valor de 2a, es:
A) 15 B) 18 C) 30
D) 36 E) 40

21. Calcule el periodo mínimo de la siguiente


0  4/3 x
función: f(x)  2  3cos
sen(x) 2
(x)


A)  B) C) 2 A) – 2 B) – 6 C) 1
2
D) 2 E) 6
3 
D) E)
2 3 26. La ecuación de la gráfica adjunta es:
f(x)  asen(bx) ; además las coordenadas
22. Determine el periodo mínimo dela función f
 10 
definida por: de “P” es:  ; 6  .
 3 
  x   x  
f(x)  sec  sen   .cos   
2  4   4  y
A) 1 B) 2 C) 3
D) 4 E) 5

 2 
23. Si el punto P  ;5n  pertenece a la
 3  0 25 x
gráfica de la cosinusoide halle “n”. 4
1 5 1 P
A) B) C) 
10 2 10
5 1 Calcular: 5b  2a
D)  E) 
2 2 A) 2 B) 4 C) 6
D) 8 E) 10
24. El grafico tienecomo regla de
1
correspondencia: f(x)  cos(2 x) 27. Calcule el área de la región sombreada.
2
Halle: 4a + b

Trigonometría 99
Prof: Breyner Oroya Cruz
y
f(x)  sen(x)
Problemas Dirigidos
01. A 02. C 03. A 04. A 05. E 06. E
07. B 08. D 09. A 10. D 11. B 12. C
0 5 x 13. E 14. D 15. A 16. D 17. B 18. C
6
19. E 20. B 21. C 22. E 23. E 24. A
25. B 26. A 27. E 28. E 29. B 30. C
Problemas Propuestos
  3 2 01. D 02. C 03. C 04. C 05. B 06. B
A) u2 B) u2 C) u
8 4 8 07. C 08. A 09. E 10. E 11. E 12. E
  2 13. D 14. D 15. E 16. D 17. E 18. D
D) u2 E) u
2 16 19. D 20. C 21. A 22. B 23. C 24. B
25. E 26. C 27. C 28. C 29. B 30. B
28. El gráfico adjunto corresponde a la función:
y
6

2
6 x

x x
A) 2sen   B) 2  4sen  
3 3
x
C) 4  2sen   D) 2  4sen  3x 
3
E) 4  2sen  3x 

29. Sea la función definida por


f(x)  tan(x) cos(x) , indique el valor de
verdad de las siguientes proposiciones.
I. El rango de f es 1;1

II. f es creciente en ;
2
3
III. f es decreciente en ;2
2
A) VFV B) VVV C) VVF
D) FVV E) FFV

30. Si x   1;1 es el dominio de la función f


cuya regla de correspondencia.
 x 
f(x)  x 2  sec   , determine su rango.
 3 
A)  3;3 B) 1;3 C)  1;3
D) 1;2 E) 0;1

Trigonometría 100

También podría gustarte